You are on page 1of 242

Item: 1 of 3 7 ~ 1 • M k -<:J 1>- Jil ~· !

:';-~
QIO: 1479 ..L ar Pre v ious Next Lab fli!ltues Not es Calcula t o r

•1 & &

A 77-year-old man presents to his primary care physician for a routine wel lness check. Upon a review of
.2
systems, the patient's only complaint is shortness of breath. Social history is significant for a 50-pack-year
•3 smoking history. On physica l examination, his vital signs are within normal limits except for his blood
·4 pressu re, which is 170/95 mm Hg. On his previous visit, his blood pressure was 155/ 90 mm Hg. His physical
•5
examination is also significant for the condition of his hand, shown in the image, as well as wheezing on pulmonary
examination.
•6 .....-~~----~~----~----~

.7 ......
·8
.9
• 10
• 11

• 12
• 13
• 14
• 15
• 16
• 17
. 18
• 19 •
I mage courtesy of James Heilman, MD
• 20
• 21

a
Lock
s
Suspend
8
End Bl ock
• 10 •
Image courtesy of James Heilman, MD
• 11
• 12
Which of the following antihypertensive agents is relatively contraindicated in this patient?
• 13
:
• 14
A. Bisopro lol
• 15
• 16
B. Furosem ide
• 17 C. Lisinopril
• 18 D. Nadolol
• 19
E. Prazosin
• 20
• 21

a
Lock
s
Suspend
8
End Block
Item: lof37 ~. , . M k <:] t> al ~· ~
QIO: 1479 .l. ar Previous Next lab 'lifllues Notes Calculator

1 •

•2
The correct answer is D. 39°/o chose this.
Nonselective ~-b l ocke rs are contraindicated in patients with lung disease because they can cause
•3
bronchoconstriction by blocking ~ 2 -receptors responsib le for relaxation of bronchia l smooth muscle.
•4 Nonselective ~-b l ocke rs are also contraindicated in cocaine overdose due to unapposed a-agon ism, leading to
•5 severe vasoconstriction. Nadolol is a nonselective ~-blocker and shou ld not be used in a patient with lung
•6
disease . Other nonse lective ~-b l ockers include propranolol, timo lol, and pindolol. Acebutolol, atenolol, esmolol,
metop rolo l, and betaxolol are ca rdioselective ~ 1 -b l ockers that should be favo red in patients with lung/airway
•7 disease . Although the vignette does not specifical ly state that th is patient has lung disease, long-time smoking
•8 causes airway hyperreactivity and bronchoconstriction that has likely resu lted in chron ic obstructive pu lmona ry
•9 disease, as evidenced by his pu lmonary exam f indings and the cyanosis evident in the image. Adding a
nonse lective ~ - b l ocke r could exagge rate these adverse effects of smoking .
• 10
Nadolol Propranolol Atenolol Metoprolol Esmolol Pindolol Chronic obstructive pulmonary disease Timolol Cyanosis Vasoconstriction Betaxolol Acebutolol Cocaine
· 11
Smooth muscle tissue Respiratory tract Bronchoconstriction Bronchus Drug overdose Muscle lung
• 12
A is not correct. 17% chose this .
• 13
Bisopro lol is a cardioselective ~ 1 - adrenerg i c receptor blocker, which can be used with caution in patients with
• 14
asthma or othe r lung disease. It is not a first-choice therapy because it is ca rdioselective but not cardiospecific
• 15 (so it will still have some effect on ~rreceptors in airways), but it is not the wo rst option among these answer
• 16 choices and can be used if needed to treat a patient with history of asthma or bronchospasm .
Bisoprolol Bronchospasm Asthma Respiratory disease
• 17

• 18 B is not correct. 11% chose this .


• 19 Furosemide is a loop diuretic and can be used in patients with asthma or other obstructive lung diseases . It is
• 20
associated with ototoxicity and can lead to gout, which is caused by hyperuricemia. However, no respi ratory
side effects are associated with fu rosemide use .
• 21 loop diuretic Furosemide Hyperuricemia Diuretic Ototoxicity Gout Asthma Adverse drug reaction lung

6
lock
s
Suspend
0
End Block
Item: lof37 ~. , . M k <:] t> al ~· ~
QIO: 1479

1
.l.
. ..ar
..
Previous
. .
Next
.
lab 'lifllues Notes Calculator

•2
Bisopro lol is a cardioselective ~radrenerg i c receptor blocker, which can be used with caution in patients with
asthma or othe r lung disease. I t is not a first-choice therapy because it is ca rdioselective but not cardiospecific
•3 (so it will still have some effect on ~rreceptors in airways), but it is not the wo rst option among these answer
•4 choices and can be used if needed to treat a patient with history of asthma or bronchospasm .
Bisoprolol Bronchospasm Asthma Respiratory disease
•5
•6 B is not correct. 11% chose this .
•7 Furosemide is a loop diuretic and can be used in patients with asthma or other obstructive lung diseases . It is
•8 associated with ototoxicity and can lead to gout, which is caused by hyperuricemia. However, no respi ratory
side effects are associated with fu rosemide use .
•9
loop diuretic Furosemide Hyperuricemia Diuretic Ototoxicity Gout Asthma Adverse drug reaction lung
• 10
C is not correct. 16% chose this.
· 11
Lisinopri l is an angiotensin-converting enzyme inh ibitor (ACEI ) and can be used to treat hypertension. It can
• 12
also be used in patients who have recently had heart attacks, but it is contraindicated in patients with kidney
• 13 disease . Though dry cough is a common side effect of the med ication, lisinop ril does not worsen ch ronic lung
• 14 diseases such as asthma and ch ronic obstructive pu lmona ry disease and is safe to use in such patients .
ACE inhibitor lisinopril Chronic obstructive pulmonary disease Angiotensin-converting enzyme Enzyme inhibitor Enzyme Asthma Hypertension Adverse effect
• 15
Kidney Cough Kidney disease Myocardial infarction Contraindication Side effect Pharmaceutical drug
• 16
• 17 E is not correct. 17% chose this .
• 18 Prazosin is an a1 -selective blocke r that can be used in patients with asthma or other obstructive lung diseases .
• 19
Prazosin is famous, however, for resu lt ing in a "f irst-dose response, " in wh ich fainting due to orthostatic
hypotension is pronounced when patients ingest the first dose of the drug. Howeve r, there are no respi rato ry
• 20
side effects associated with prazosin use .
• 21 Prazosin Orthostatic hypotension Hypotension Asthma Syncope (medicine) Side effect

6
lock
s
Suspend
0
End Block
Item: lof37 ~. , . M k <:] t> al ~· ~
QIO: 1479 .l. ar Previous Next lab 'lifllues Notes Calculator

1
- - - - .. -...... . - - - -- ... --- .. ... - -· ... - .. ... .. ~ - . -- .
side effects are associated with fu rosemide use.
•2 loop diuretic Furosemide Hyperuricemia Diuretic Ototoxicity Gout Asthma Adverse drug reaction lung

•3
C is not correct. 16% chose this.
•4
Lisinopri l is an angiotensin-converting enzyme inh ibitor (ACEI ) and can be used to treat hypertension. It can
•5 also be used in patients who have recently had heart attacks, but it is contraindicated in patients with kidney
•6 disease . Though dry cough is a common side effect of the med ication, lisinop ril does not worsen ch ronic lung
•7
diseases such as asthma and ch ronic obstructive pu lmona ry disease and is safe to use in such patients.
ACE inhibitor lisinopril Chronic obstructive pulmonary disease Angiotensin-converting enzyme Enzyme inhibitor Enzyme Asthma Hypertension Adverse effect
•8
Kidney Cough Kidney disease Myocardial infarction Contraindication Side effect Pharmaceutical drug
•9
• 10 E is not correct. 17% chose this .
· 11
Prazosin is an a1 -selective blocke r that can be used in patients with asthma or other obstructive lung diseases.
Prazosin is famous, however, for resu lt ing in a "f irst-dose response, " in wh ich fainting due to orthostatic
• 12
hypotension is pronounced when patients ingest the first dose of the drug. Howeve r, there are no respi rato ry
• 13 side effects associated with prazosin use .
• 14 Prazosin Orthostatic hypotension Hypotension Asthma Syncope (medicine) Side effect

• 15
• 16 Bottom Line:
• 17
Be carefu l when prescribing nonselective ~-bloc kers to patients with lung disease or airway hyperreactivity,
• 18 incl uding ind ividuals with extensive smoking histories. Se lective ~ 1 -b l ocke rs should be favored in these
• 19 patients .
Respiratory disease lung Respiratory tract
• 20
• 21

6
lock
s
Suspend
0
End Block
Item: 1 of 3 7 ~ 1 • M k -<:J 1>- Jil ~· !:';-~
QIO: 1479 ..L ar Pre v ious Next Lab fli!ltues Not es Calcula t o r
& &
1
.2 FA17 p237.1

•3 P-blockers Acebutolol, atenolol, betaxolol, bisoprolol, carvedilol, esmolol, labetalol, metoprolol, nadolol,
·4 nebivolol, pindolol, propranolol, timolol.
APPLICATION AGIONS NOTES/EXAMPLES
•5
•6 Angina pectoris ' heart rate and contractility, resulting in ' 0 2
consumption
.7
Myocardial infarction ' mortality
·8
Supraventricular ' AV conduction ,-elocity (class II ~ letoprolol, esmolol
.9
tachycardia antiarrhythmic)
• 10
Hypertension ' cardiac output, ' renin secretion (due to
• 11
~ 1 -receptor blockade on JGA cells)
• 12
Heart failure ' mortality (bisoprolol, c<uved ilol, metoprolol)
• 13
Glaucoma l production of aqueous humor Timolol
• 14
Variceal bleeding ' hepatic venous pressure gradient and portal adolol, propranolol
• 15
hypertension
• 16
AOVERSE EFFECTS Erectile dysfunction, cardiovascu lar adverse Use with caution in cocaine users due to risk
• 17
effects (bradycardia, AV block, II F'), CNS of unopposed a-adrenergic receptor agonist
. 18 adverse effects (seizures, sedation, sleep activity
• 19 alterations), dyslipidcmia (mctoprolol), and
• 20 asthma/COPD exacerbations
• 21 SUECTIVITY ~rselective antagonists (~ 1 > ~) - acebutolol Selective antagonists mostly go from A to ::\1 (~ 1

a
Lock
s
Suspend
8
End Bl ock
Item: lof37
QIO: 1479

1
•2
~ -, . M k
.l. ar

SELECTIVITY
<:]
Previous
t>
Next

... al
lab 'lifllues
'
~ 1 -selective antagonists (~ 1

Notes
~
Calculator

> ~) - acebutolol Selective antagonists mostly go from A to M (~ 1


(partial agon ist), atenolol, betaxolol, bisoprolol, with l st hal f of alphabet)
•3
esmolol, m etoprolol
•4
Nonselective antagonists (~ 1 = ~2 ) -n adolol, N onselective antagon ists mostly go from N to Z
•5
pindolol (partial agonist), propranolol, t imolol (~2 with 2nd half of alphabet)
•6
Nonselecti,·e ex- and ~-a ntagonists - can·edi l ol , lonselective ex- and ~-antagon i sts ha,·e modified
•7
labctalol suffi xes (instead of "-olol")
•8
Nebivolol combines c01rdiac-selective
•9 ~ 1 -a dren ergic blockade with stimulation of
• 10 ~rreceptors (activate nitric oxide synthase in
· 11 the vasculature and ! SVR)
• 12
• 13 FA17p234.1
• 14 Sympathomimetics
DRUG ACTION APPLICATIONS
• 15
• 16 Direct sympathomimetics
• 17 Albuterol, salmeterol ~z> ~~ Albuterol for acute asthma or COPD. Salmeterol
for long-term asthma or COPD control.
• 18
• 19 Dobutamine ~~> ~'a Heart fail ure (llF) (inotropic > chronotropic),
cardiac stress testing.
• 20
• 21
Dopamine 0 1 = Dz> ~ > <X Unstable bradycardia, HF, shock; inotropic and
• r-hr(")nntr n n ir pff prtc: "3t lnu •p r rl ncpc rlnP tn R •

lock
6 s
Suspend
0
End Block
Item: 1 of 3 7 ~ 1 • M k -<:J 1>- Jil ~· !:';-~
QIO: 1479 ..L ar Pre v ious Next Lab fli!ltues Not es Calcula t o r
& &
1
•2 FA17 p234.1
•3 Sympathomimetics
DRUG ACTION APPLICATIONS
·4
•5 Direct sympathomimetics
•6 Albuterol, salmeterol l3z > p, Albuterol for acute asthma or COPD. Salmetcrol
•7
for long-term asthma or COPD control.

·8
Dobutamine p, > 13z, <X llearl failure (HF) (inotropic> chronotropic),
cardiac stress testing.
.9
• 10
Dopamine o, = 0 2> p> <X Unstable bradycardia, HF, shock; inotropic and
chronotropic effects at Jm,·er doses due to p
• 11
effects; vasoconstriction at high doses due to <X
• 12 effects.
• 13 Epinephrine Anaphylaxis, asthma, open-angle glaucoma;
P> a
• 14 <X effects predominate at high doses.
• 15 Significantly stronger effect at 13z-receptor than
• 16
norepinephrine.
• 17 Fenoldopam o, Postoperative hypertension, hypertensive crisis.
Vasodilator (coronary, periphera l, renal, and
• 18
splanchnic). Promotes natriuresis. Can cause
• 19
hypotension and tachycardia.
• 20
Isoproterenol p, = l3z Electrophysiologie evaluation of
• 21
• . .
laclwarrhvthmias. Can \\'Orsen ischemia .

a
Lock Suspend
s 8
End Bl ock
Item: 1 of 3 7 ~ 1 • M k -<:J 1>- Jil ~· !:';-~
QIO: 1479 ..L ar Pre v ious Next Lab fli!ltues Not es Calcula t o r
&
- - - -
&
1
Midod rine Autonomic insufficiency and postmal
.2
hypotension. May exacerbate supine
•3 hypertension.
·4 Mirabeg ro n Urinary urge incontinence or O\ eractive bladder.
•5
No re p ine phrine llypotension, septic shock.
•6
.7 Phenylephrine l lypotension (\·asoconstrictor), ocular procedures
·8 (mydriatic), rhinitis (decongestant).
.9 Indirect sympathomimetics
• 10 Ampheta mine Indirect general agonist, reuptake inhibitor, also 'arcolepsr, obesity, ADHD.
• 11 releases stored catecholamines
• 12 Co caine Indirect general agonist, reuptake inhibitor Causes \'asoconstriction and local anesthesia.
• 13 'c\·er gi\·e ~-bl ockers if coca ine intoxication is
• 14 suspected (can lead to unopposed a 1 activation
and extreme hypertension).
• 15
Ephe drine Indirect general agonist, releases stored asa l decongestion (pseudoephedrine), urinary
• 16
catecholamines incontinence, hypotension.
• 17
. 18
• 19 FA17 p638.1

• 20 Obstructive lung Obstruction of air Aow ..... air trapping in lungs. Airwa)S close prematurely at high lung \'olumes
• 21
d isea ses --: .t RV and t F'RC, t TLC. PF'Ts: l l FE 1, l FVC ..... l F EV 1/FVC ratio (hallmark),
• \ //r'\ ro.... :,.....,...,...,.&--...,.J...,. ('J..,. .,."...,;,.. 1.... ,,....,..,, ;,.. ...... ,J •.,.,. ,...,., ..,..,.. , • .....,,.,.. ,..,.. •.,.,.t-.. :,..a.-:,.....-. ....,,......,. ) ,.......,,1 &--.,...., ,....... .,. .-.. •• J...,....,.. •..,. .... )...,. r""J.., ......,,...,;_

a
Lock Suspend
s 8
End Bl ock
Item: lof37 ~ ., . M k <:] t> al ~· ~
QIO: 1479 .l. ar Previous Next lab 'lifllues Notes Calculator

1 • •
FA17 p638.1
•2
Obstructive lung Obstruction of air fl ow - air trapping in lungs. Airways close prematurely at high lung volumes
•3 diseases - t RV and t FRC, t TLC. PFTs: U FEV 1, l F'VC - l FEV 1/F'VC ratio (ha llmark),
•4 V/Q mismatch. C hronic, hypoxic pulmonary vasoconstriction can lead to cor pulmonale. Chronic
•5 obstructive pulmonary disease (COPD) includes chronic bronch itis and e mphysema.
TYPE PRESENTATION PATHOLOGY OTHER
•6
Chronic bronchitis Findings: wheezing, c rackles, Hypertrophy and hyperplas ia Diagnostic crite ria: productive
•7
(" blue bloater") cyanosis (hypoxemia due of mucus-secreting glands cough for > 3 months in a
•8
to shunting), dyspnea, C02 in bronchi - Reid index year for> 2 consecutive years.
•9 retention, 2° po lycythemia. (th ickness of mucosal gla nd
• 10 layer to thickness of wall
· 11 between epithe lium a nd
cartilage) > 50%.
• 12
• 13
Emphysema ("pink Centriacinar-associated with Enlargement of air spaces CXR: t AP diameter, flattened
puffer" ) sm oking rJ (!]. Freque ntly in l recoil, t com pi iance, diaphragm, t lung field
• 14
upper lobes (smo ke rises up). l DLCO from destruction of lucency.
• 15 Pa nacinar- associated with alveolar walls (arrow in Ej). Barrel-shaped chest (!].
• 16 a .-antitrypsin d efic iency. t elastase activity - t loss Exhalation th rough pursed lips
• 17 Frequently in lower lo bes. of e lastic fibers - t lung to increase airway pressure
• 18
compliance. and prevent airway collapse.

• 19 Asthma Findings: cough, wheezing, Bronchial hyperresponsiveness Aspirin-induced asthma: COX


tachypnea, dyspnea, -+ reversible inhibition -+ le ukotriene
• 20
hypoxe mia, l inspirato ry/ bronc hoconstriction. Smooth o\·erpro duction -+ a irway
• 21 •
• exoirato rv ratio. oulsus muscle hvo ertroohv and constric tion. Associated with

6
lock
s
Suspend
0
End Block
Item: 2 of 3 7 ~ 1 • M k -<:J 1>- Jil ~· !:';-~
QIO: 1755 ..L ar Pre v ious Next Labfli!llues Not es Calcula t o r
& &
1
A 69-year-old man is hospitalized for an exacerbation of asthma . He is placed on albuterol and an inhaled
•2
corticosteroid, but due to low oxygen saturation, he is intubated. After 3 days in the hospital, he has a
•3 temperature of 39.4° C ( 103.1 ° F) , a blood pressure of 104/63 mm Hg, a pulse of 108/min, and a respiratory
·4 rate of 35/min. Physical examination reveal coarse rhonchi at the base of the lung fields bi laterally. The patient is
•5
producing purulent, foul-smelling sputum that grows many oxidase-positive gram-negative rods on culture.
•6
Which of the following agents could be used to treat this infection?
•7

·8 :
A. Cefazolin
.9
• 10 B. Ceftazidime
• 11 C. Erythromycin
• 12
D. Penicillin
• 13
• 14
E. Vancomycin
• 15
• 16
• 17
• 18
• 19
• 20
• 21

a
Lock
s
Suspend
8
End Bl ock
Item: 2 of 37 ~. , . M k <:] t> al ~· ~
QIO: 1755 .l. ar Previous Next lab 'lifllues Notes Calculator

1 •

2 The correct answer is B. 58°/o chose this.


•3
The organism and culture results described in the vignette are suggestive of Pseudomonas pneumonia. Classic
features of Pseudomonas pneumonia specific to this vignette include the fact that the patient was intubated for
•4
3 days and has lung findings on physical exam, as Pseudomonas is a very common cause of ventilator-
•5 associated and hospital-acquired pneumonias. The morphology and oxidase positivity are also supportive of
•6 Pseudomonas. The classic "fruity odor" and "blue-green pigment" of Pseudomonas are more typical of skin and
soft tissue infections and in laboratory colonies than with Pseudomonas pneumonias . Fou l-smelling sputum in
•7
this scenario is not referring to a "fruity odor" but rather refers to fou l smel ling sputum indicative of pneumon ia
•8 infections. Cephalosporins are ~-lactam antibiotics that inhibit bacterial cel l wal l synthesis . They are generally
•9 well tolerated and easy to administer; however, adverse effects include anaphylaxis and nephrotoxicity when
• 10
combined with aminoglycosides. Three cephalosporins may be used to treat Pseudomonas aeruginosa infection
(and shou ld be memorized) : cefoperazone, ceftazid ime, and cefepime . Cefoperazone and ceftazidime, third-
· 11 generation cephalosporins, are simi lar except that ceftazidime has better activity against the Enterobacteriaceae
• 12 family and is more common ly used . Cefepime is a fourth-generation cephalosporin with activity against both
• 13 gram-positive and gram-negative organisms, includ ing P. aeruginosa. The extended-spectrum penicillins
(ticarcil lin, carben icill in, and piperacillin), given with an aminoglycoside, are also effective against Pseudomonas .
• 14
Cephalosporin Cefepime Aminoglycoside Piperacillin Ceftazidime Enterobacteriaceae Carbenicillin Pseudomonas aeruginosa Anaphylaxis Cefoperazone
• 15
Gram-negative bacteria Gram-positive bacteria Nephrotoxicity Pneumonia Sputum Cell wall Penicillin Pseudomonas Antibiotics Oxidase lung Organism
• 16
Tracheal intubation Hospital-acquired infection
• 17

• 18
A is not correct. 13% chose this .
• 19
Cefazolin is a first-generation cephalosporin that has excellent activity against gram-positive and covers some
gram-negative rods, includ ing Proteus mirabilis, Escherichia coli, and Klebsiella pneumoniae. Cefazol in is not
• 20 used for Pseudomonas infection .
• 21 Cephalosporin Klebsiella pneumoniae Escherichia coli Cefazolin Proteus mirabilis Gram-negative bacteria Gram-positive bacteria Pseudomonas Proteus (bacterium)

6
lock
s
Suspend
0
End Block
Item: 2 of 37 ~. , . M k <:] t> al ~· ~
QIO: 1755 .l. ar Previous Next lab 'lifllues Notes Calculator

1 •
A is not correct. 13% chose this.
2 Cefazolin is a first-generation cephalosporin that has excellent activity against gram-positive and covers some
•3 gram-negative rods, includ ing Proteus mirabilis, Escherichia coli, and Klebsiella pneumoniae. Cefazol in is not
•4
used for Pseudomonas infection .
Cephalosporin Klebsiella pneumoniae Escherichia coli Cefazolin Proteus mirabilis Gram-negative bacteria Gram-positive bacteria Pseudomonas Proteus (bacterium)
•5
Klebsiella Proteus Infection
•6
•7 C is not correct. 17% chose this.
•8 Erythromycin is a macrolide that inhibits protein synthesis by blocking translocation . While erythromycin can be
used in pneumonias, it is used mainly in atypical pneumonias including Mycoplasma, Legionella, and Chlamydia
•9
pneumon ia. Erythromycin is not used for Pseudomonas infection .
• 10 Erythromycin Macrolide legionella Pneumonia Chlamydia (genus) Mycoplasma Pseudomonas Protein Protein biosynthesis Chlamydia infection Protein synthesis

· 11 Infection

• 12
D is not correct. 6°/o chose this .
• 13
Penicll in is a bactericidal antibiotic used to treat gram-positive infections as well as infections by gram-negative
• 14 cocci . I t is not effective against gram-negative rods such asP. aeruginosa. Penicillin acts by bind ing pen icill in-
• 15 binding proteins, thereby blocking transpeptidase cross-l inking of the bacteria l cell wall and leading to the
activation of autolytic enzymes .
• 16
Penicillin Antibiotics Gram-negative bacteria Gram-positive bacteria Cell wall Penicillin binding proteins Coccus Pseudomonas aeruginosa Autolysis (biology)
• 17
Bactericide Peptidyl transferase Enzyme 00-transpeptidase Protein
• 18
• 19 E is not correct. 6°/o chose this .
• 20
Vancomycin is a bactericidal antibiotic that inh ibits cell wall mucopeptide formation. I t is used for serious
infections with gram-positive multidrug -resistant organisms including meth icillin-resistant Staphylococcus
• 21 aureus and Clostridium difficile. Vancomycin is not used to treat Pseudomonas infection .

6
lock
s
Suspend
0
End Block
Item: 2 of 37 ~. , . M k <:] t> al ~· ~
QIO: 1755

1
.l.
p
ar
y
Previous
y
Next

. lab 'lifllues
- . Notes Calculator

Erythromycin Macrolide legionella Pneumonia Chlamydia (genus) Mycoplasma Pseudomonas Protein Protein biosynthesis Chlamydia infection Protein synthesis
2
Infection
•3
•4 D is not correct. 6°/o chose this.
•5 Penicll in is a bactericida l antibiotic used to treat gram-positive infections as we ll as infections by gram-negative
cocci . I t is not effective against gram-negative rods such asP. aeruginosa. Pen ici llin acts by bind ing pen ici llin-
•6
binding proteins, thereby blocking t ranspeptidase cross-l inking of the bacte ria l cell wa ll and leading to the
•7 activation of autolytic enzymes.
•8 Penicillin Antibiotics Gram-negative bacteria Gram-positive bacteria Cell wall Penicillin binding proteins Coccus Pseudomonas aeruginosa Autolysis (biology)

•9 Bactericide Peptidyl transferase Enzyme 00-transpeptidase Protein

• 10 E is not correct. 6°/o chose this.


· 11 Vancomycin is a bacte ricidal antibiotic that inh ibits ce ll wall mucopeptide for mation. I t is used fo r serious
• 12 infections with gram-positive multidrug -resistant organisms including meth icillin-resistant Staphylococcus
• 13
aureus and Clostridium difficile . Vancomycin is not used to t reat Pseudomonas infection .
Vancomycin Staphylococcus aureus Antibiotics Cell wall Gram-positive bacteria Clostridium difficile colitis Clostridium difficile (bacteria) Pseudomonas
• 14
Staphylococcus Bactericide Multiple drug resistance Clostridium
• 15
• 16
• 17 Bottom Line:
• 18 Ceftazidime, a third-generation cepha lospor in, is an appropr iate t reatment for pneumonia due to
• 19 Pseudomonas .
Cephalosporin Ceftazidime Pneumonia Pseudomonas
• 20
• 21

6
lock
s
Suspend
0
End Block
Item: 2 of 3 7 ~ 1 • M k -<:J 1>- Jil ~· !:';-~
QIO: 1755 ..L ar Pre v ious Next Labfli!llues Not es Calcula t o r
& &
1
FA17 p 185.1
2
Cephalosporins (generations 1- V)
•3
MECHANISM ~-lactam drugs that inhibit cell wall S) nthesis Organisms typically not co,·ered by 1st-4th
·4 but are less su sceptible to penicillinases. generation cephalosporins are LA~t E:
•5 Bactericidal. Listeria, \t~-picals (Chlamydia, Mycoplasma),
•6 ~ l RSA, and E nterococci .
.7 CLINICALUSE 1st generation (cefazolin, cephalexin)-gram Ef> lsi generation-PEcK.
·8 cocci, Proteus mirabilis, E coli, Klebsiella
.9 pneumoniae. Cefazolin used prior to surgef) to
pre,·ent S aureus wound infections.
• 10
2nd generation (cefaclor, ccfO'I.ilin, Etke fo:\ fur .
• 11
cefuroxime)- gram Ef> cocci, II influenzae, 2nd generation- HEl'S PEcK.
• 12 Enterobacter aerogenes, '\leisserict spp., Serratia
• 13 marcescens, Proteus mirabilis, E coli, Klebsiella
• 14 pneumoniae.
3rd generation (ceftriaxone, cefotaxime, C<m cross blood-brain barrier.
• 15
• 16
cefpodoxime, ceftazidime)-serious gram e Ccftriaxone-mening itis, gonorrhea,
infections resistant to other ~-l actams. disseminated Lyme disease.
• 17 Ceftazidime- Pseudomonas.
. 18 e
4th generation (cefcpimc)-gram organisms,
• 19 with t activity against Pseudomonas and gram
• 20 Ef> organisms.
• 21
5th generation (ceftaroline)-broad gram Ef> and

a
Lock
s
Suspend
8
End Bl ock
Item: 2 of 37 ~. , . M k <:] t> al ~· ~
QIO: 1755 .l. ar Previous Next lab 'lifllues Notes Calculator

2 FA17 p 175.2
•3 Common causes of pneumonia
.4 NEONATES(< 4 WK) CHILDREN (4 WK- 18 YR) ADULTS (18- 40 YR) ADULTS (40- 6S YR) ELDERLY

•5 Group B streptococci Viruses (RSV) Mycoplasma S pneumoniae S pneumoniae


E coli Mycoplasma C pneumoniae H influenzae lnAuenza virus
•6
C trachomatis S pneumoniae Anaerobes Anaerobes
.7
(infants-3 yr) Viruses (eg, influenza) Viruses 1-1 i nfluenzae
•8 C pneumoniae Mycoplasma Gram 8 rods
•9 (school-aged
• 10 children)
S pneumoniae
· 11
Runts May C ough
• 12
Chunky Sputum
• 13 Special gro ups
• 14 Alcoholic Klebsiella, anaerobes usually due to aspiration (eg, Peptostreptococcus, Fusobacterium, Prevotella,
• 15 Bacteroides)
• 16 IV drug users S pneumoniae, S aureus
• 17 Aspiration Anaerobes
• 18 Atypical Mycoplasma, Legionella, Chlamydia
• 19
Cystic fibrosis Pseudomonas, S au reus, S pneumoniae, Burkholderia cepacia
• 20
lmmunocompromised S aureus, enteric gram 8 rods, fungi, viruses, P jirovecii (with I II V)
• 21
c .................. o......... .J ........... ..... _.,.., ...... h ..... - ..... ............ -: ......... ....... .,.....a _...., ...1,.

6
lock
s
Suspend
0
End Block
Item: 2 of 3 7 ~ 1 • Ma rk -<:J 1>- Jil ~· !:';-~
QIO: 1755 ..L Pre v ious Next Labfli!llues Not es Calcula t o r

2 FA17 p 139.3
•3 Pseudomonas Treatments include "CAl\IPFIRE" drugs:
Aerobic, motile, gram 8 rod. Non-lactose
·4 aeruginosa fermenting, oxidase@. Produces pyocyanin C arbapcnems
•5 ( blue-green pigment }; has a grape-like odor. \ m inoglycosides
•6 Produces endotoxin (b-cr, shock}, exotoxin A \lonobactams
(inactivates EF-2}, phospholipase C (degrades Polymyxins (eg, polymyxin B, colistin)
.7
cell membranes), and procyanin (generates F luoroquinolones (eg, ciprofloxacin,
·8 reacti,·e oxygen species). levoAoxaci n)
.9 PSEUD0~10X\S is associated "ith: Th IRd- and fourth-generation
• 10 P neumonia, pyocyanin cephalosporins (eg, ceftazidime, cefepime)
• 11 Sepsis Extended-spectrum penicillins (eg,
Ecthyma gangrenosum pipcracillin, ticarcillin)
• 12
• UT ls Acrugi nosa-aerobic.
• 13 Mucoid polysaccharide capsule may contribute
• D iabetes, drug use
• 14 • O steomyelitis (eg, puncture wou nds} to chronic pneumonia in cystic fibrosis patients
• 15 • M ucoid polysaccharide capsule due to biofilm formation.
• 16 • O ti tis extcrna (swimmer's car} Can cause wound infection in burn victims.
• N osocomial infections (catheters, Cornea l ulcers/keratitis in contact lens wearers/
• 17
equipment) minor eye trauma.
. 18 Frequently found in water .... hot tub folliculitis.
Exotoxin A
• 19 Skin infections (hot tub foll iculitis) Ecthyma gangrenosum - rapidly progressive,
• 20 necrotic cutaneous lesion : caused b)
• 21 Pseudomonas bacteremia. l )'pically seen in
• =· - ·~ . ...... . . . . . . . .-------=---.. 1 _. . ._; __. . .
a
Lock
s
Suspend
8
End Bl ock
Item: 3 of 37 ~ 1 • M k -<:J 1>- Jil ~· !:';-~ ~
QIO: 1731 ..L ar Pre v ious Next Labfli!llues Notes Calcula t o r

IAA]
& &
1
A 16-year-old boy has a syndrome characterized by steatorrhea, chronic persistent cough, and inferti lity. He
2
has been hospitalized on multiple occasions because of pneumonia. He is admitted to the hospital. He has a
•3 productive cough , yielding sputum with a fruity odor. Gram stain of the patient's sputum is performed , as
·4 shown in the image. Physica l examination reveals dullness to percussion over the lower left lobe.
•5
,
•6
I
"'....
•7
,, -
·8
.9
• 10
>
c.
'
,,
I

·~
,
- ·~

• 11 I
.,.,. "fl ( , ,
• 12 ' -~
"- , '\ ;
• 13
• 14
• 15
" ' "'<'"~)

- ,, ' ~,,,'-<r
., ,
....
• 16
") '• I \ > \/
• 17 - t I
.... " 'l'
• 18
\ I ... I ~
• 19
I
• 20 Ill
\ \
• 21

Itt\ I

a
Lock
s
Suspend
8
End Bl ock
Item: 3 of 37 ~ 1 • M k -<:J 1>- Jil ~· !:';-~
QIO: 1731 ..L ar Pre v ious Next Labfli!llues Notes Calcula t o r

IAA]
& &
1
A 16-year-old boy has a syndrome characterized by steatorrhea, chronic persistent cough, and inferti lity. He
2
has been hospitalized on multiple occasions because of pneumonia. He is admitted to the hospital. He has a
•3 productive cough , yielding sputum with a fruity odor. Gram stain of the patient's sputum is performed , as
·4 shown in the image. Physica l examination reveals dullness to percussion over the lower left lobe.
•5
,
•6
I
"'....
•7
,, -
·8
.9
• 10
>
c.
'
,,
I

·~
,
- ·~

• 11 I
.,.,. "fl ( , ,
• 12 ' -~
"- , '\ ;
• 13
• 14
• 15
" ' "'<'"~)

- ,, ' ~,,,'-<r
., ,
....
• 16
") '• I \ > \/
• 17 - t I
.... " 'l'
• 18
\ I ... I ~
• 19
I
• 20 Ill
\ \
• 21

Itt\ I

a
Lock
s
Suspend
8
End Bl ock
Item: 3 of 37 ~ 1 • M k -<:J 1>- Jil ~· !:';-~
QIO: 1731 ..L ar Pre v ious Next Labfli!llues Notes Calcula t o r

1
,
2
•3 -
,, '
)

,,,.,
~ r ',
i"""(
' '
·4
'- ' ....'> \/'
•5 - 't I
'I
't
•6
\ I... I ~
.7
I
·8 Ul
\
.9
• 10
Image courtesy of CDC
'\;
'"''
• 11

• 12
Which of the following would be the most appropriate fi rst-line therapy for th is patient?
• 13
:
• 14
A. Cefazolin + vancomyci n
• 15
• 16
B. Colistin

• 17 C. Piperaci llin/tazobacta m + gent am icin


. 18 D. Trim ethoprim-su lfamethoxazole + car bapenem
• 19
E. Vancomycin + gentam icin
• 20
• 21

a
Lock
s
Suspend
8
End Bl ock
Item: 3 of 37 ~ 1 • M k -<:J 1>- Jil ~· !:';-~
QIO: 1731 ..L ar Prev ious Next Labfli!ll ues Notes Calculat o r

& &
1 The correct answer is C. 70°/o chose this.
2
This patient has cystic fibrosis (CF) . Patients with CF are often colonized with
3
Pseudomonas aeruginosa. It is an aerobic gram-negative rod that produces
·4 pyocya nin, a blue-g reen pigment, as shown in the image here, and causes
•5 many different types of infection, including pneumonia in patients with CF,
•6 swimmer's ear, urinary tract infections, and hot-tub folliculitis. Treatment of
Pseudomonas species infection can be achieved by many agents, including
•7
antipseudomonal penicillins, ciprofloxacin, and antipseudomonal
·8 cephalosporins. However, initial treatment for Pseudomonas bacteremia,
•9 wound infection, pneumonia, or endocarditis is antipseudomonal penicillin
plus an aminoglycoside such as gentamicin. Possible antibiotic combinations
• 10
include:
• 11
1) An antipseudoma l ~-lactam PLUS an antipseudomonal quinolone Image courtesy of Wikimedia
• 12
Commons
2) An antipseudoma l ~-lactam PLUS an aminog lycoside
• 13
• 14 3) An antipseudomonal quinolone PLUS an aminoglycoside
• 15 Antipseudomonal ~- l actams include pipercil lin-tazobactam, ceftazid ime, cefipi m e, imipenem, meropenem, and
• 16
doripenem .
• 17 Antipseudomonal f lou roqu inolones include ciprofloxacin and levofloxacin .
• 18 Aztreonam may be substitut ed for one of these if al lergy to penicil lin .
• 19 Aminoglycoside Aztreonam Meropenem Imipenem Gentamicin Ceftazidime Pseudomonas aeruginosa Cystic fibrosis Levofloxacin Pyocyanin Quinolone Bacteremia

• 20 Dorlpenem Penicillin Extended-spectrum penicillin Gram-negative bacteria Ciprofloxacin Endocarditis Antibiotics Cephalosporin Pseudomonas

• 21 Urinary tract infection Otitis externa Urinary system Aerobic organism Pneumonia Fibrosis Pigment Infection Folliculitis Allergy

a
Lock
s
Suspend
8
End Bl ock
Item:3of37 ~. , . M k <:] t> al ~· ~
QIO: 1731 .l. ar Previous Next Lab 'lifllues Notes Calculator

1 p • q • p •
2 Aztreonam may be substituted for one of these if al lergy to penicillin.
3 Aminoglycoside Aztreonam Meropenem Imipenem Gentamicin Ceftazidime Pseudomonas aeruginosa Cystic fibrosis levofloxacin Pyocyanin Quinolone Bacteremia

•4 Doripenem Penicillin Extended-spectrum penicillin Gram-negative bacteria Ciprofloxacin Endocarditis Antibiotics Cephalosporin Pseudomonas

•5 Urinary tract infection Otitis externa Urinary system Aerobic organism Pneumonia Fibrosis Pigment Infection Folliculitis Allergy
•6
A is not correct. 9°/o chose this.
•7
Cefazolin is a fi rst-gene ration cephalosporin. It is often given perioperatively as a prophylactic treatment against
•8 development of bacterial infection, and its administration is often intravenous or injected. It is not effective
•9 against Pseudomonas. Vancomycin is a drug that inhibits cell wa ll synthesis in bacteria by inhibiting
• 10 peptidoglycan synthesis . One of its key uses is in the t reatment of pseudomembranous colitis caused by
Clostridium difficile infections .
· 11 Clostridium difficile colitis Cephalosporin Peptidoglycan Vancomycin Cefazolin Cell wall Clostridium difficile (bacteria) Pathogenic bacteria Intravenous therapy
• 12
Pseudomonas Colitis Clostridium Bacteria Preventive healthcare Infection
• 13
• 14
B is not correct. 3°/o chose this .
Colistin has antipseudomonal activity but, due to its associated toxicity, it is general ly reserved for the setting of
• 15
multidrug resistance .
• 16 Colistin Multiple drug resistance

• 17
Dis not correct. 11% chose this .
• 18
Trimethoprim-sulfam ethoxazole (TMP-SMX) is a bacter icidal antibiotic interfer ing w ith folate metabol ism . It has
• 19 good gram-negative cover and is frequently used to treat UTi s. It is generally not effective aga inst
• 20 Pseudomonas .
Trimethoprim/sulfamethoxazole Gram-negative bacteria Antibiotics Folic acid Pseudomonas Bactericide Metabolism
• 21

6
lock
s
Suspend
0
End Block
Item:3of37 ~. , . M k <:] t> al ~· ~
QIO: 1731 .l. ar Previous Next Lab 'lifllues Notes Calculator

1 •
Pseudomonas Colitis Clostridium Bacteria Preventive healthcare Infection
2
3 B is not correct. 3°/o chose this.
•4 Col istin has antipseudomona l activity but, due to its associated toxicity, it is gene ral ly reserved for the setting of
multidrug resistance.
•5
Colistin Multiple drug resistance
•6
Dis not correct. 11% chose this.
•7
Trimethoprim-sulfamethoxazole (TMP-SMX) is a bactericidal antibiotic interfering with folate metabol ism . It has
•8
good gram-negative cover and is frequently used to t reat UTi s. It is generally not effective aga inst
•9 Pseudomonas .
• 10 Trimethoprim/sulfamethoxazole Gram-negative bacteria Antibiotics Folic acid Pseudomonas Bactericide Metabolism

· 11 E is not correct. 7°/o chose this.


• 12 Vancomycin inh ibits bacteria l cel l wal l synthesis by binding to the D-ala-D-ala site on a cell wal l precurso r. It is
• 13 used for broad cove rage of gram-positive infections, but is not usefu l against a gram -negative bacterium such
as Pseudomonas .
• 14
Vancomycin Cell wall Gram-negative bacteria Gram-positive bacteria Pseudomonas Bacteria
• 15
• 16
• 17 Bottom Line:
• 18 I nitial treatment for Pseudomonas bacte remia, wound infection, pneumon ia, or endocarditis is with an
• 19
antipseudomonal penici llin plus an aminog lycoside .
Aminoglycoside Bacteremia Penicillin Endocarditis Pneumonia Pseudomonas Infection
• 20
• 21

6
lock
s
Suspend
0
End Block
Item: 3 of 37 ~ 1 • M k -<:J 1>- Jil ~· !:';-~
QIO: 1731 ..L ar Pre v ious Next Labfli!llues Notes Calcula t o r
& &
1

2 FA17 p 184.3

3 Antipseudomonal Piperacillin, ticarcillin.


·4 penicillins
•5 MECHANISM Same as penicillin. Extended spectrum.
•6 CLINICAL USE Pseudomonas spp. and gram 8 rods; susceptible to penicillinase; usc with ~-lactam ase inhibitors.
.7 ADVERSE EFFECTS Hypersensiti,ity reactions.
·8
.9 FA17 p 187.2
• 10
Aminoglycosides G entamicin, :\'eomycin, \ mikaci n, "\lean" (aminoglycoside) C:\ \ TS ca "~OT
• 11 Tobramycin, Streptomycin. kill an aerobes .
• 12 MECHANISM Bactericidal; irreversible inhibition of initiation
• 13 complex through binding of the 30S subunit.
• 14 C an cause misreading of mR1 A. lso block
• 15 translocation. Require 0 2 for uptake; therefore
ineffective agai nst anaerobes .
• 16
CLINICAL USE Severe gram 8 rod infections. S>nergistic with
• 17
~-lactam antibiotics.
. 18
Neomycin for bo"'el surgerr .
• 19
AOVERSEEFFECTS r\ephrotoxicity, :\euromuscular blockade,
• 20
O totoxicity (especially when used with loop
• 21 diuretics). Teratogen .

a
Lock
s
Suspend
8
End Bl ock
Item: 3 of 37 ~ 1 • Ma rk -<:J 1>- Jil ~· !:';-~
QIO: 1731 ..L Pre v ious Next Labfli!llues Notes Calcula t o r

1
FA17 p 139.3
2
Pseudomonas Aerobic, motile, gram 8 rod. 1 on-lactose Treatments include "CAl\IPFIRE " drugs:
3 aeruginosa fermenting, oxidase <:B. Produces pyocyanin C arbapcnems
·4 ( blue-green pigment ); has a grape-like odor. \ minoglycosides
•5 Produces endotoxin (fe\'er, shock), exotoxin A \ lonobactams
•6 (inacti,·ates EF-2}, phospholipase C (degrades Polymyxins (eg, polymyxin B, colistin)
cell membranes), and pyocyanin (generates F luoroquinolones (eg, ciproAoxaci n ,
.7
reacti,·e oxygen species). le\'oAoxaci n)
·8
PSEUD0~10X\S is associated '' ith: Th IRd- and fourth-generation
.9 P neumonia, pyocyanin cephalosporins (eg, ceftazidime, cefepime)
• 10 Sepsis Extended-spectrum penicillins (eg,
• 11 Ecthyma gangrenosum pipcracillin, ticarcillin)
• UTls Aeruginosa-aerobic.
• 12
• D iabetes, drug use Mucoid polysaccharide capsule may contribute
• 13
• O steomyelitis (eg, puncture wounds) to chronic pneumonia in cystic ~bros is patients
• 14 • M ucoid polysaccharide capsule due to biofilm formation.
• 15 • O titis extcrna (swimmer's car} Can cause wound in fection in burn victims.
• 16 • N osocomial infections (catheters, Cornea J ,dcers/keratitis in contact lens wearers/
equipment) minor eye trauma.
• 17
Exotoxin A Frequently found in water .... hot tub folliculitis.
. 18
Skin infections (hot tub foll iculitis) Ecthyma gangrenosum - rapidly progressive,
• 19 necrotic cutaneous lesion : caused b)
• 20 Pseudomonas bacteremia. Typically seen in
• 21 immunocompromised patients.

a
Lock
s
Suspend
8
End Bl ock
Item:4of37 ~. , . M k <:] t> al ~· ~
QIO: 3369 .l. ar Previous Next lab 'lifllues Notes Calculator

1 •
A 37-year-old man presents to the emergency department because of severe shortness of breath that
2 started while he was cross-country skiing. He is active and otherwise healthy, with no significant medica l
3 history. The patient has never had an attack this severe before but says he often has similar symptoms wh ile
•4 running or when he has a co ld . Physica l examination shows labored breathing with polyphonic wheezes in the lung
f ields bi lateral ly and a nasa l polyp in the right nares. X-ray of the chest shows hyperinflation without any evidence
•5
of consolidation or infi ltrate .
•6
•7
Wh ich phar macologic stimulus has the greatest chance of precipitating a similar attack in this patient'
•8
:
•9
A. Aspirin
• 10
B. Captopril
· 11
• 12 C. Corticosteroids
• 13 D. Penicil lin
• 14
E. Theophylline
• 15
• 16
• 17

• 18
• 19
• 20
• 21

6
lock
s
Suspend
0
End Block
Item:4of37 ~. , . M k <:] t> al ~· ~
QIO: 3369 .l. ar Previous Next lab 'lifllues Notes Calculator

1 •
The correct answer is A. 60°/o chose this.
2 The patient's history is suggestive of asthma . Asthma is commonly t r iggered by exercise (especially in the cold),
3 viral upper respiratory tract infections, allergens, and stress. Drugs most common ly associated with acute
4 asthma attacks include aspirin, food-coloring agents like tartrazine (a yellow food dye), ~-adrenerg i c
antagon ists, and sulfating agents. Aspirin -sensitive asthma is also known as the Samter triad : a combination of
•5
nasal polyps, asthma, and aspi rin sensitivity .
•6 Asthma Aspirin Nasal polyp Respiratory tract Food coloring Allergen Virus Tartrazine Upper respiratory tract infection Dye

•7
B is not correct. 14% chose this .
•8
Captopril is an angiotensin-converting enzyme (ACE) inhibitor used to treat hypertension . Although cough is an
•9 adverse effect of ACE inhibitors attributed to impaired degradation of bradykinin, captopril would not be
• 10 expected to precipitate an asthma attack. Other possible side effects of captopr il include dizziness, nausea,
diarr hea, itch ing, and insomnia.
· 11
Bradykinin Captopril Angiotensin-converting enzyme ACE inhibitor Enzyme Asthma Insomnia Hypertension Nausea Diarrhea Itch Cough Adverse effect
• 12
Adverse drug reaction Enzyme inhibitor Side effect
• 13
• 14 C is not correct. 7°/o chose this .
• 15
Corticosteroids are powerful anti -inflammatory agents that are used as first-line therapy for ch ron ic asthma .
They wou ld not be expected to precipitate an asthma attack. Long-term use of corticosteroids can lead to
• 16 osteoporosis, high blood suga r, increased risk of infection, and weight gain.
• 17 Osteoporosis Asthma Anti-inflammatory Blood sugar Corticosteroid Hyperglycemia Sugar Weight gain Precipitation (chemistry) Infection

• 18
D is not correct. 8°/o chose this .
• 19 Although penicil lins can cause hypersensitivity reactions that trigger life-threatening anaphylaxis, there is no
• 20 evidence to suggest that this patient is allergic to penicillin. Common side effects of pen icill in include nausea,
• 21 vomiting, diarrhea, abdominal pa in, and itch ing .
• A-.-.-.k •• t-.. :,.. n ...... : ...:u: ... '"':-...."- ... - T• ...&.. • .................... ,..:.:,.:.,, ,.,_,,,.. ... _ A&-...1 .......... : ... -1 ,...._: ... ,,.,.......,:.: ...... A.-I ............. .-1 ....... ..... - ......: ...... r:.-1 ... ... u ......... All ......... .

6
lock
s
Suspend
0
End Block
Item:4of37 ~. , . M k <:] t> al ~· ~
QIO: 3369 .l. ar Previous Next lab 'lifllues Notes Calculator

1 • g, •
Bradykinin Captopril Angiotensin-converting enzyme ACE inhibitor Enzyme Asthma Insomnia Hypertension Nausea Diarrhea Itch Cough Adverse effect
2
Adverse drug reaction Enzyme inhibitor Side effect
3
4 C is not correct . 7°/o chose this.
•5 Corticosteroids are powerfu l anti-infl ammatory agents that are used as first-line therapy for chron ic asthma.
They wou ld not be expected to precipit ate an ast hma att ack. Long -term use of corticosteroids can lead to
•6
osteoporosis, high blood sugar, increased risk of infection, and weight gain.
.7 Osteoporosis Asthma Anti-inflammatory Blood sugar Corticosteroid Hyperglycemia Sugar Weight gain Precipitation (chemistry) Infection

•8
D is not correct. 8°/o chose this .
•9
Although penicil lins can cause hypersensit ivit y react ions t hat t rigger life-threatening anaphylaxis, there is no
• 10 evidence t o suggest t hat t his patient is allergic to pen ici llin . Common side effects of pen ici llin include nausea,
· 11 vomit ing, diarr hea, abdomina l pa in, and itch ing.
Anaphylaxis Penicillin Diarrhea Itch Hypersensitivity Nausea Abdominal pain Vomiting Adverse drug reaction Side effect Allergy
• 12
• 13 E is not correct. 11% chose this .
• 14 Theophylline is a bronchod ilator used for maintenance t herapy in asthma and would not be expected to
• 15
precipitat e an asthma attack . Common side effects of theophylline include stomach cramping, nausea, vomiting,
diarrhea, restlessness, f lush ing, and irritability .
• 16 Bronchodilator Theophylline Asthma Diarrhea Nausea Vomiting Irritability Adverse drug reaction Stomach Side effect Flushing (physiology)
• 17

• 18
• 19
Bottom Line:
• 20 Aspirin-sensit ive asthma involves a combination of nasa l polyps, asthma, and aspirin sensit ivity (Samter triad) .
Aspirin Asthma Nasal polyp Polyp (medicine)
• 21

6
lock
s
Suspend
0
End Block
Item:4of37 ~. , . M k <:] t> al ~· ~
QIO: 3369 .l. ar Previous Next lab 'lifllues Notes Calculator

1 • •
FA17 p638.1
2
Obstructive lung Obstruction of air flow- air trapping in lungs. Airways close prematurely at high lung volumes
3
diseases - t RV and t FRC, t TLC. PFTs: U FEV 1, l FVC - l FEV 1/FVC ratio (hallmark),
4
V/Q mismatch. Chronic, hypoxic pulmonary vasoconstriction can lead to cor pulmonale. Chronic
•5 obstructive pulmonary disease (COPD) includes chronic bronchitis and emphysema .
•6 TYPE PRESENTATION PATHOLOGY OTHER
•7 Chronic bronchitis Findings: wheezing, crackles, Hypertrophy and hyperplasia Diagnostic criteria: productive
•8 ("blue bloater") cyanosis (hypoxemia due of mucus-secreting glands cough for> 3 months in a
•9 to shunting), dyspnea, C02 in bronchi - Reid index year for> 2 consecutive years.
retention, zopolyc)themia. (thickness of mucosal gland
• 10
layer to thickness of wall
· 11 between epithelium and
• 12 cartilage) > 50%.
• 13 Emphysema ("pink Centriacinar-associated with Enlargement of air spaces CXR: t AP diameter, flattened
• 14 puffer") smoking rJ IE). Frequently in l recoil, t compliance, diaphragm, t lung field
• 15 upper lobes (smoke rises up). l DLCO from destruction of lucency.
• 16
Panacinar-associated with alveolar walls (arrow in ~). Barrel-shaped chest [!].
a 1-antitrypsin deficiency. t elastase activity - t loss Exhalation through pursed lips
• 17
Frequently in lower lobes. of elastic fibers - t lung to increase airway pressure
• 18 compliance. and prevent airway collapse.
• 19
Asthma Findings: cough, wheezing, Branchial hypcrrcsponsi,·cncss Aspirin-i nduced asthma: COX
• 20 tachypnea, dyspnea, - reversible inhibition - leukotriene
• 21 hypoxemia, l inspiratory/ bronchoconstriction. Smooth overproduction - airway •

6
lock
s
Suspend
0
End Block
Item: 4 of 3 7 ~ 1 • M k -<:J 1>- Jil ~· !:';-~
QIO: 336 9 ..L ar Pre v ious Next Lab fli!ltues Not es Calcula t o r
& &
1
FA17 p649.1
2
Asthma drugs Bronchoconstriclion is mediated b~ (I) inAammatory processes and (2) parasympathetic tone;
3
therapy is directed at these 2 pathways.
4
~-agonists Albuterol- rclaxes bronchial smoot h muscle (short acting 13z-agonist). Used during acute
•5
exacerbation .
•6
Salmeterol, formoterol - long-acting agents for prophylaxis. Ad,·erse effects are tremor and
.7 arrlwthmia.
·8 Inhaled Fluticasone, budesonide -inhibit the synthesis of ,·irtuallr all cytokines. lnacli\Cite NF-KB, the
.9 corticosteroids transcription factor that induces production ofT W-a. and other inAammatory agents. 1st-line
• 10 therapy for chronic asthma . .Vlay cause oral thrush .
• 11 Muscarinic Tiotropium, ipratropium-compet itively block muscarinic receptors, preventing
• 12 antagonists bronchoconstriction. Also used for COPD. Tiotropium is long acting.
• 13 Antileukotrienes Montelukast, zafirlukast- block lcukotriene Exposure to an~gen
• 14 receptors (CysLTl). Especially good for (dust pollen, etc}
aspirin-induced and exercise-induced asthma .
• 15
Zileuton -5-lipoxygenase pathway inh ibitor. ~ Avoidance
• 16
Blocks conversion of arachidonic acid to
• 17 leukotrienes. Hepatotoxic.
. 18 Antigen and lgE ~ Omahzumab
Anti-lgE monoclonal Omalizumab- binds most!)' unbound serum on mast cells
• 19 therapy lgE and blocks binding to FceRI. Used in
• 20 allergic asthma with t lgE levels resistant to ~ Steroids
• 21
inhaled steroids and long-acting 13z-agonists.

a
Lock
s
Suspend
8
End Bl ock
Item: 5 of 3 7 ~ 1 • M k -<:J 1>- Jil ~· !:';-~
QIO: 1730 ..L ar Pre v ious Next Labfli!llues Not es Calcula t o r
& &
1
A 35-year-old HIV-positive woman with a CD4+ count of 175/1-JL (normal = 500-1500/1-J L) presents to the
2
cl inic with a 2-week history of fever, nonproductive cough, and progressive dyspnea . She has a history of
3 sulfa allergy. Physical examination reveals diffuse crackles and rhonchi. X-ray of the chest shows diffuse,
4 bilateral interstitial infiltrates. Laboratory studies are remarkable only for an elevated lactate dehydrogenase level.
•5
•6 Which of the following is the best choice for prophylaxis against this infection in a patient with a sulfa allergy?
•7 :

·8 A . Aerosolized pentamidine
.9 B. Ciprofloxacin
• 10
C. Fluconazole
• 11
D. Terbinafine
• 12
• 13 E. Trimethoprim-su lfamethoxazole
• 14
• 15
• 16
• 17
• 18
• 19
• 20
• 21

a
Lock
s
Suspend
8
End Bl ock
Item: 5 of 3 7 ~ 1 • M k -<:J 1>- Jil ~· !:';-~
QIO: 1730 ..L ar Prev ious Next Labfli!ll ues Not es Calculat o r

& &
1
2 The correct answer is A. 47°/o chose this.
3 This vignette suggests the pat ient has Pneumocystis jirovecii
4 pneumon ia based on the HIV status, physical examination, and X-
5
ray findings (shown in the image), as well as the elevated lactate
dehydrogenase level. Prophylactic therapy for P. jirovecii
•6
pneumonia is indicated for an HIV- positive patient with a CD4+ T-
•7 lymphocyte count <200/1-JL. The standard prophylactic therapy for
·8 P. jirovecii pneumonia is trimethoprim- sulfamethoxazole. This
combination, however, is contraindicated for patients with a sulfa
.9
allergy, because sulfamet hoxazole is a sulfa drug. In these cases,
• 10 the best alternative treatment is aerosolized pentamidine.
• 11 Pentamidine works by inhibiting DNA, RNA, phospholipid, and
• 12
protein synthesis .
Pentamidine Sulfonamide (medicine) Trimethoprim/sulfamethoxazole Pneumocystis jirovecii
• 13
Phospholipid Lactate dehydrogenase HIV Pneumonia Protein T cell Sulfamethoxazole Lactic acid
• 14
Allergy DNA RNA X·ray Physical examination Preventive healthcare Protein synthesis
• 15
Pneumocystis pneumonia
• 16
• 17
B is not correct. 21 °/o chose this .
• 18
Ciprofloxacin is a f luoroquinolone agent used to treat urinary tract or gast rointestinal infections caused by gram-
• 19 negative organisms. Adverse effect s include gast rointestinal upset and tendinitis.
• 20 Quinolone Ciprofloxacin Gram-negative bacteria Tendinitis Urinary system Gastrointestinal tract Human gastrointestinal tract

• 21 C is not correct. 15°/o chose this.



a
Lock
s
Suspend
8
End Block
Item:5of37 ~. , . M k <:] t> al ~· ~
QIO: 1730 .l. ar Previous Next Lab 'lifllues Notes Calculator

1 g g • I g p I •

Quinolone Ciprofloxacin Gram-negative bacteria Tendinitis Urinary system Gastrointestinal tract Human gastrointestinal tract
2

3 C is not correct. 15% chose this.


4 Fluconazole is an antifungal agent used to t reat systemic infections. It is particula r ly effective against candida !
infections and can be used as funga l prophylaxis in severely immunocompromised individuals. It is also used in
5
the t reatmentof cryptococcal mening it is .
•6 Fluconazole Cryptococcosis Meningitis Immunodeficiency Antifungal Fungicide Cryptococcus Fungus Cryptococcus neoformans Preventive healthcare
.7
D is not correct. 10% chose this .
•8
Ter binafine is an antifunga l agent that blocks ergosterol synthesis by inhibiting squalene epoxidase. It is used to
•9 treat dermatophytoses.
• 10 Dermatophytosis Terbinafine Ergosterol Squalene monooxygenase Squalene Fungicide Antifungal

· 11
E is not correct. 7°/o chose this .
• 12 Trimethoprim-sulfamethoxazole is the initial choice for prophylactic treatment of Pneumocystis jirovecii
• 13 pneumon ia un less patients are unable to to lerate its harsh adverse effects, or if they have a su lfa allergy. The
• 14 most common adverse effects are fever, rash, and bone marrow suppression .
Trimethoprim/sulfamethoxazole Pneumocystis jirovecii Bone marrow suppression Pneumonia Bone marrow Rash Sulfonamide (medicine) Allergy
• 15
Preventive healthcare Pneumocystis pneumonia Bone Fever
• 16
• 17

• 18 Bottom Line:
• 19 Aeroso lized pentamidine is the treatment of choice for P. jirovecii infection in patients with su lfa allergies .
• 20 Pentamidine Sulfonamide (medicine) Allergy

• 21

6
lock
s
Suspend
0
End Block
Item:5of37 ~. , . M k <:] t> al ~· ~
QIO: 1730 .l. ar Previous Next Lab 'lifllues Notes Calculator

1
FA17 p 150.1
2
Pneumocystis jirovecii Causes Pneumocystis pneumonia (PCP), a diffuse interstitial pneumonia fJ. Yeast-like
3
fungus (originally classified as protozoan). Inhaled. Most infections are asymptomatic.
4
Immunosuppression (eg, AIDS) predisposes to disease. Diffuse, bilatera l ground-glass opacities on
5 CXR/CT [iJ. D iagnosed by lung biopsy or lavage. Disc-shaped yeast seen on methenamine silver
•6 stain of lung tiss ue ~.
.7 Treatment/prophylaxis: TMP-SMX, pentamidine, dapsone (prophylaxis only), atovaquone. Start
•8
prophylaxis when C04+ count drops to< 200 cells/mm 3 in l-IlY patients.

•9
• 10

· 11
• 12
• 13
• 14
• 15
• 16
• 17

• 18
• 19 FA11 p 194.2
Prophylaxis in HIV patients
• 20
CELL COUNT PROPHYLAXIS INFECTION
• 21

6
lock
s
Suspend
0
End Block
Item: 5 of 3 7 ~ 1 • M k -<:J 1>- Jil ~· !:';-~
QIO: 1730 ..L ar Pre v ious Next Labfli!llues Not es Calcula t o r

1
&
-· ~~ .. - ·.. ... . ... . .. . . -- - . ... . - &

CXR/CT [lJ. Diagnosed by lung biopsy or lavage. Disc-shaped yeast seen on methenamine sih er
2 stain of lung t issue ~-
3 Treatment/prophylaxis: TYIP-S.tviX, pentamidine, dapsone (prophylaxis only), atovaquone. Start
4 prophylaxis when CD4+ count drops to < 200 cells/mm3 in HIV patients.
5
•6
.7
·8
.9
• 10
• 11

• 12
• 13
• 14
• 15 FA17 p 194.2
• 16 Prophylaxis in HIV patients
CELL COUNT PROPHYLAXIS INFECTION
• 17
C04 < 200 cells/mm3 TMP-SMX Pneumocystis pneumonia
. 18
• 19
C04 < 100 cells/mm3 TYJP-SMX Pneumocystis pneumonia and toxoplasmosis
• 20 C04 < 50 cells/mm3 Azithromycin or clarithromycin J\lrcob(ICierium avium complex
• 21

a
Lock
s
Suspend
8
End Bl ock
Item: 6 of 3 7 ~ 1 • M k -<:J 1>- Jil ~· !:';-~
QIO: 3356 ..L ar Pre v ious Next Lab fli!ltues Notes Calcula t o r

IAA]
& &
1
A 4-year-old girl is brought to the emergency department with shortness of breath . Her parents tell the
2
physician that their daughter has had difficu lty breathing on many other occasions, especial ly when they take
3 her outside in the cold. On physical examination, the sound in the audio clip is heard . Her oxygen saturation
4 is 93% and dropping.
5 OPE N MEDIA

•6
•7
Adm inistering which of the following types of drugs would make this patient's current condition worse?
·8
.9 :
A. A drug that decreases production of hydroperoxides
• 10
• 11 B. A drug that decreases production of tumor necrosis factor-a
• 12 C. A drug that increases production of arachidonic acid
• 13
D. A drug that increases production of epinephrine
• 14
• 15
• 16
• 17
• 18
• 19
• 20
• 21

a
Lock
s
Suspend
8
End Bl ock
Item: 6 of 37 ~. , . M k <:] t> al ~· ~
QIO: 3356 .l. ar Previous Next lab 'lifllues Notes Calculator

1 • •
The correct answer is C. 74°/o chose this.
2
This child is suffering from an acute asthma attack and has difficu lty breath ing along with wheezing due to the
3 intense bronchoconstriction (as heard in the aud io clip) . I n her case, the symptoms seem to be triggered by
4 co ld weather. Treatment for an acute asthma attack invariably includes administration of a ~-agonist, such as
albuterol. This increases cAMP production in the bronchial smooth muscle cel ls and promotes dilation of the
5
airway. In the short term, epineph r ine can be used to achieve a strong bronchodi latory effect. Various other
6 drugs mar keted for long-term asthma control cause bronchodilation by changing the relative amounts of
•7 arach idonic acid derivatives. Corticosteroids, often used to t reat asthma exacer bations, block the enzyme
•8 phospholipase A 2 and therefore decrease the production of arachidonic acid and its downstream products .
Epinephrine Arachidonic acid Bronchoconstriction Asthma Enzyme Bronchodilator Corticosteroid Smooth muscle tissue Cyclic adenosine monophosphate
•9
Respiratory tract Phospholipase Salbutamol Wheeze Phospholipase A2 Dyspnea Vasodilation Bronchus Bronchiole
• 10

· 11 A is not correct. 6°/o chose this.


• 12 Hyd roperoxides (HPETEs) are the precursors to leukotr ienes, which are potent bronchoconstr ictors. If HPETE
production were decreased by a med ication, a drop in leukotr iene production would occur, wh ich wou ld then
• 13
resu lt in bronchodi lation. As this gir l is having an asthma exacerbation, bronchodi lation is desirable and this
• 14 drug's mechanism of action wou ld therefore be beneficial to the patient . The antiasthmatic agent, zileuton, acts
• 15 through this mechanism by blocking the lipoxygenase enzyme that nor mal ly converts arach idonic acid to
• 16
HPETEs .
Zileuton Arachidonic acid leukotriene Enzyme Asthma Bronchodilator Mechanism of action lipoxygenase Organic peroxide Pharmaceutical drug
• 17

• 18 B is not correct. 7°/o chose this .


• 19
New research show ta rgeting tumor necrosis factor-a can help reduce symptoms in patients with asthma who
are receiving chronic steroid treatment. Dr ugs like etanercept work by blocking the production of tumor necrosis
• 20 factor -a.
• 21 Etanercept Asthma Necrosis Steroid Neoplasm •

6
lock
s
Suspend
0
End Block
Item: 6 of 37 ~. , . M k <:] t> al ~· ~
QIO: 3356 .l. ar Previous Next lab 'lifllues Notes Calculator

1
• I I I I I I I I y
phospholipase A 2 and therefore decrease the production of arachidonic acid and its downstream products.
2 Epinephrine Arachidonic acid Bronchoconstriction Asthma Enzyme Bronchodilator Corticosteroid Smooth muscle tissue Cyclic adenosine monophosphate
3
Respiratory tract Phospholipase Salbutamol Wheeze Phospholipase A2 Dyspnea Vasodilation Bronchus Bronchiole
4
5
A is not correct. 6°/o chose this.
Hydroperoxides (HPETEs) are the precursors to leukotrienes, which are potent bronchoconstrictors. I f HPETE
6
production were decreased by a medication, a drop in leukotriene production would occur, which would then
•7 resu lt in bronchodilation. As this girl is having an asthma exacerbation, bronchodilation is desirable and this
•8 drug's mechanism of action would therefore be beneficial to the patient . The antiasthmatic agent, zileuton, acts
•9 through this mechanism by blocking the lipoxygenase enzyme that normal ly converts arach idonic acid to
HPETEs .
• 10 Zileuton Arachidonic acid leukotriene Enzyme Asthma Bronchodilator Mechanism of action lipoxygenase Organic peroxide Pharmaceutical drug
· 11
B is not correct. 7°/o chose this .
• 12
New research show targeting tumor necrosis factor-a can help reduce symptoms in patients with asthma who
• 13
are receiving chronic steroid treatment. Drugs like etanercept work by blocking the production of tumor necrosis
• 14 factor-a .
• 15 Etanercept Asthma Necrosis Steroid Neoplasm

• 16 D is not correct. 13% chose this .


• 17 Epinephrine is an adrenergic agonist capable of activating the ~ 2 - receptors on bronchia l smooth muscle cells .
• 18 The effect of this stimulation is airway dilation and therefore a relief of asthmatic symptoms. Although effective,
epinephrine is not the first-line agent for patients with acute asthma exacerbation . Short-acting ~-agnoists, like
• 19
albuterol, are considered the first-line treatment for patients presenting with asthma exacerbation .
• 20 Epinephrine Asthma Smooth muscle tissue Agonist Adrenergic agonist Salbutamol Respiratory tract Bronchiole Adrenergic Muscle Bronchus Vasodilation Therapy
• 21

lock
6 s
Suspend
0
End Block
Item: 6 of 37 ~. , . M k <:] t> al ~· ~
QIO: 3356 .l. ar Previous Next lab 'lifllues Notes Calculator

1 • Zileuton Arachidonic acid leukotriene Enzyme Asthma Bronchodilator Mechanism of action lipoxygenase Organic peroxide Pharmaceutical drug

2 B is not correct. 7°/o chose this.


3 New research show targeting tumor necrosis factor-a can help reduce symptoms in patients with asthma who
4 are receiving chronic steroid treatment. Drugs like etanercept work by blocking the production of tumor necrosis
5
factor-a.

I
Etanercept Asthma Necrosis Steroid Neoplasm
6
•7 D is not correct. 13% chose this.
•8 Epinephrine is an adrenergic agonist capable of activating the ~ 2 -receptors on bronchia l smooth muscle cells .
The effect of this stimulation is airway dilation and therefore a relief of asthmatic symptoms. Although effective,
•9
epinephrine is not the first-line agent for patients with acute asthma exacerbation . Short-acting ~-agnoists, like
• 10 albuterol, are considered the first-line treatment for patients presenting with asthma exacerbation .
Epinephrine Asthma Smooth muscle tissue Agonist Adrenergic agonist Salbutamol Respiratory tract Bronchiole Adrenergic Muscle Bronchus Vasodilation Therapy
· 11
• 12
• 13 Bottom Line:
• 14
The ~-receptor is an adrenergic receptor that acts through a G-protein-coupled mechan ism to activate
• 15 adenylate cyclase to produce cyclic adenosine monophosphate, which promotes dilation of airways by the
• 16 relaxation of smooth muscle cel ls. Drugs, such as albuterol and epinephrine, wh ich act as agon ists to the ~­
• 17
receptor can therefore provide immediate relief of asthmatic symptoms. Long-term therapy for asthma often
includes an inhaled steroid, wh ich acts to decrease arach idonic acid derivatives that lead to airway constriction
• 18 and inflammation.
• 19 Epinephrine Arachidonic acid Cyclic adenosine monophosphate Adenylyl cyclase Asthma Corticosteroid Smooth muscle tissue Adrenergic receptor Steroid Adenosine
Salbutamol Adenosine monophosphate Adrenergic Inflammation Bronchoconstriction Respiratory tract Receptor (biochemistry) Vasodilation Muscle
• 20
• 21

6
lock
s
Suspend
0
End Block
Item: 6 of 3 7 ~ 1 • M k -<:J 1>- Jil ~· !:';-~
QIO: 3356 ..L ar Pre v ious Next Lab fli!ltues Notes Calcula t o r
& &
1
FA17 p 455.1
2
3 Arachidonic acid pathway
4
MEMBRANE
5 PHOSPHOLIPIDS GlucocortKoids
6 (Corticosteroids)
0 7 Betamethasone Methy\pfedrvsolone
Phospholipase A 2 Cortisone Predr~solone
o8 Dexamethasone Predr~sone
HydrOCortisone Tnamcinolone
.9
• 10 ENDOPEROXIDE SYNntESIS
I~ (cyclooxygenase)
• 11 Arachidonic acid
• 12 ( Z1leuton J COX-1, COX-2

• 13 Celecoxib J Aspirin (irreversible)


'-- Other NSAIDs(reversible)
0 14
Diclofenac Ketorolac
• 15 Ibuprofen Naproxen
Indomethacin
0
16
Montelukast
0
17 5-HPETE Cyclic endoperoxides

\
Zafirlukast
• 18
• 19
I
Leukotrienes
\
Prostacyclin Prostaglandins
~
Thromboxane

r
~

• 20 LTC 4 LTD4 LTE4 LTB4 PGI2 PGE, PGE 2 PGFza TXAz


• 21 I bronchial tone I neutrophil I platelet I vascular I utenne I ut enne I platelet
• chemotaxis aoarfM1ataon tone tone tone aooreoat10n

a
Lock
s
Suspend
8
End Bl ock
Item: 6 of 3 7 ~ 1 • M k -<:J 1>- Jil ~· !:';-~
QIO: 3356 ..L ar Pre v ious Next Lab fli!ltues Notes Calcula t o r
& &
1
FA17 p649.1
2
Asthma drugs Bronchoconstriclion is mediated b~ (I) inOammalory processes and (2) parasympathetic lone;
3
therapy is directed at these 2 pathways.
4
~-agonists Albuterol-rclaxes bronchial smoot h muscle (short acting 13z-agonist). Used during acute
5
exacerbation.
6
Salmeterol, formoterol - long-acting agents for prophylaxis. Ad,·erse effects are tremor and
7
0
arrln thmia.
o8
Inhaled Fluticasone, budesonide -inhibit the synthesis of ,·irtually all cytokines. lnacti,·ate NF-KB, the
.9 corticosteroids transcription factor that induces production ofT lfi'-a. and other inflammatory agents. 1st-line
• 10 therapy for chronic asthma . .\ lay cause oral thrush .
• 11 Muscarinic Tiotropium, ipratropium-compel itively block muscarinic receptors, preventing
• 12 antagonists bronchoconstriction. Also used for COPD. Tiotropium is long acting.
• 13 Antileukotrienes Montelukast, zafirlukast- block lcukotriene Exposure to anttgen
0 14 receptors (CysLTl). Especially good for (dust. pollen, etc)

• 15
aspirin-induced and exercise-induced asthma .
Zileuton -5-lipoxygenase pathway inh ibitor. ~ Avoidance
0
16
Blocks conversion of arachidonic acid to
0
17 leukotrienes. Hepatotoxic.
. 18 Antigen and lgE ~ Omahzumab
Anti-lgE monoclonal Omalizumab- binds mostly unbound serum on mast cells
• 19 therapy lgE and blocks binding to FceRI. Used in
• 20 allergic asthma with t lgE levels resistant to ~ Steroids
• 21 inhaled steroids and long-acting 13z-agonisls.

a
Lock
s
Suspend
8
End Bl ock
Item: 6 of 37 ~. , . M k <:] t> al ~· ~
QIO: 3356 .l. ar Previous Next lab 'lifllues Notes Calculator

1 • •
FA17 p638.1
2
3
Obstructive lung Obstruction of air flow- air trapping in lungs. Airways close prematurely at high lung volumes
diseases - t RV and t FRC, t TLC. PFTs: U FEV 1, l FVC - l FEV 1/FVC ratio (hallmark),
4
V/Q mismatch. Chronic, hypoxic pulmonary vasoconstriction can lead to cor pulmonale. Chronic
5 obstructive pulmonary disease (COPD) includes chronic bronchitis and emphysema.
6 TYPE PRESENTATION PATHOLOGY OTHER
•7 Chronic bronchitis Findings: \\·heezing, crackles, Hypertrophy and hyperplasia Diagnostic criteria: productive
•8 ("blue bloater") cyanosis (hypoxemia due of mucus-secreting glands cough for> 3 months in a
•9 to shunting), dyspnea, C02 in bronchi - Reid index year for> 2 consecutive years.
retention, zopolyc)themia. (thickness of mucosal gland
0 10
layer to thickness of wall
· 11
between epithelium and
• 12 cartilage) > 50%.
• 13 Emphysema ("pink Centriacinar-associated with Enlargement of air spaces CXR: t AP diameter, flattened
• 14 puffer") smoking rJ (£). Frequently in l recoil, t compliance, diaphragm, t lung field
0 15 upper lobes (smoke rises up). l DLCO from destruction of lucency.
• 16
Panacinar-associated with alveolar walls (arrow in m). Barrel-shaped chest 1!].
a 1-antitrypsin deficiency. t elastase activity - t loss Exhalation through pursed lips
• 17
Frequently in lower lobes. of elastic fibers - t lung to increase airway pressure
• 18 compliance. and prevent airway collapse.
• 19
Asthma Findings: cough, wheezing, Branchial hypcrrcsponsi\·cncss Aspirin-i nduced asthma: COX
0 20 tachypnea, dyspnea, - reversible inhibition - leukotriene
• 21 hypoxemia, l inspiratory/ bronchoconstriction. Smooth overproduction - airway •

6
lock
s
Suspend
0
End Block
Item: 7 of 3 7 ~ 1 • M k -<:J 1>- Jil ~· !:';-~
QIO: 2355 ..L ar Pre v ious Next Lab fli!ltues Not es Calcula t o r

IAA]
& &
1
A 54-year-old woman presents w ith headache and vomiting for the past 6 hours. Her partner said they had
2
just skied the tallest mountain in Colorado. On exam, she appears m ildly confused and irritable. She has a
3 respiratory rate of 31/ min and an oxygen saturation of 85% on room air. Arterial blood gas is obtained and
4 shows the following results:
5 pH: 7.57
6 Pao 2 : 47 mm Hg
Paco 2 : 31 mm Hg
0 7
Hco 3- : 21 mEq/ L
o8
•9
• 10 A diuretic with wh ich of the following mechan isms wou ld be most beneficial in treating th is patient's alt itude
sickness?
• 11
• 12 :
A. Blocks sodium channel insertion in the collecting tubules
• 13
0 14 B. Increases osmolarity of the tubular fluid at the proxima l tubules
• 15 C. Inhibits sodium chloride reabsorption in dista l tubule
0
16
D. I nh ibits sodium/potassium/chloride cotransporter at the loop of Henle
0
17
• 18
E. Prevents carbonic acid breakdown at the proxima l tubule
• 19
• 20
• 21

a
Lock
s
Suspend
8
End Bl ock
Item: 7 of 37 ~. , . M k <:] t> al ~· ~
QIO: 2355 .l. ar Previous Next lab 'lifllues Notes Calculator

1 •

2
The correct answer is E. 76°/o chose this.
This patient is suffering from altitude sickness. Low oxygen pressure at high altitudes causes capil laries to leak,
3
leading to brain swelling (resulting in headache and altered mental status) and pulmonary edema (respiratory
4 distress). Additionally, low oxygen induces tachypnea, causing loss of C0 2 and a respiratory al kalosis.
5 Acetazolamide is a carbon ic anhydrase inhibitor used to treat altitude sickness. Car bonic anhyd rase is an
6
enzyme used to reversibly convert carbonic acid (Hco 3 - and H+) to C0 2 and H20, depending on the site of
activity . Blocking carbonic anhydrase in the rena l tubules results in Hco3 - wasting and diuresis. Diuresis
7
decreases the swelling in the brain, and wasting Hco3 - leads to a metabolic acidosis, wh ich wou ld help th is
•8
patient's acid-base status . Acetazolamide is also used to manage urinary alkalinization (eg, treatment of
•9 cystinuria), pseudotumor cerebri, and glaucoma .
Cystinuria Acetazolamide Respiratory alkalosis Glaucoma Metabolic acidosis Altitude sickness Tachypnea Carbonic anhydrase inhibitor Carbonic acid
• 10
Pulmonary edema Idiopathic intracranial hypertension Carbonic anhydrase Enzyme Capillary Acidosis Diuretic Edema Alkalosis Polyuria Headache
· 11
• 12 Arterial blood gas Dyspnea Altered level of consciousness Oxygen Cerebral edema Kidney Metabolism Partial pressure

• 13 A is not correct. 4°/o chose this .


• 14 This mechanism describes the potassium-sparing diuretic spironolactone, wh ich is an aldosterone antagonist
• 15 and prevents sod ium channel insertion on the luminal side of the collecting tubu le (norma l action of
• 16
aldosterone). Although effective for decreasing cerebral edema th rough diuresis, potassium-spar ing diu retics
are not used in the treatment of altitude sickness .
• 17 Potassium-sparing diuretic Antimineralocorticoid Spironolactone Diuretic Aldosterone Cerebral edema Collecting duct system Sodium channel Sodium
• 18 Receptor antagonist Edema Polyuria Forced diuresis Nephron
• 19
B is not correct. 7°/o chose this .
• 20
This mechanism describes an osmotic diuretic, mannitol. Mannitol is a sugar molecule that is filtered by the
• 21 kidneys but cannot be reabsorbed, so it increases the osmolarity of the flu id, pu llinq water with it, thus causinq •

6
lock
s
Suspend
0
End Block
Item: 7 of 37 ~. , . M k <:] t> al ~· ~
QIO: 2355 .l. ar Previous Next lab 'lifllues Notes Calculator

1

2 A is not correct. 4°/o chose this.


3 This mechanism describes the potassium-sparing diuretic spirono lactone, wh ich is an aldosterone antagonist
and prevents sod ium channel insertion on the lumina l side of the col lecting tubu le (norma l action of
4
aldosterone). Although effective for decreasing cerebral edema th rough diuresis, potassium-spar ing diuretics
5 are not used in the treatment of altitude sickness .
6 Potassium-sparing diuretic Antimineralocorticoid Spironolactone Diuretic Aldosterone Cerebral edema Collecting duct system Sodium channel Sodium

7 Receptor antagonist Edema Polyuria Forced diuresis Nephron

•8
B is not correct. 7°/o chose this .
•9
This mechanism describes an osmotic diuretic, mannitol. Mannitol is a sugar mo lecu le that is filtered by the
• 10 kidneys but cannot be reabsorbed, so it increases the osmolarity of the f luid, pu lling water with it, thus causing
· 11 diuresis. Mannitol is used common ly in t rauma where there is high intracranial pressure, but it is not used to
treat altitude sickness because mannitol also can worsen pulmonary edema .
• 12
Diuretic Mannitol Pulmonary edema Intracranial pressure Altitude sickness Osmotic concentration Osmosis Edema Kidney Polyuria Osmotic diuretic
• 13
Forced diuresis Sugar Major trauma
• 14
• 15
C is not correct. 6°/o chose this .
This mechanism describes th iazide diuretics (eg, hydroch lorothiazide [ HCTZ]). HCTZ is not used for managing
• 16
alt itude sickness, as it can further this patient's al kalosis .
• 17 Hydrochlorothiazide Thiazide Diuretic Alkalosis Altitude sickness

• 18
D is not correct. 7°/o chose this .
• 19
This mechanism describes loop diuretics (eg, furosemide) . Although they are more powerful diuretics than
• 20 acetazolamide, loop diuretics can cause alka losis, which would worsen this patient's acid-base status.
• 21 Acetazolamide Furosemide Diuretic Alkalosis Arterial blood gas loop diuretic

6
lock
s
Suspend
0
End Block
Item: 7 of 37 ~. , . M k <:] t> al ~· ~
QIO: 2355 .l. ar Previous Next lab 'lifllues Notes Calculator

1 • kidneys but cannot be reabsorbed, so it increases the osmolarity of the flu id, pu lling water with it, thus causing
2 diuresis. Mannitol is used common ly in t rauma where there is high intracranial pressure, but it is not used to
treat altitude sickness because mannitol also can worsen pulmonary edema .
3
Diuretic Mannitol Pulmonary edema Intracranial pressure Altitude sickness Osmotic concentration Osmosis Edema Kidney Polyuria Osmotic diuretic
4
Forced diuresis Sugar Major trauma
5
6
C is not correct. 6°/o chose this.
This mechanism describes th iazide diuretics (eg, hydroch lorothiazide [ HCTZ]). HCTZ is not used for managing
7
altitude sickness, as it can further this patient's al kalosis .
•8 Hydrochlorothiazide Thiazide Diuretic Alkalosis Altitude sickness

•9
D is not correct. 7°/o chose this .
• 10
This mechanism describes loop diuretics (eg, furosemide) . Although they are more powerful diuretics than
· 11 acetazolamide, loop diuretics can cause alka losis, which would worsen this patient's acid-base status.
• 12 Acetazolamide Furosemide Diuretic Alkalosis Arterial blood gas loop diuretic

• 13
• 14
Bottom Line:
• 15
Acetazolamide is a carbon ic inhibitor diuretic that causes bicarbonate wasting and metabolic acidosis . It is used
• 16 to treat altitude sickness, pseudotumor cerebri, glaucoma, and urinary alka linization. It is importa nt to note
• 17 that early recognition and treatment of high-altitude cerebral edema are critical, and that descent is the
• 18 definitive t reatment and should begin immediately at the first suspicion of this diagnosis .
Acetazolamide Glaucoma Diuretic Metabolic acidosis Idiopathic intracranial hypertension Altitude sickness Cerebral edema High-altitude cerebral edema Edema
• 19
Acidosis Bicarbonate Metabolism Enzyme inhibitor
• 20
• 21

6
lock
s
Suspend
0
End Block
Item: 7 of 3 7 ~ 1 • M k -<:J 1>- Jil ~· !:';-~
QIO: 2355 ..L ar Pre v ious Next Lab fli!ltues Not es Calcula t o r
& &
1
FA17 p 575.2
2
Acetazolamide
3
MECHAN ISM Carbonic anhydrase inhibitor. Causes self-
4
limited aHC03 diuresis and l total body
5
HC0 3- stores.
6
CLINICALUSE Glaucoma, urinary alkalinization, metabolic
7 alkalosis, altitude sickness, pscudotumor
·8 cerebri.
.9
• 10
• 11

• 12
• 13 ADVERSE EFFEC TS Proximal renal tubular <lcidosis, parcsthcsias, ".\ClD"azolamide causes ACIDosis .
• 14 H3 toxicity, su lfa allergy, h)rpokalcmia .
• 15
• 16 FA17 p 575.3
• 17 Loop diuretics
. 18 Furosemide, bumetanide, torsemide
• 19 MECHANISM Sulfonamide loop diuretics. Inhibit cot ransport
• 20 system ( la+JK+f2CI-) of thick ascend ing limb
• 21
of loop of llcnlc. Abolish hypcrtonicit) of

a
Lock
s
Suspend
8
End Bl ock
Item: 7 of 3 7 ~ 1 • M k -<:J 1>- Jil ~· !:';-~
QIO: 2355 ..L ar Pre v ious Next Lab fli!ltues Not es Calcula t o r
&
1 Loop diuretics &

2 Furosemide, bumetanide, torsemide


3 MECHANISM Sulfonamide loop diuretics. Inhibit cotransport
4 system (1 a+fK' iZCl-) of thick ascending limb
5 of loop of llcnlc. Abolish hypcrtonicit) of
6
medulla, preventing concentration of urine.
Stimulate PC E release (\'asodilatory effect
7
on afferent arteriole); inhibited b) i\'SA ID~.
·8 t CaZ+ excretion. Loops Lose Ca 2• .
.9
CLINICAL USE Edematous states (H F, cirrhosis, nephrotic
• 10 syndrome, pulmonar) edema), h) pertension,
• 11 hypercalcem ia .
• 12 ADVERSE EFFECTS O totoxicity, llypokalcmia, Ilypomagncscmia, O llll D.\.\1'\C!
• 13 Dehydration, Allergy (sulfa), metabolic
• 14
Alkalosis, Nephritis (interstitia 1), Gout.
• 15 Ethacrynlc acid

• 16 MECHANISM Nonsulfonamide inhibitor of cotransport system


(Na+fK+fZCl -) of thick ascending limb of loop
• 17
of llcnlc.
. 18
CLINICAL USE Diuresis in patients allergic to sulfa drugs.
• 19
ADVERSE EFFECTS Similar to furosemide, but more ototoxic. Loop earrings hurt your ears.
• 20
• 21

a
Lock
s
Suspend
8
End Bl ock
Ite m: 8 of 3 7 ~ 1 • M k -<:J 1>- Jil ~· !:';-~
QIO: 2660 ..L ar Previous Next La b fli!ltues Not es Calculator

IAA]
& &
1
A 7-year-old boy presents for t he t hird t ime in 1 month because of acute onset cough, dyspnea, and
2
wheezing. His symptoms improve with administration of a short-acting inhaled ~ragonist, which he uses at
3 home as well whenever he begins feeling short of breath. He uses no other medications for his condition. On
4 discharge the boy is prescribed an add itional drug that acts by inhibiting a transcription factor required for the
synthesis of inflammatory mediators such as tumor necrosis factor-a.
5
6
Which of the following is a potential adverse effect of the newly prescribed drug if used chronically?
7
·8 :
A. Cardiotoxicity
.9
• 10 B. Headache
• 11 C. Oral thrush
• 12
D. Tachycard ia
• 13
• 14
E. Tremor
• 15
• 16
• 17
• 18
• 19
• 20
• 21

a
Lock
s
Suspend
8
End Block
Item:8of37 ~. , . M k <:] t> al ~· ~
QIO: 2660 .l. ar Previous Next lab 'lifllues Notes Calculator

1 •
The correct answer is C. 73°/o chose this.
2
This child presented with symptoms consistent with an asth ma exacerbation.
3 He was likely prescribed an inhaled corticosteroid (eg, beclomethasone),
4 which acts by inactivating nuclear facto r-KB . However, the immune-
5 modu lating properties of inhaled steroids also attenuate the local immune
response of the oropharynx and upper airways to the extent that oral thrush
6
can deve lop (shown in the image). Chi ldren are especial ly vulne rable to this.
7 Ora l thrush is the most common local side effect of inhaled corticosteroids.
8 Other local side effects to be aware of include sore th roat and hoarseness,
•9 and systemic effects can include decreased bone mine ral density (causing an
increased fracture r isk), skin bruising, cataracts, and suppressed
• 10
hypothalamic-pituitary-adrenal axis function.
· 11 Corticosteroid Beclometasone dipropionate Pharynx Asthma Oral candidiasis Steroid Immune system Dysphonia
Image copyright © 2010 Negrato
• 12 Hypothalamic-pituitary-adrenal axis Cataract Side effect Bone Candidiasis Bone density Sore throat and Tarzia; licensee BioMed
• 13 Adverse effect Adverse drug reaction
Central Ltd.
• 14
A is not correct. 7°/o chose this .
• 15
Cardiotoxicity is not a common adverse effect of inhaled corticoste roid therapy, wh ich this ch ild is like ly
• 16 receiv ing. An example of a drug that can cause ca rdiotoxicity would be the methylxanthines (eg, theophyll ine),
• 17 which are sometimes used in asthma prophylaxis. They wo rk by inh ibiting phosphodiesterase, inhibiting the
• 18
hydrolysis of cAMP. Adve rse effects of chronic the rapy may include cardiotoxicity and neurotoxicity .
Corticosteroid Theophylline Asthma Adverse effect Cardiotoxicity Neurotoxicity Xanthine Hydrolysis Preventive healthcare Phosphodiesterase
• 19
• 20 B is not correct. 6°/o chose this .
• 21
Headache is a common adverse effect of drugs like zi leuton. Zileuton is a lipoxygenase inh ibitor used to manage
• ;.c::thn"l;. hv nr-.:>v.:>nti nn 1.:>1d,-ntr-i.:>n.:> nr-nrl1 1rtinn· it h;,c:: nn .:>ff.:>rt nn h l n"ll"l r" n.:>r r-nc:: ic:: f;.rtnr--n /lc:: n"l;>n\1 ;,c:: ?t; O/n nf

6
lock
s
Suspend
0
End Block
Item:8of37 ~. , . M k <:] t> al ~· ~
QIO: 2660 .l. ar Previous Next lab 'lifllues Notes Calculator

• Corticosteroid Theophylline Asthma Adverse effect Cardiotoxicity Neurotoxicity Xanthine Hydrolysis Preventive healthcare Phosphodiesterase
1
2 B is not correct. 6°/o chose this.
3 Headache is a common adverse effect of drugs like zi leuton. Zileuton is a lipoxygenase inh ibitor used to manag e
4 asthma by preventing leukotriene production; it has no effect on t umor necrosis factor-a . As many as 25% of
patient s using this drug experience headache . Headache is not, however, a common adverse effect of chron ic
5
inhaled corticostero id use, wh ich is what this child is likely using .
6 Corticosteroid Zileuton leukotriene Asthma Necrosis Headache Neoplasm Arachidonate 5-lipoxygenase inhibitor lipoxygenase Enzyme inhibitor Adverse effect

7
D is not correct. 7°/o chose this.
8
Tachyca rdia may resu lt from t he use of nonspecific ~ - agonists, but wou ld not be expect ed t o occu r in the setting
•9 of ch ron ic inha led corticosteroid t herapy.
• 10 Corticosteroid Tachycardia

· 11 E is not correct. 7°/o chose this .


• 12 Tremors can result from use of long - acting ~ 2 - agonists (eg, salmeterol), which are another prophylact ic
• 13 t reatment for chronic asthma . Adverse effects associat ed with t hese agents include tremor and arrhythmia .
• 14 However, this ch ild is likely receiv ing inhaled corticosteroids, wh ich do not cause tremors .
Salmeterol Asthma Tremor Cardiac arrhythmia Corticosteroid Preventive healthcare
• 15
• 16
• 17 Bottom Line:
• 18 Inhaled corticosteroids are an establ ished therapy for ch ron ic ast hma, but their use may attenuate loca l
• 19 immune defenses, pred isposing patients, particularly ch ildren, to oral thrush .
Asthma Oral candidiasis Corticosteroid Immune system Thrush (bird) Candidiasis
• 20
• 21

6
lock
s
Suspend
0
End Block
Item: 8 of 3 7 ~ 1 • M k -<:J 1>- Jil ~· !:';-~
QIO: 2660 ..L ar Pre v ious Next Lab fli!ltues Notes Calcula t o r
& &
1
FA17 p649.1
2

3
Asthma drugs Bronchoconstriction is mediated b) (I) inOammatory processes and (2) parasympathetic lone;
therapy is directed at these 2 pal hways.
4
5
Albuterol- rclaxes bronchial smooth muscle (short acting 13z-agonist). Used during acute
exacerbation.
6
Salmeterol, formoterol - long-acling agents for prophylaxis. Ad,·erse effects are tremor and
7
arrh' thmia.
8
Inhaled Fluticasone, budesonide- inhibit the synthesis of \'irtually all cytokines. Inacti,-atc NF-KB, the
.9
corticosteroids transcription factor that induces production ofT 'F-cx and other in8ammatory agents. 1st-line
• 10 therapy for chronic asthma. Vlay cause oral thrush.
• 11 Muscarinic Tiotropium, ipratropium-competitivel) bloc!.. muscarinic receptors, preventing
• 12 antagonists bronchoconstriction. Also used for COPD. Tiotropium is long acting.
• 13 Antileukotrienes Montelukast, zafirlukast- bloc!.. lcul..otricnc Exposure to antigen
• 14 receptors (CysLTl). Especially good for (dust pollen, etc)

• 15 aspirin-induced and exercise-induced asthma .


• 16
Zileuton -5-lipoxygenase pathway inh ibitor. ttr Avoidance
Blocks conversion of arachidonic acid to
• 17
leukotrienes. Hepatotoxic.
. 18
Anti-lgE monoclonal Omalizumab-binds mostly unbound serum Antigen and tgE ttr Omatizumab
on mast cells
• 19 therapy lgE and blocks binding to FctRI. Used in
• 20 allergic asthma with t lgE b ·cls resistant to .._t:L Steroids
• 21 inhaled steroids and long-acting 13z-agonisls.

a
Lock
s
Suspend
8
End Bl ock
Item: 8 of 3 7 ~ 1 • M k -<:J 1>- Jil ~· !:';-~
QIO: 2660 ..L ar Pre v ious Next Lab fli!ltues Notes Calcula t o r
& &
1
FA17 p 149.1
2
Opportunistic fungal infections
3
Candida albicans alba = '' hite. Dimorphic; forms pseudohyphae and budding) easts at 20°C fJ, germ tubes at
4
37°C rn.
5
Systemic or superficial fungal infection. Causes oral and esophageal thrush in
6 immunocompromised (neonates, steroids, diabetes, AI OS),' uho,·aginitis (diabetes, use of
7 antibiotics), diaper rash, endocarditis (IV dmg users), disseminated candidiasis (espcciallr in
8 neutropenic patients}, chronic mucocutaneou~ candidiasis.
.9
Treatment: oral Auconazole/topical azoic for vaginal; nystatin, Auconazole, or caspofungin for oral/
esophageal; fluconazole, caspofungin, or amphotericin B for systemic.
• 10
Aspergillus Septate hyphae that branch at 45° Acute Angle • . Produces conidia in radiating chains at end of
• 11
fumigatus conidiophore 0 .
• 12
Causes invasive aspergillosis in immunocompromised, patients \\'ith chronic granulomatous disease.
• 13 Can cause aspergillomas in pre-existing lung cavities, especially afterTB infection .
• 14 Some species of Aspergillus produce AOatoxins (associated with hepatocellular carcinoma).
• 15 Allergic bronchopulmonary aspergillosis ( BPA}: hypersensitivity response associated with
• 16 asthma and cystic fibrosis; may cause bronchiectasis and eosinophi lia.
• 17 Cryptococcus 5-10 !Jm with narrow budding. lleavily encapsulated yeast. l ol dimorphic.
• 18 neoformans Found in soil, pigeon droppings. Acquired through inhalation with hematogenous dissemination
• 19 to meninges. Culture on Sabouraud agar. l ligh lighted with India ink (clear halo ) and
mucicarmine (red inner capsule [!)). Latex agglutination lest detects polysaccharide capsular
• 20
antigen and is more specific.
• 21 Causes crn>tococcosis, cryptococcal meningitis, cryptococcal encephalitis (''soap bubble" lesions

a
Lock Suspend
s 8
End Bl ock
Item: 9 of 3 7 ~ 1 • M k -<:J 1>- Jil ~· !:';-~
QIO: 1339 ..L ar Pre v ious Next Lab fli!ltues Not es Calcula t o r

IAA]
& &
1
A 12-year-old Sout h Asian boy with a past med ical history of severe asthma presents to his physician. He
2
has recently been more compl iant with his medications because of the worsening severity of his respiratory
3 symptoms. On physical examination, the physician sees white patches in the oropharynx that can be scraped
4 off with a tongue blade.
5
6 Which of the following mechanisms of action corresponds to the medications that may have contributed to the
pathology described?
7
8 :
A. a-Agonism
.9
• 10 B. Bronchodilation by way of ~ - agonism

• 11 C. Decreased transcription of inflammatory cytokines


• 12
D. Leukotriene inhibition
• 13
• 14
E. Phosphodiesterase inhibition
• 15
• 16
• 17
• 18
• 19
• 20
• 21

a
Lock
s
Suspend
8
End Bl ock
Item:9of37 ~. , . M k <:] t> al ~· ~
QIO: 1339 .l. ar Previous Next lab 'lifllues Notes Calculator

1 •

2
The correct answer is C. 69°/o chose this.
The pathology described is oral cand idiasis, also known as thrush, as shown in the
3
image. It is common ly seen in immunocompromised patients as well as in smal l ch ildren .
4 I n this case, the patient has deve loped thrush as a result of increased use of aerosol
5 corticosteroids such as beclomethasone. Corticosteroids suppress the inflammatory and
6 immune response, mainly by downregu lating the production of inflammatory proteins.
The exact mechanisms of action of aerosol corticosteroids like beclamethasone are
7
unknown, but they are believed to drive general anti-inflammatory and vasoconstricting
8 responses on administration . When inhaled, they general ly do not have the systemic side
9 effects of ora l corticosteroids, but can predispose to th r ush . Patients who rinse their
0 10
mouths after the use of an inhaled steroid, who learn to use their inha lers proper ly, and
who use a spacer are less likely to exper ience oral cand idiasis resulting from inha led
· 11 corticosteroid use.
0
12 Corticosteroid Oral candidiasis Beclometasone dipropionate Candidiasis Immunodeficiency Steroid Immune system
Image copyright Mol
13 Vasoconstriction Anti -inflammatory Aerosol Pathology
Indian J Palliative
0

0 14 Care
0 15
0
16
A is not correct. 2°/o chose this.
0 17
a1 -Agonists such as phenylephrine are sometimes used as a nasa l decongestant but are not known to
pred ispose patients to th r ush . Thei r mechanism of action rel ies on their ability to cause vasoconstriction in the
0
18 blood vessels of the nose and ears, decreasing swe lling in these areas. As a result, this medication is common ly
0 19 used for temporary relief of nasal congestion, sinus, and ear symptoms caused by the common cold, flu, or
0 20 allergies . It is also used to di late pupils and is contraind icated in the setting of narrow angle g lacoma . Other
known adverse effects of phenyleph r ine include rebound nasa l congestion and high blood pressu re.
0
21 Phenylephrine Deconqestant Vasoconstriction Common cold Nasal conqestion Blood pressure Hypertension Allerqy Contraindication Mechanism of action

6
lock
s
Suspend
0
End Block
Item:9of37 ~. , . M k <:] t> al ~· ~
QIO: 1339 .l. ar Previous Next lab 'lifllues Notes Calculator

2
B is not correct. 13% chose this.
~-Agonists, such as the short-acting ~-agon i st albuterol, are a common treatment for asthma but are not known
3
to increase the risk of thrush. Albuterol is a sympathomimetic bronchod ilator given for treatment or prevention
4 of bronchospasm and can be given to patients with asthma. Important side effects of ~-agon i sts include
5 tachycardia and hypokalemia.
Bronchodilator Hypokalemia Bronchospasm Sympathomimetic drug Asthma Tachycardia Salbutamol Adverse drug reaction Side effect
6
7 D is not correct. 12% chose this.
8 Zafirlukast, a leukotriene receptor blocker, is used for asthma t reatment but does not predispose patients to
9 thrush. I t is not effective for acute asthma attacks, but is used for prevention of symptoms. Common side
effects of leukotriene receptor blockers include headache, transaminitis, nausea, dia r rhea and insomnia. These
0 10
agents may rarely cause vascul itis.
· 11 Zafirlukast Asthma Insomnia leukotriene Vasculitis Nausea Diarrhea Headache Receptor (biochemistry) Side effect Adverse drug reaction

0
12
E is not correct. 4°/o chose this.
0
13
Theophylline, a phosphod iesterase inhibitor, is used for asthma treatment but is not associated with thrush .
0 14 Theophylline is a xanthine derivative natura lly found in cocoa beans and is used in therapy for ch ron ic
0 15 obstructive pu lmonary diseases, infant apnea, and for asthma . Common reactions associated with theophy lline
0
16
include vomiting, headache, insomnia, irritability, and tremor. Th is med ication is uncommonly used due to
ser ious reactions includ ing seizures and arrhythmias.
0 17 Phosphodiesterase inhibitor Theophylline Xanthine Asthma Insomnia Cocoa bean Headache Epileptic seizure Vomiting Tremor Apnea of prematurity
0
18 Phosphodiesterase Cardiac arrhythmia Chronic obstructive pulmonary disease Theobroma cacao Apnea Irritability Enzyme inhibitor
0 19
0 20
Bottom Line:
0
21

6
lock
s
Suspend
0
End Block
Item:9of37 ~. , . M k <:] t> al ~· ~
QIO: 1339 .l. ar Previous Next lab 'lifllues Notes Calculator

1 • of bronchospasm and can be given to patients with asthma. I mportant side effects of ~-agon i sts include
2
tachycardia and hypokalemia.
Bronchodilator Hypokalemia Bronchospasm Sympathomimetic drug Asthma Tachycardia Salbutamol Adverse drug reaction Side effect
3
4 D is not correct. 12% chose this.
5
Zafirlukast, a leukotriene receptor blocker, is used for asthma t reatment but does not predispose patients to
thrush. I t is not effective for acute asthma attacks, but is used for prevention of symptoms. Common side
6 effects of leukotriene receptor blockers include headache, transaminitis, nausea, dia r rhea and insomnia. These
7 agents may rarely cause vascul itis.
Zafirlukast Asthma Insomnia leukotriene Vasculitis Nausea Diarrhea Headache Receptor (biochemistry) Side effect Adverse drug reaction
8
9 E is not correct. 4°/o chose this.
0 10 Theophylline, a phosphod iesterase inhibitor, is used for asthma treatment but is not associated with thrush .
· 11 Theophylline is a xanthine derivative natura lly found in cocoa beans and is used in therapy for ch ron ic
obstructive pu lmonary diseases, infant apnea, and for asthma . Common reactions associated with theophy lline
0
12
include vomiting, headache, insomnia, irritability, and tremor. Th is med ication is uncommonly used due to
0
13 ser ious reactions includ ing seizures and arrhythmias.
0 14 Phosphodiesterase inhibitor Theophylline Xanthine Asthma Insomnia Cocoa bean Headache Epileptic seizure Vomiting Tremor Apnea of prematurity

0 15 Phosphodiesterase Cardiac arrhythmia Chronic obstructive pulmonary disease Theobroma cacao Apnea Irritability Enzyme inhibitor

0
16
0 17
Bottom Line:
0
18
Ora l candid iasis manifests with white, stuck-on plaques in the oropharynx and often occu rs in patients who
19
0
have an inadequate immu ne response .
0 20 Oral candidiasis Candidiasis Pharynx Immune system

0
21

6
lock
s
Suspend
0
End Block
Item: 10 of 3 7 ~ 1 • M k -<:J 1>- Jil ~· !:';-~
QIO: 5129 ..L ar Pre v ious Next Labfli!llues Notes Calcula t o r

IAA]
& &
1
A 25-year-old woman presents to the emergency department with shortness of breath and the sound heard
2
in the aud io clip. She states that she has a history of asthma for wh ich she is on maintenance therapy. She
3 has been admitted to the hospital four times in the past year because of severe asthma exacerbations. After
4 initial t reatment in the emergency department, she feels much better. The resident notices a slight limp as the
patient wa lks to the restroom. Upon questioning, the patient mentions a several-month history of hip pain and
5
denies any injury to the area.
6
OPE N MEDIA
7
8
9 Which medication has likely contributed to the patient's symptoms?
• 10
:
• 11 A . Albuterol
• 12
B. Fluticasone
• 13
C. Methylprednisolone
• 14
• 15 D. Montelukast
• 16 E. Salmeterol
• 17
• 18
• 19
• 20
• 21

a
Lock
s
Suspend
8
End Bl ock
Item: 10 of 3 7 ~ 1 • M k -<:J 1>- Jil ~· !:';-~
QIO: 5129 ..L ar Pre v ious Next Labfli!llues Notes Calcula t o r
& &
1 The correct answer is C. 58°/o chose this.
2 The patient described in the vignette is suffering an acute asthma
3 exacerbation, as evidenced by expiratory wheezes heard on her lung exam.
4 Al lergen-induced asthma attacks may be worse in certain seasons, such as
spring and fa ll. This patient has been hospitalized frequently and also takes
5
maintenance medication (likely a combined long-acting 13-agonist and inhaled
6 corticosteroid). The patient's isolated limp and hip pain, in the absence of
7 other symptoms indicating a rheumatologic process, are concerning for
medication-induced avascular necrosis of the femoral head (shown in this
8
image). This is a well-known complication of repeated and chronic oral and
9 intravenous steroid use. Other long -term adverse effects of steroid use
10 include abdominal striae and increased girth, suppression of the
• 11 hypothalamic-pituitary-adrena l axis, hyperglycemia, osteoporosis, moon
facies, buffalo hump, immunosuppression, and impaired wound healing . It is
• 12
important to note that only systemic corticosteroids have these significant Image courtesy of Wikimedia
• 13 side effects. Inhaled corticost eroids do not have the same side effects . Commons
• 14 Corticosteroid Hypothalamic-pituitary-adrenal axis Osteoporosis Avascular necrosis Hyperglycemia Asthma
Immunosuppression Intravenous therapy Lung
• 15
Rheumatology Lipodystrophy Necrosis Steroid Wound healing Blood vessel Wheeze Pharmaceutical drug
• 16
• 17 A is not correct. 5°/o chose this .
• 18 Albuterol is a short-acting 13ragonist used for immediate relief of bronchospasm associated with asthma. It is
not associated with joint pain or avascu lar necrosis. Adverse effects include tachycardia and anxiety .
• 19
Avascular necrosis Bronchospasm Asthma Tachycardia Arthralgia Salbutamol Anxiety Necrosis Blood vessel
• 20
B is not correct. 21 °/o chose this .
• 21

a
Lock
s
Suspend
8
End Bl ock
Item: 10 of 37 ~. I • M k <:] t> al ~· ~
QIO: 5129 .l. ar Previous Next lab 'lifllues Notes Calculator

1 l • ' e I I I

not associated with joint pain or avascu lar necrosis. Adverse effects include tachycardia and anxiety.
2
Avascular necrosis Bronchospasm Asthma Tachycardia Arthralgia Salbutamol Anxiety Necrosis Blood vessel
3
B is not correct. 21% chose this.
4
Fluticasone is an inha led corticosteroid often paired with long-acting ~-agonists such as salmeterol for daily use.
5
At the doses given for inhalational fluticasone, it does not have sufficient bioavailabi lity to induce this patient's
6 hip pain . Severe adverse effects are less frequent ly associated with inhaled corticoste roids. Howeve r, this
7 depends somewhat on age of patient and duration of exposure. Fluticasone is somewhat special in this regard
8
because it is extensively inactivated by hepatic fi rst-pass metabol ism. Thus it is safer in pediatric patients who
might swal low a significant amount of drug. Additional ly, inhaled ste roids are not associated with the seve re
9 adverse effects that occu r with system ic administration of steroids, but they do increase the risk of mucosal
10 infections such as ora l thrush.
Corticosteroid Salmeterol First pass effect Oral candidiasis Bioavailability Fluticasone Steroid Metabolism liver Anabolic steroid
· 11
• 12 D is not correct. 9°/o chose this .
• 13 Montelukast is a leukotriene receptor antagonist used for maintenance in asthma and allergy sufferers. Adve rse
• 14 effects include gastrointestinal distress, hype rsensitivity reaction, and sleep distu rbances .
Antileukotriene Montelukast leukotriene Asthma Allergy Receptor antagonist Hypersensitivity Gastrointestinal disease Receptor (biochemistry)
• 15
Gastrointestinal tract
• 16
• 17 E is not correct. 7°/o chose this .
• 18 Salmeterol is a long-acting ~ragonist used in combination inhalers for daily maintenance. Adve rse effects are
simila r to those associated with short-acting ~-agonists. In addition, when used alone, salmetero l has been
• 19
shown to cause a smal l documented increase in asthma-re lated deaths .
• 20 Salmeterol

• 21

6
lock
s
Suspend
0
End Block
Item: 10 of 37 ~. I • M k <:] t> al ~· ~
QIO: 5129 .l. ar Previous Next lab 'lifllues Notes Calculator

1 • because it is extensively inactivated by hepatic first-pass metabolism. Thus it is safer in pediatric patients who
2
might swallow a significant amount of drug. Additional ly, inhaled steroids are not associated with the severe
adverse effects that occur with system ic administration of steroids, but they do increase the risk of mucosal
3 infections such as oral thrush.
4 Corticosteroid Salmeterol First pass effect Oral candidiasis Bioavailability Fluticasone Steroid Metabolism liver Anabolic steroid

5
D is not correct. 9°/o chose this.
6 Montelukast is a leukotriene receptor antagonist used for maintenance in asthma and allergy sufferers. Adverse
7 effects include gastrointestinal distress, hypersensitivity reaction, and sleep disturbances.
Antileukotriene Montelukast leukotriene Asthma Allergy Receptor antagonist Hypersensitivity Gastrointestinal disease Receptor (biochemistry)
8
Gastrointestinal tract
9
10 E is not correct. 7°/o chose this.
· 11 Salmeterol is a long-acting ~ragonist used in combination inhalers for daily maintenance. Adverse effects are
• 12 similar to those associated with short-acting ~-agonists. In addition, when used alone, salmeterol has been
shown to cause a smal l documented increase in asthma-related deaths .
• 13
Salmeterol
• 14
• 15
• 16 Bottom Line:
• 17 Long-term adverse effects of steroid use include abdomina l striae and increased girth, suppression of the
• 18
hypothalamic-pituitary-adrenal axis, hyperglycemia, osteoporosis, moon facies, buffalo hump,
immunosuppression, avascular necrosis, and impa ired wound healing .
• 19 Osteoporosis Avascular necrosis Hyperglycemia Hypothalamic-pituitary-adrenal axis Immunosuppression Steroid Blood vessel Necrosis lipodystrophy
• 20 Wound healing

• 21

6
lock
s
Suspend
0
End Block
Item: 10 of 3 7 ~ 1 • M k -<:J 1>- Jil ~· !:';-~
QIO: 5129 ..L ar Pre v ious Next Labfli!llues Notes Calcula t o r
& &
1

2 FA17 p649.1
3
Asthma drugs Bronchoconstriction is mediated b) (I) inOam malory processes and (2) parasympal hel ic lone;
4 therapy is directed at these 2 pathways.
5 ~-agonist s Albuterol- rclaxes bronchial smooth muscle (short acti ng 13z-agonist). Used during acute
6 exacerbation.
7 Salmeterol, fo rmote ro l- long-acting agents for prophylaxis. ,\dverse effects are tremor and
8 arrh' thmia.
9 Inhaled Fluticasone, budesonide - inhibit the synthesis of \'irtually all cytokines. Inacti\<Jtc N F-KB, the
10 corticoste roids transcription factor that induces produc tion of T 'F-a and other inflamm atory agents. 1st-line
• 11
therapy for chronic asthma. Vlay cause oral thrush .

• 12
Muscarinic Tiotro pium, ipratro pium- competitively bloc!. muscarin ic receptors, preventing
a ntago nists bronchoconstriction. Also used for C OPD. Tiolropium is long acting.
• 13
Antile ukotrienes Montelukast, zafirlukast - bloc!. leu l-ot ricnc Exposure to antigen
• 14
receptors (C ysLTl). Especially good for (dust pollen. etc)
• 15
aspirin-induced and exercise-induced :~ sthma .
• 16 Zileuton - 5-lipoxygcnasc pathway inh ibitor. t-0- Avoidance
• 17 Blocks conversion of arac hidonic acid to
. 18 leukotrienes. Hepatotoxic.
• 19 Anti-lgE monoclo nal Omalizumab-binds mostly unbound serum
Antigen and lgE t-0- Omalizumab
on mast celts
• 20
the rapy lgE and blocks binding to Fc£RI. Used in
allergic asthma with t lgE levels resistant to
• 21 ~ Steroids
• inh:.I Pcl ~IPrni cl~ :.n rlln n o-:.l'l ino 1\...:.onni~l ~

a
Lock
s
Suspend
8
End Bl ock
Item: 10 of 3 7 ~ 1 • M k -<:J 1>- Jil ~· !:';-~
QIO: 5129 ..L ar Pre v ious Next Labfli!llues Notes Calcula t o r
t • • t t W • I • I
1 Anti lgE monoclonal Omahzumab-bmds mostly unbound serum on mast cells
2 therapy lgE and blocks binding to FctRI. Used in
allergic asthma with t IgE levels resistant to
3 ~ Stero•ds
inhaled steroids and long-acting ~-agonisls.
4
5
Methylxanthines Theophylline- likely causes bronchodilation Med~ators
6 by inhibiting phosphodiesterase - t c1\ \I P (leukotrienes. histamine. etc)
7 b·els due to 1 ct\~ IP hydrolysis. Usage is
8 limited because of narrO\\ therapeutic index 1}-agomsts
(cardiotoxicit}', neurotoxicity); metabolized Theophy~hne ___r.::..__.
9
10
.
br. cvtochrome P-450. Blocks actions of Muscanmc
antagonists
~

adenosine.
• 11
ATP
• 12
Bronchodilation ~ ~agonists
• 13 Early response: Late response .
bronchoconstriction inflammation
• 14 cAMP
• 15
Bronchial tone
• 16
• 17 Bronchial
Symptoms
hyperreactiVity
• 18 ACh -{+~ +--< + r- Adenosine
• 19 Mu.scarinic ~ "R Theophylline
• 20 antagonists
Bronchoconstriction
• 21

a
Lock
s
Suspend
8
End Bl ock
Bronchoditation
2 Early response: Late response
bronchoconstriction inflammation
3
4
Bronchial tone
5
6 AMP Bronchial
Symptoms
hyperreactrv1ty
7 ACh - • - Adenosine

8 Muscarinic ~ ~ Theophylline
9 antagonists
Broochoconstriction
10
• 11
FA17 p 436.3
• 12
Osteonecrosis Infarction of bone and marrow, usually \'Cry Branch of
• 13
(avascular necrosis) painfuL lost common site is femoral obturator artery ...
-ur~

• 14 head rJ (due to insuffi ciency of media I


• 15 circumAex femoral <Htcry). Causes include
Medial femoral
• 16 Corticosteroids, Alcoholism, Sickle cell circumflex
• 17
disease, Trauma, "the Bends" (c11isson/ artery (posterior)
decompression disease), LEgg-Calve-Perl hes Lateral femoral
• 18 circumflex
disease (idiopath ic), Gaucher disease, Slipped
• 19 artery (anterior)
capital femoral epiphysis-C.\ ST Bent I.EGS.
• 20
• 21

a
Lock
s
Suspend
8
End Block
Item: llof37 ~. , . M k <:] t> al ~· ~
QIO: 3268 .l. ar Previous Next Lab 'lifllues Notes Calculator

1 •
A 33-year-old immigrant from Peru comes to a women's hea lth clinic because she has missed her period for ~~AI
2 the past 2 months. When her pregnancy test comes back posit ive, she becomes distraught, saying that she
3 has been taking oral contraceptive pills for the past year and has not missed a sing le dose. As she starts to
4 cry, her tears are noted to have an orange t int. The physician tel ls her that the most likely reason her oral
contraceptives were ineffective is an interaction with one of her other medications.
5
6
What is the mechan ism of action of the dr ug that the patient is most likely taking'
7

8 :

9
10 A. Blocks RNA synthesis by inhibiting DNA-dependent RNA po lymerase
• 11
B. Blocks sod ium channels, which prevents the release of the excitatory neurotransmitter glutamate from
• 12 the presynaptic neurons
• 13 C. Cationic and basic proteins bind to the cell membrane and disrupt the osmotic properties
• 14
D. Formation of toxic metabolites with a bactericidal effect
• 15
E. Prolongs action potential phases 1 and 3 via blockage of sodium and potassium channels
• 16
• 17

• 18
• 19
• 20
• 21

6
lock
s
Suspend
0
End Block
Item: llof37 ~. , . M k <:] t> al ~· ~
QIO: 3268 .l. ar Previous Next Lab 'lifllues Notes Calculator

1 •
The correct answer is A. 67°/o chose this.
2
Rifampin suppresses RNA synthesis by inhibit ing DNA-dependent RNA polymerase and is an antibiotic used to
3
treat tube rculosis. One major adve rse effect of rifampin is that it is metabo lized by and also ind uces the
4 cytochrome P-450 isoenzyme system; thus drugs such as oral contraceptives, warfa r in, and ketoconazole may
5 need to be given in higher doses in orde r to be therapeutic. Th is is probably the reason why this woman's oral
contraceptive pills fa iled. Anothe r wel l-known adve rse effect that can be fr ightening to patients is that rifampin
6
turns al l bod ily fluids (tea rs, sweat, and urine) orange.
7 Warfarin Ketoconazole Rifampicin Isozyme Tuberculosis Antibiotics Cytochrome P4SO RNA Transcription (genetics) Urine RNA polymerase

8 Combined oral contraceptive pill Oral contraceptive pill Metabolism Birth control Adverse effect Perspiration Drug metabolism Cytochrome

9
B is not correct. 8°/o chose this.
10
Phenytoin acts by blocking sodium channels, which inhibits glutamate release from excitato ry presynaptic
11 neurons. I t is used to treat epilepsy, particula r ly ton ic-clon ic and partia l seizu res. It has many adverse effects,
• 12 incl uding induction of the cytochrome P-450 isoenzyme system, and thus would interact with oral contraceptive
• 13 pills. However, phenytoin does not cause red-orange bodi ly fluids .
Phenytoin Epilepsy Isozyme Sodium channel blocker Glutamic acid Cytochrome P4SO Sodium channel Generalised tonic-clonic seizure Sodium Neuron
• 14
Chemical synapse Partial seizure Epileptic seizure Oral contraceptive pill Combined oral contraceptive pill Synapse
• 15
• 16 C is not correct. 8°/o chose this .
• 17 Polymyxins 8 and E are cationic basic proteins that act as dete rgents that bind to cell membranes and disrupt
the osmotic and cell membrane integ r ity of the bacteria. They are used in resistant gram-negative infections .
• 18
Adverse effects incl ude neu rotoxicity and acute rena l tubu lar necrosis, but does not cause orange body fluids .
• 19 Cell membrane Gram-negative bacteria Polymyxin Neurotoxicity Osmosis Bacteria Ion Necrosis Protein Detergent Cationic Biological membrane Kidney

• 20
Dis not correct. 11% chose this .
• 21
• Metronidazo le forms toxic metabol ites that have a bactericida l effect. I t is an effective antibiotic aaainst
6
lock
s
Suspend
0
End Block
Item: llof37 ~. , . M k <:] t> al ~· ~
QIO: 3268 .l. ar Previous Next Lab 'lifllues Notes Calculator

1
-
... -- .-- - -
.. . -- - - -
. - -- ... -
Adverse effects include neurotoxicity and acute rena l tubu lar necrosis, but does not cause orange body flu ids.
2 Cell membrane Gram-negative bacteria Polymyxin Neurotoxicity Osmosis Bacteria Ion Necrosis Protein Detergent Cationic Biological membrane Kidney

3
Dis not correct. 11% chose this.
4
Metronidazo le forms toxic metabol ites that have a bactericida l effect. It is an effective antibiotic against
5 amoebae and anaerobes. It is also used in Giardia infection. One of metronidazole's best known adve rse effects
6 is a disu lfira m -like reaction when taken wit h ethano l. Metronidazo le is also highly te ratogenic and shou ld not be
7
taken by pregnant women.
Metronidazole Antibiotics Ethanol Teratology Bactericide Anaerobic organism Amoeba Giardia Metabolite Toxicity
8
9 E is not correct. 6°/o chose this.
10
Amiodarone is an antiarrhyth mic drug that prolongs the action potentia l in cardiac phases 1 and 3 via blockage
of sodium and potassium channels. Amioda rone is infamous for its many adverse effects, includ ing interstitial
11 pul monary fibrosis, t hyroid dysfunction (both hype r- and hypothyroidism), and hepatocel lular necrosis.
• 12 Amiodarone and other antiarrhythmic drugs do not cause orange bodi ly fluids, though it does turn the skin
• 13 bluish with ch ronic use .
Amiodarone Action potential Hypothyroidism Pulmonary fibrosis Antiarrhythmic agent Thyroid Potassium channel Sodium Necrosis Fibrosis Potassium
• 14
Thyroid disease
• 15
• 16
• 17 Bottom Line:
• 18 Rif ampin is used to treat tube rculosis. Adverse effects incl ude causing an orange tint to body fluids. It also
• 19 induces the cytochrome P-450 system, so the drug may lead to subtherapeutic doses of wa rfarin,
• 20
ketoconazole, and ora l contraceptive pi lls .
Warfarin Ketoconazole Rifampicin Tuberculosis Cytochrome P450 Oral contraceptive pill Combined oral contraceptive pill Birth control
• 21

6
lock
s
Suspend
0
End Block
Item: 11 of 3 7 ~ 1 • -<:J I> ~ ~· !:';-~
QIO· 3268 M a rk ·""
· ..L Prev ious Next Lab lues Notes Cal culat o r

1
FA17 p 192.2
2
Rifamycins Rifampin, rifabutin.
3
MECHANISM Inhibit D 'A-dependent R 'A polymerase. Rifampin's -+ R'~:
4
CLINICAL USE Mycobacterium tuberculosis; delay resistance R1 polrmerase inhibitor
5 Ramps up microsomal cytochrome P-450
to dapsone when used for leprosy. Used
6
for meningococcal prophylaxis and Red/orange body fluids
7 chemoprophylaxis in contacts of children\\ ith Rapid resistance if used alone
8 1-/aemophi/us influen::ae I} pe B. Rifampin ramps up cytochrome P--+50, but
rifabutin does not.
9 ADVERSE EfFECTS linor hepatotoxicil} and drug interactions
10 (t cytochrome P--+50); orange body fluids
(nonhazardous side effect). Rifabutin favored
11
0\'er rifampin in patients\\ ith I I IV infection
• 12
due to less cytochrome P-450 stimulation .
• 13
MECHANISM OF RESISTANCE Mutations reduce drug binding to RNA
• 14
polymerase. Monotherapy rapidly leads to
• 15 resistance.
• 16
• 17 FA17p138.1
. 18 Primary and secondary tuberculosis
• 19
PPD ® if current infection or past exposme .
• 20 PPD 8 if no infection and in sarcoidosis or
~Hllar nodes
• 21 Ghon + I ll infection (especially with IO\\ CD-++ cell

a
Lock
s
Suspend
8
End Bl ock
Item: 11 of 3 7 ~ 1 • Ma r k -<:J I> ~ £!1}>'
• !!":-~
QIO: 3268 ..L Prev ious Next Lab lues Notes Calculat o r

& &
1
Primary and secondary tuberculosis
2
PPD ®if current in fection or past exposure.
3
PPD 8 if no infection and in sarcoidosis or
4
Ill infection (especially with lo'' CD4+ cell
5 count).
6 lnterferon-y release assay (IC RA) has fe,,er false
Primary tu~rculosis posili\'eS from BCG \'accinalion.
7
>90% <10% Caseating granulomas fJ with central necrosis
8
Heilbng by librosls Plogressrve pnmary tuberculosis (upper left) and Langhans giant cells (arro\\')
9 CaloficiltiOO wos. mamtnuonl
(tubercwn ~)
I are characteristic of 2° tuberculosis.
l
10
11

• 12
1
ReactiVatiOO
2' tuberculosis
f1brocaseous
ca'lltary leSIOn
I Progrewve
lung disease

J
• 13 (usually upper
to~)
..
_. Miliary
• 14
·~ ~losis
• 15
• 16
1
Loulized dellructive disease Lymph nodes
• 17 Ca'llty
Caseation
caseatoon
. 18
• 19 Adrenal
gland
• 20
"r- Joonts and
• 21 ) long bones
• CJ

a
Lock
s
Suspend
8
End Bl ock
Item: 12 of 3 7 ~ 1 • M k -<:J 1>- Jil ~· !:';-~
QIO: 1701 ..L ar Pre v ious Next Labfli!llues Not es Calcula t o r

IAA]
& &
1
A 57-year-old man presents to the emergency department (ED) with fever, night sweats, and a productive
2
cough with occasional hemoptysis. He is started empirically on several medications for his underlying
3 disease. An X-ray of his chest is shown. At fol low-up several months later, he reports difficulty reading the
4 paper in the morning and has been found to wear unusual color combinations at work.
5
6
7
8
9
10
11
• 12
• 13
• 14
• 15
• 16
• 17
• 18 Image courtesy of CDC

• 19
• 20 Wh ich of the following is the most likely cause of this patient's new symptoms?
• 21 :

a
Lock
s
Suspend
8
End Bl ock
2
3
4
5
6
7
8
9
10
Image courtesy of CDC
11
• 12
Which of the following is the most likely cause of this patient's new symptoms?
• 13
:
• 14
A. Ethambutol toxicity
• 15
• 16
B. I soniazid toxicity

• 17 C. Pyrazinam ide toxicit y


• 18 D. Rifampin toxicity
• 19
E. Tuberculous eye infection
• 20
• 21

a
Lock
s
Suspend
8
End Block
Item: 12 of 37 ~. I • M k <:] t> al ~· ~
QIO: 1701 .l. ar Previous Next lab 'lifllues Notes Calculator

1 •
The correct answer is A. 55°/o chose this.
2
Ethambutol is active aga inst Mycobacterium tuberculosis, and it is one of the medications used to treat
3
tubercu losis (TB) infection . I n an adu lt with a known or high ly suspicious active pulmonary TB, the four
4 medications include ison iazid, rifampin, pyrazinam ide, and ethambuto l. All four medications are given in the
5 intensive phase, wh ich lasts 2 months, and followed by 4 - 7 months in the continuation phase in which patients
6
are t reated with isoniazid and rifampin . During the intensive phase if susceptibility data is ava ilable, ethambuto l
may be discontinued. Ethambuto l's mechanism of action appears to be the inh ibit ion of polymerization of ce ll-
7
wall precursors . Although the drug generally is well to lerated, its most common adverse effects involve ocu lar
8 toxicity such as loss of v isua l acu ity and red- green color blindness, which usua lly appears severa l months after
9 the initiation of t reatment. For children, most literature supports an intensive phase for 2 months with isoniazid,
r ifampicin, pyrazinamide, and ethambutol followed by a continuation phase of at least 4 months with isoniazid
10
and rifampicin .
11 Isoniazid Pyrazinamide Ethambutol Rifampicin Mycobacterium tuberculosis Tuberculosis Mycobacterium Color blindness Red-green color blindness Infection

12 Toxicity

• 13
B is not correct. 18% chose this.
• 14
Although rifampin is considered the best antitubercu lous agent, ison iazid is used for prophylaxis in
• 15 asymptomatic patients with a positive purif ied protein derivative test. A 6- month course of ison iazid prevents
• 16 activation of latent tubercu losis in 90% of patients for at least 20 years . I soniazid blocks myco lic acid ce ll wa ll
• 17 synthesis and is bactericida l for rapidly mu ltip lying organ isms. Major adverse effects include hepatotoxicity and
peripheral neuropathy, but many other adverse effects occu r, such as lupus- like syndrome and optic atrophy .
• 18
I soniazid can also cause sideroblastic anemia through depletion of pyr idoxine. This toxicity can be prevented by
• 19 giving patients 86 supplements .
• 20 Isoniazid Mycolic acid Hepatotoxicity Rifampicin Pyridoxine latent tuberculosis Tuberculosis Sideroblastic anemia Cell wall Peripheral neuropathy Bactericide

• 21 Optic neuropathy Anemia Protein Asymptomatic Mantoux test Preventive healthcare


6
lock
s
Suspend
0
End Block
Item: 12 of 37 ~. I • M k <:] t> al ~· ~
QIO: 1701 .l. ar Previous Next lab 'lifllues Notes Calculator

1 • C is not correct. 10% chose this.


2 Like isoniazid, the spectrum of action of pyrazinamide is limited to Mycobacterium tuberculosis. The site of
3
activity for pyrazinamide is thought to be a fatty acid synthase gene. The major adverse effect of pyrazinamide
therapy is hepatotoxicity, but it is rare at recommended dosages. Another major adverse effect is hyperuricemia
4
and subsequent gout.
5 Isoniazid Pyrazinamide Mycobacterium tuberculosis Fatty acid synthase Hyperuricemia Hepatotoxicity Fatty acid Gene Gout Tuberculosis Mycobacterium

6
D is not correct. 14% chose this.
7
Rifampin is the most potent antituberculous agent avai lable. Rifampin blocks DNA-dependent RNA polymerase,
8 preventing RNA synthesis. Although it is a better agent than isoniazid for preventing active tuberculosis
9 infection, it has a significant risk of liver toxicity that outweighs its benefits as a preventive medicine. Rifampin
is also noted to cause discoloration of bod ily secretions to a red-orange color.
10
Isoniazid Rifampicin Tuberculosis Hepatotoxicity Transcription (genetics) liver Preventive healthcare RNA polymerase RNA Toxicity
11

12
E is not correct. 3°/o chose this.
Mycobacterium tuberculosis can have extrapulmona ry manifestations, but the eye is not commonly involved .
• 13
Miliary tuberculosis infection can affect the eye and cause chorioretinit is, uveitis, and conjunctivit is, but these
• 14 manifestations are rare. Color bl indness wou ld not be associated with such an infection.
• 15 Mycobacterium tuberculosis Miliary tuberculosis Tuberculosis Uveitis Conjunctivitis Chorioretinitis Color blindness Mycobacterium lung Visual impairment Infection

• 16
• 17
Bottom Line:
• 18
Treatment of active tubercu losis is with the RIPE drugs (Rifampin, Isoniazid, Pyrazinamide, and Ethambutol) .
• 19
Ethambutol toxicity man ifests with reduced visua l acu ity and inabi lity to see the color green .
• 20 Ethambutol Tuberculosis Visual acuity Toxicity

• 21

6
lock
s
Suspend
0
End Block
Item: 12 of 3 7 ~ 1 • Ma r k -<:J I> ~ £!1}>'
• !!":-~
QIO: 1701 ..L Pre v ious Next Lab lues Not es Calcula t o r
& &
1
FA17 p 136.1
2
Primary and secondary tuberculosis
3
4
PPD (!) if current infection or past exposm e.
lillarnodes PPD 8 if no infection and in sarcoidosis or

t
5
Ghon t Ill infection (especially " ith lo'' C D-++ cell
6 count).
complex Ghon locus
7 (usually rrodl
lnterferon-y release assay (IC RA) has fewer false
lower lobes)
Primary tuberculom posili,·es from BC C vaccination.
8
>90% <10% Caseating granulomas fJ with central necrosis
9 •
Hea!mg by fibr()ljs

Progresswe primary tuberculosis (upper left) and Langhans giant cells (arrow)
10 (AIDS, malnutniJOil)
CalohcatiOO
(tubercoon ®I
I zo
arc cha racteri~tic of tuberculosis.
l
1
l
11
ReactiVation Progresllve

..,,
12 lung dill!ase
2' tub6culosls
• 13 Fibrocaseous
' J
• 14 cavttary le!j()n

• 15
(usuallyupper
lobes)
..
__. Miliary

• 16
·~ ~losis
• 17 1
Loulized destructive disease
. 18 Cavtty
Caseatioo Lungs --~
• 19 caseaoon
Scar Liver ~:.~·
• 20 Adrenal
gland
• 21

a
Lock
s
Suspend
8
End Bl ock
Item: 12 of 3 7 ~ 1 • M k -<:J 1>- Jil ~· !:';-~
QIO: 1701 ..L ar Pre v ious Next Labfli!llues Not es Calcula t o r

1
&
Healong by fibrOSis Progressive primary tuberculosis (11ppcr left) and Langhans giant cells (arrow) &

(AIDS. malnutrrtion)
2
Calofication
(tuberculm (£))
I arc characteristic of zotuberculosis.
l
3
4
5
Fibrocaseous
Reactivatoon

2' tuberculosis

Col'fltary le51011
I
Bacteremia
_j
ProgresSIVe
lung diSI!ase

6
(uSIIillly upper
lobes) 7
.. Miliary
7 ,~ ~losis
8
9 LoQUzed destructift disease
Ca'<lty
10 Caseation
CaSI!atoon
11

12
• 13 "r- Jooms and
> long bones
• 14 g

• 15
• 16 FA17 p 193.3

• 17
Ethambutol

. 18 MECHANISM ! carbohydrate polymcri~a t ion of mycobacterium cell wall by blocking arabinosyltransfcrasc.


• 19 (LINI(AL USE Mrcobacterium tuberculosis.
• 20 ADVERSE EFFECTS Optic neuropathy (red-green color blindness). Pronounce "eyethambutol."
• 21

a
Lock
s
Suspend
8
End Bl ock
Item: 13 of 3 7 ~ 1 • M k -<:J 1>- Jil ~· !:';-~
QIO: 3449 ..L ar Pre v ious Next Lab fli!ltues Not es Calcula t o r

IAA]
& &
1
A 36-year-old African-American woman presents to the clinic with chest pain, cough ing, and shortness of
2
breath . She has no sign ificant medical history except for asthma as a chi ld. She denies any history of fevers,
3 rashes, or recent travel. On physica l examination, the lungs are clear to auscu ltation . A plain film X-ray is
4 shown in the image. PPD is negative. Her serum angiotensin-converting enzyme level is elevated. Her doctor
initiates appropriate therapy .
5
6
7
8
9
10
11
12
• 13
• 14
• 15
• 16
• 17
• 18
• 19
• 20
• 21 If given at too high a dose, what is most likely adverse effect of this treatment?

a
Lock
s
Suspend
8
End Bl ock
8
9
10
11
12
I f given at too high a dose/ what is most likely adverse effect of th is treatment?
• 13
:
• 14
A . Hemolysis/ if patient is glucose-6-phosphate dehydrogenase deficient
• 15
• 16
B. Ototoxicity

• 17 C. Pseudomembranous col itis


• 18 D . Reddish-brown discoloration of urine
• 19
E. Weight gain located in the face 1 neck/ and upper back
• 20
• 21

a
Lock
s
Suspend
8
End Block
Item: 13 of 3 7 ~ 1 • M k -<:J 1>- Jil ~· !:';-~
QIO: 3449 ..L ar Prev ious Next Lab fli!ltues Not es Calculat o r

& &
1
The correct answer is E. 53°/o chose this.
2
This patient is suffering from sarcoidosis, a disease characterized
3
histolog ical ly by noncaseating granulomas. Lung involvement is
4 very common in these patients and is often diagnosed with X-ray
5 of the chest. Commonly seen are bilateral hilar lymphadenopathy
6
and interstitial infiltrates (shown in the vignette X-ray). Treatment
for sarcoid is a corticosteroid taper. Overdose, or even a
7
therapeutic dose, of corticosteroids can lead to iatrogenic Cushing
8 syndrome, commonly characterized by moon facies (image a) and
9 a buffalo hump, among other symptoms similar to those shown in
these images (image b, striae).
10
Corticosteroid Sarcoidosis Iatrogenesis Cushing's syndrome Lymphadenopathy Chest radiograph
11
Granuloma Histology Lung Root of the lung Hilar lymphadenopathy X-ray
12 Images copyright © 2012 Ce/ik eta/.;
licensee BioMed Central Ltd.
13
• 14
A is not correct. 12°/o chose this .
• 15
This is an adverse effect of isoniazid, a drug used in the t reatment of tubercu losis (TB). PPD stands for purified
• 16 prot ein derivative and is the standard test for TB .
Isoniazid Tuberculosis Mantoux test Protein Adverse effect
• 17
• 18 B is not correct. 14°/o chose this .
• 19 This is an adverse effect of several drugs, including aminog lycosides and vancomycin .
Vancomycin Aminoglycoside Adverse effect
• 20
• 21 C is not correct. 9°/o chose this.

a
Lock
s
Suspend
8
End Block
Item: 13 of 37 ~. I • M k <:] t> al ~· ~
QIO: 3449 .l. ar Previous Next lab 'lifllues Notes Calculator

1 licensee BioMed Central Ltd.


2
A is not correct. 12% chose this.
3
This is an adverse effect of isoniazid, a drug used in the treatment of tuberculosis (TB). PPD stands for purified
4
protein derivative and is the standa rd test for TB.
5 Isoniazid Tuberculosis Mantoux test Protein Adverse effect

6
B is not correct. 14% chose this.
7
This is an adverse effect of several dr ugs, including aminog lycosides and vancomycin.
8 Vancomycin Aminoglycoside Adverse effect

9
C is not correct. 9°/o chose this.
10
This is t raditionally an adverse effect of clindamycin and ampici llin/amoxici llin, drugs used to treat anaerobic
11 infections and certain gram-positive bacteria and gram-negative rods, respectively. However, it has now been
12 described with nearly al l antibiotics.
Clindamycin Gram-negative bacteria Gram-positive bacteria Anaerobic organism Antibiotics Bacteria Adverse effect Anaerobic infection
13
• 14 D is not correct. 12% chose this .
• 15 This is an adverse effect of rifampin, a dr ug used in the t reatment of tubercu losis .
Rifampicin Tuberculosis Adverse effect
• 16
• 17

• 18 Bottom Line:
• 19
Sarcoidosis is treated with corticosteroids, which can cause Cushing syndrome if dosage is not well calibrated.
• 20 Sarcoidosis Cushing' s syndrome Corticosteroid

• 21

6
lock
s
Suspend
0
End Block
Item: 13 of 3 7 ~ 1 • M k -<:J 1>- Jil ~· !:';-~
QIO: 3449 ..L ar Pre v ious Next Lab fli!ltues Not es Calcula t o r
& &
1

2 FA17 p 444.1
3 Sarcoidosis Characterized by immune-mediated, widespread noncaseating granulomas r.J, elev-dted serum
4 ACE levels, and elevated CD-f+/CD8+ ratio in bronchoalveolar lavage fluid. More common in
5 frican-American females. Often asymptomatic except for en larged lrmph nodes. Findings on
6 CXR of bilateral adenopathy and coarse reticular opacities III; CT of the chest better demonsI rates
the extensi,·e hilar and mediastinal adenopathy
7
Associated with restrictive lung disease (interstitial fibrosis), erythema nodosum, lupus pernio (skin
8
lesions on face resembling lupus), Bell pals), epithelioid granulomas containing microscopic
9 Schaumann and asteroid bodies, uveitis, hrpercalcemia (due to t la-hyd ro:~:ylase-mediated
10 vitamin D acti,·ation in macrophages).
11 Treatment: steroids (if symptomatic).
12
13
• 14
• 15
• 16
• 17
. 18
• 19
• 20
• 21

a
Lock
s
Suspend
8
End Bl ock
Item: 13 of 3 7 ~ 1 • M k -<:J 1>- Jil ~· !:';-~
QIO: 3449 ..L ar Pre v ious Next Lab fli!ltues Not es Calcula t o r
& &
1
FA17 p 323.1
2
Cushing syndrome
3
EIIOLOGY f cortisol due to a variety of causes:
4
Exogenous corticosteroids-resu lt in l CTII , bilateral adrenal atrophy. Most common cause.
5 • Primary adrenal adenoma, h} perplasia, or carcinoma-result in l ACTH, atrophy of
6 uninvolved adrenal gland. Can also present" ith pscudoh) peraldosteronism.
7 • ACTH-secreting pituitary adenoma (Cushing disease); paraneoplastic ACTH secretion (eg,
8
small cell lung cancer, bronchial carcinoids)-result in f ACTH, bilateral adrenal hyperplasia.
Cushing disease is responsible for the majorit) of endogenous cases of Cushing syndrome.
9
FINDINGS Hypertension, weight gain, moon facies , abdominal striae [l) and truncal obesity, buffalo hump,
10
skin changes (eg, thinning, striae), osteoporosis, h} perglycemia (insulin resistance), <lmenorrhca,
11
immunosuppression.
12
DIAGNOSIS Screening tests include: f free cortisol on 24.h r urinalysis, f midnight salivary cortisol, and no
13 suppression with overnight low-dose dexamethasone test. Leasure serum ACT II. If l, suspect
• 14 adrenal tumor or exogenous glucocorticoids. If t, distinguish between Cush ing disease and
• 15 ectopic ACTII secretion (cg, from small cell lung cancer) wit h a high-dose dexamct·hasonc
• 16 suppression test and C RI I stimulation test. Ectopic secretion will not decrease with
dexamethasone because the source is resistant to negative feedback; ectopic secretion wi ll not
• 17
increase with CRH because pituitary ACTII is suppressed .
. 18
• 19 i 24·hr unne rree cortisol. f late noghl sa~vary C()(lasol, and/or
anadequale suppresSIOn on I mg overn.ghl dexamethasone test
• 20
• 21

a
Lock
s
Suspend
8
End Bl ock
Item: 13 of 3 7 ~ 1 • M k -<:J 1>- Jil ~· !:';-~
QIO: 3449 ..L ar Pre v ious Next Lab fli!ltues Not es Calcula t o r
& &
1 increase with C RI-l because pituitary ACT II is suppressed.
2
3
i 24-lv unne rree cortisol. f late noght sa~vary CortiSOl and/or
onadequate suppresSIOn on I mg overnoght dexamethasone test
4
5
~easureACTH
6
7
(
Suppressed
l
Elevated
8
9 ACTH -dependent
Cushong syndrome
10
11 r
12 Exogenoos glucocorticoids
H1gh-dose dexamethasone CRH Stimulation t~t
or adrenal tumor
(consoder adrenal CT to confirm) suppresSion test
13
• 14 \
• 15 No suppress1on Adequate i ACTHand cortisol No i inACTH
• 16 Ectopic ACTH suppressoon Cushing disease and cortisol
secretion Cushing disease Ectopic ACTH
• 17 secretion
• 18
• 19 a of the ch~tlabdomen/pelviS MRI or the potuitary CT or the chest/abdomen/pelves
• 20 13
• 21

a
Lock
s
Suspend
8
End Bl ock
Item: 14 of 37 ~. I • M k <:] t> al ~· ~
QIO: 3421 .l. ar Previous Next lab 'lifllues Notes Calculator

1 •
A 54-year-old woman compla ins about a persistent cough she has had for the past 3 months. The cough has
2
been bothering her a lot and making her anxious . She th inks the anxiety is why she has lost some weight
3 recently. She also blames the anxiety whenever she wakes up in the middle of the night and finds herself
4 drenched in sweat. Further history reveals she has rheumatoid arth ritis (RA), but her joint pa ins and swell ings are
wel l control led by medications her r heumatolog ist has prescribed her. Following a physica l examination, the
5
physician orders an x- ray of the chest, which is shown in the image. Based on the resu lts, the physician
6 immediately prescribes an antibiotic reg imen and asks her to discontinue one of the dru gs used to t reat her RA.
7

8
9
10
11

12
13
• 14
• 15
• 16
• 17

• 18
• 19
• 20
• 21 Image courtesy of Radiopaedia/Or. Frank Gaillard

6
lock
s
Suspend
0
End Block
Item: 14 of 37 ~. I • M k <:] t> al ~· ~ ~
QIO: 3421 .l. ar Previous Next lab 'lifllues Notes Calculator

1 •
A 54-year-old woman compla ins about a persistent cough she has had for the past 3 months. The cough has
2
been bothering her a lot and making her anxious . She th inks the anxiety is why she has lost some weight
3 recently. She also blames the anxiety whenever she wakes up in the middle of the night and finds herself
4 drenched in sweat. Further history reveals she has rheumatoid arth ritis (RA), but her joint pa ins and swell ings are
wel l control led by medications her r heumatolog ist has prescribed her. Following a physica l examination, the
5
physician orders an x- ray of the chest, wh ich is shown in the image. Based on the resu lts, the physician
6 immediately prescribes an antibiotic reg imen and asks her to discontinue one of the drugs used to t reat her RA.
7

8
9
10
11

12
13
• 14
• 15
• 16
• 17

• 18
• 19
• 20
• 21

6
lock
s
Suspend
0
End Block
Image courtesy of Radiopaedia/Dr. Frank Gaillard
11
12
Which of the fol lowing drugs increased her risk of developi ng the disease shown on the radiograph?
13
:
• 14
A. Etanercept
• 15
• 16
B. Methotrexate

• 17 C. Nonsteroidal anti-inflammatory drugs


• 18 D. Risedronate
• 19
E. Su lfasalazine
• 20
• 21

a
Lock
s
Suspend
8
End Block
Item: 14 of 37 ~. I • M k <:] t> al ~· ~
QIO: 3421 .l. ar Previous Next lab 'lifllues Notes Calculator

1 •

2
The correct answer is A. 43°/o chose this.
Th is pat ient is likely suffering from a tubercu losis infection that was react ivat ed by her use of etanercept. The x -
3
ray of the chest shows a dense cavitary apica l lung lesion that is high ly indicative of a reactivat ed t uberculosis
4 infection . Etanercept is a fusion protein t hat cont ains two ident ical tumor necrosis fact or (TN F)- receptor
5 monomers fused to a human IgG Fe doma in. Therefore, it act s as a TNF antagon ist. In t uberculosis infections,
6 TN F (secreted by activated macrophages) recruits monocyt es t o form the epit helioid granu lomas required to
cont ain the mycobacteria . When TNF is effectively removed from the infection sit e (by drugs or ot her forms of
7
immunosuppression), pat ients face an increased risk of react ivat ion with caseation and cavitary lesions .
8 TNF inhibitor Etanercept Immunosuppression Mycobacterium Tuberculosis Immunoglobulin G Macrophage Tumor necrosis factor alpha Granuloma Fusion protein

9 Epithelium Protein Monocyte Tumor necrosis factors lesion Necrosis Neoplasm lung Fragment crystallizable region X-ray Receptor antagonist Infection

10
B is not correct. 32% chose this .
11
Methotrexate is an anti-i nfl ammatory agent used in the t reatment of RA. It inh ibits dihydrofolate reductase and
12 blocks thymine synt hesis. It is not an inh ibitor of tumor necrosis fact or. The dosage of methot rexate used in t he
13 t reatment of RA does not induce myelosuppression, although higher doses can produce this complication .
Methotrexate Bone marrow suppression Dihydrofolate reductase Thymine Dihydrofolic acid Anti-inflammatory Necrosis Neoplasm Tumor necrosis factors
14
Enzyme inhibitor Tumor necrosis factor alpha
• 15
• 16 C is not correct. 11% chose this .
• 17 Nonst eroidal anti-inflammatory drugs (NSAIDs) do not impair immun ity, although they do impair plat elet
• 18
function . These drugs can help decrease inflammation, but do not slow the progression of RA. I t is qu ite un likely
that a patient with RA who complains of well-controlled joint pain wou ld be re lying on NSAIDs alone .
• 19 Anti-inflammatory Platelet Nonsteroidal anti-inflammatory drug Arthralgia Immunity (medical) Inflammation
• 20
D is not correct. 7°/o chose this .
• 21

6
lock
s
Suspend
0
End Block
Item: 14 of 37 ~. I • M k <:] t> al ~· ~
QIO: 3421 .l. ar Previous Next lab 'lifllues Notes Calculator

1 • t reatment of RA does not induce myelosuppression, although higher doses can produce this complication .
Methotrexate Bone marrow suppression Dihydrofolate reductase Thymine Dihydrofolic acid Anti -inflammatory Necrosis Neoplasm Tumor necrosis factors
2
Enzyme inhibitor Tumor necrosis factor alpha
3
4 C is not correct. 11% chose this.
5 Nonst eroidal anti-inflammatory drugs (NSAIDs) do not impair immun ity, although they do impair plat elet
6
function . These drugs can help decrease inflammation, but do not slow the progression of RA. I t is qu ite un likely
that a patient with RA who complains of well-controlled joint pain wou ld be relying on NSAIDs alone.
7 Anti-inflammatory Platelet Nonsteroidal anti-inflammatory drug Arthralgia Immunity (medical) Inflammation
8
D is not correct. 7°/o c hose this.
9
Rised ronate is a bisphosphonate used in t he prevention and treat ment of osteoporosis. Bisphosphonat es are not
10
used in t he t reatment of RA, and do not have immunosuppressive effect s.
11 Bisphosphonate Risedronic acid Osteoporosis Immunosuppression

12
E is not correct. 7°/o c hose this.
13 Although t he exact mechanism of act ion of su lfasa lazine is not known, it is be lieved t o suppress the activit y of
14 natu ral killer cells and impai r lymphocyt e transformation, which would not directly allow mycobact eria to
• 15 overcome immune surveillance and reactivat e .
Sulfasalazine lymphocyte Natural killer cell Mycobacterium Mechanism of action Immune system
• 16
• 17

• 18 Bottom Line:
• 19 Etanercept can exacerbate t uberculosis by antagon izing tumor necrosis factor.
• 20 Etanercept Tuberculosis Tumor necrosis factors Tumor necrosis factor alpha Necrosis Neoplasm

• 21

6
lock
s
Suspend
0
End Block
Item: 14 of 3 7 ~ 1 • Ma rk -<:J I> ~ £!1}>'
• !!":-~
QIO: 3421 ..L Pre v ious Next Lab lues Not es Calcula t o r
& &
1
FA17 p 136.1
2

3
Primary and secondary tuberculosis
4 PPD ® if current infection or past exposme.
PPD 8 if no infection and in sarcoidosis or
5
1 lillarnodes
Ill infection (especially'' ith lo" CD-++ cell
6 Ghon j . +
com!*x Ghon locus count).
7 (usually rOO/
lnterferon-y release assay (ICR ) has fewer fal se
lower lobes)
8 Primary tuberculosis posili,·es from BCC vaccination.
9 >90X <10% Caseating granulomas rJ with central necrosis
10
HeM9 by librOils Progressr;e primary tuberculosis (upper left) and Langhans giant cells (arrow)
Calotic.lt100 IAJDS. matnutntJOnl
ltubercwn ®l
I arc characteri ~tic of 2° tuberculosis.
l
l
11

12 1 ReactNation ProgreSS<ve
lung disease
2' tubvculosls
13
Fibrocaseous
'
..\ , Bacteremia
~~~Av· ._ _______ _j
14 CaVItary le!j()O

• 15
(usually upper
lobes) 7
.. Miliary ~Meninges

• 16 ·~ ~losis
/""' Vertebrae
• 17 1
Loulized d"tructive disease lymph nodes
f { ......- I~ disease)

. 18 •• •
CaVIty •
• 19 Caseation t:O '-
Lungs _ _.,.
caseaoon
• 20 Liver ~:·'· Adrenal
• 21 gland

a
lock
s
Suspend
8
End Block
Item: 14 of 3 7 ~ 1 • M k -<:J 1>- Jil ~· !:';-~
QIO: 3421 ..L ar Pre v ious Next Lab fli!ltues Not es Calcula t o r
& &
1

2 FA17 p 136.2
3 Mycobacteria 1\Iycobacterium tuberculosis (TB, often resistant TB symptoms include fe,·er, night sweats,
4 to multiple drugs). weight loss, cough (nonproductive or
5 1\I avium-intracellulare (causes disseminated, producti,·e), hemoptysis.
6 non-TB disease in J\IDS; often resistant to Cord factor creates a "serpentine cord"
mulliple drugs). Proph) laxis \\ ith azithrom)cin appearance in virulent J\I tuberculosis
7
when CD4+ count< 50 cells/mm 3. strains; activates macrophages (promoting
8
J\1 scro{ulaceum (cen icallymphadenitis in granuloma fom1ation) and induces release of
9 children). TN F-a. Sulfatides (surface glycolipids) inhibit
10 l\I marinum (hand infection in aquarium phagolysosomal fusion.
11 handlers).
All mycobacteria are acid.fast organisms (pink
12
rods; arrows in r.J).
13
14
FA17 p 457.3
• 15
• 16 TNF-c:x inhibitors All T t F-a inhibitors predispose to infect ion, including reactivation oflatentTB, since T IF is
important in granuloma formation and ~tabi l ization .
• 17
DRUG MECHANISM CLINICALUSE
. 18
Etane rcept Fusion protein (receptor forT 1F-a + lgC 1 Fe), Rheumatoid arthritis, psoriasis, ankylosing
• 19
produced by recombinant Dt A. spondylitis
• 20 Etanercept intercepts T;\ F.
• 21 lnfliximab, Anti-TNF-a monoclonal antibod~·. lnAammatorr bo\\el disease, rheumatoid arthritis,

a
Lock
s
Suspend
8
End Bl ock
Item: 14 of 37 ~ .I • M k <:] t> al ~· ~
QIO: 3421 .l. ar Previous Next lab 'lifllues Notes Calculator

1 M avium-intracellulare (causes disseminated, productive), hemoptysis.


2 non-TB disease in AIDS; often resistant to Cord factor creates a "serpentine cord"
3
multiple drugs). Prophylaxis with azithromycin appearance in virulent M tuberculosis
when CD4+ count< 50 cells/mm 3. strains; activates macrophages (promoting
4
M scrofulaceum (cervical lymphadenitis in granuloma formation) and induces release of
5 children). T F-a. Sulfatides (surface glycolipids) inhibit
6 M marinwn (hand infection in aquarium phagolysosomal fusion.
7 handlers}.
8 All mycobacteria are acid-fast organisms (pink
rods; arrows in rl1).
9
10
11 FA17 p457.3

12 TN F-a inhibitors All T t F-a inhibitors predispose to infection, including reactivation of latent TB, since T IF is
13 important in granuloma formation and stabilization.
DRUG MECHANISM CLINICAL USE
14
Etanercept F'usion protein (receptor for T t F'-a + lgC1 F'c), Rheumatoid arthritis, psoriasis, ankylosing
• 15
produced by recombinant DNA. spondylitis
• 16
Etanerccpt intercepts '1'1\ F.
• 17
lnfliximab, Anti-T F-a monoclonal antibody. InAammatory bowel disease, rheumatoid arthritis,
• 18 adalimumab, ankylosing spondylitis, psoriasis
• 19 certolizumab,
• 20 golimumab
• 21

6
lock
s
Suspend
0
End Block
Item: 15 of 37 ~. I • M k <:] t> al ~· ~
QIO: 1335 .l. ar Previous Next lab 'lifllues Notes Calculator


1
2
A 55-year-old man with a significant smoking history complains of dyspnea and productive cough for 1 week. I"' Aj
Physical examination revea ls diffuse wheezes, a prolonged expiratory phase, hyperresonance to percussion, A
3 and a bar rel chest. The patient is afebrile and has a heart rate of 105/ min . The physician prescibes a
4 common ly used med ication in hopes that it will alleviate this patient's symptoms. Due to uncontrolled dyspnea, the
patient uses th is new med ication sign ificantly more frequently than directed .
5
6
One week later, the patient returns to the office with recent onset of tremors and feels as though his heart is
7
racing . After counse ling the patient, what additiona l tests should be ordered?
8
:
9
A. Basic metabol ic pane l
10
B. Blood cu ltu res
11

12 C. Chest X-ray
13 D. Complete blood count
14
E. Urine tox screen
0 15
0
16
0 17
0
18
0 19
0 20
0
21

6
lock
s
Suspend
0
End Block
Item: 15 of 37 ~. I • M k <:] t> al ~· ~
QIO: 1335 .l. ar Previous Next lab 'lifllues Notes Calculator

1 •

2 The correct answer is A. 54°/o chose this.


3 Salmeterol and albuterol act selectively at ~radre n e rgic receptors, causing bronchial smooth muscle relaxation .
Catecho lamines in genera l promote potassium entry into cel ls by increasing the activit ies of the Na-K-ATPase
4
pump, and the Na-K- 2CI (NKCCl) cotransporter. These ~-agon i sts have been proven in randomized, controlled
5 trials and meta -analyses to improve symptoms and lung function . They are typica lly prescribed on an as-needed
6 basis rather than regu larly scheduled, to decrease exposure in patients with less severe symptoms. Risks of ~­
7 agonist overuse include t remor, tachycardia, and hypokalemia. As th is patient is like ly presenting with overdose
of his albuterol inha ler, it is possible that he has developed hypokalemia, which can be very dangerous. The
8
easiest way to measure ser um potassium is th rough a basic metabolic panel, wh ich quantifies serum sodium,
9 potassium, ch loride, bicarbonate, creatinine, BUN, and glucose.
Na+/K+-ATPase Salmeterol Hypokalemia Creatinine Tachycardia Bicarbonate Smooth muscle tissue Potassium Catecholamine Salbutamol Tremor Sodium
10
11 Bronchus Blood plasma Blood urea nitrogen Glucose Na-K-CI cotransporter Meta-analysis Cotransporter Metabolism Chloride lung Receptor (biochemistry) Muscle

12 Inhaler Serum (blood)

13 B is not correct. 7°/o chose this.


14 Blood cu ltu res are useful for detecting infections. This is highly un likely in this patient given his presentation and
15 recent initiation on a new med ication, which is likely a ~-agonist.
Blood culture
• 16
• 17 C is not correct. 14% chose this .
• 18 Patients with pre-existing pu lmonary conditions are at a heightened risk of developing infections, such as
• 19
pneumon ia. Nonetheless, th is patient is not presenting with any symptoms of infection or pneumonia. Although
chest X-rays are useful in mon itoring the status of chronic lung cond it ions such as chronic obstructive
• 20 pulmona ry disease (COPD), it is highly unl ikely that this patient's COPD has changed sign if icantly since his office
• 21 visit 1 week ago .

6
lock
s
Suspend
0
End Block
Item: 15 of 37 ~. I • M k <:] t> al ~· ~
QIO: 1335

1
.l.
- p
ar Previous
gp y
Next

.
lab 'lifllues
g
Notes Calculator
p g . I .
pneumon ia. Nonetheless, th is patient is not presenting with any symptoms of infection or pneumonia. Although
2
chest X-rays are useful in mon itoring the status of chronic lung cond it ions such as chronic obstructive
3 pulmona ry disease (COPD), it is highly unl ikely that this patient's COPD has changed sign if icantly since his office
4 visit 1 week ago .
Chronic obstructive pulmonary disease Pneumonia X-ray lung Chest radiograph Pulmonology
5
6 Dis not correct. 11% chose this.
7 Complete blood counts are a usefu l tests for infection. Although this patient is at a heightened r isk of infection,
such as pneumonia, given his chronic lung condition (chronic obstr uctive pu lmonary disease), he is not
8
exh ibiting any symptoms suggestive of infection. It is far more likely that he is overusing his newly prescribed
9
~-agon i st .
10 Chronic obstructive pulmonary disease Pneumonia lung Complete blood count Pulmonology Infection

11
E is not correct. 14% chose this.
12 Although intoxication with a va r iety of drugs can lead to tremors and tachycardia, it is much more likely that
13 this patient's symptoms are due to his overuse of a ~-agon i st, which was recently prescribed to reduce his
14 chronic obstr uctive pu lmonary disease exacerbations.
Chronic obstructive pulmonary disease Tachycardia Alcohol intoxication Respiratory disease Substance intoxication
15
• 16
• 17 Bottom Line:
• 18 Bronchodi lators are the therapeutic mainstay for patients with chronic obstructive pulmonary disease (COPD) .
• 19 ~-Agonists include albuterol and levalbuterol, which have been proven to improve symptoms and lung function
in patients with COPD. Risks of ~-agon i st over use include tremor, reflex tachyca rdia, and hypoka lemia .
• 20
Hypokalemia Chronic obstructive pulmonary disease Tremor Tachycardia Bronchodilator Salbutamol Respiratory disease lung Reflex
• 21

6
lock
s
Suspend
0
End Block
Item: 15 of 3 7 ~ 1 • M k -<:J 1>- Jil ~· !:';-~
QIO: 1335 ..L ar Pre v ious Next Lab fli!ltues Not es Calcula t o r
& &
1

2 FA17 p649.1
3 Asthma drugs Bronchoconstriction is mediated b~ (I) inOammatory processes and (2) parasympathetic tone;
4 therapy is directed at these 2 pathways.
5 ~-ag onists Albute rol-relaxcs bronchial smooth muscle (short acting ~-agonist). Used during acu te
6 exacerbation.
7 Salmete rol, fo rmote ro l- long-acting agents for prophylaxis. Adverse effects are tremor and
8 arrh' thmia.
9 Inhaled Fluticasone, budesonide -inhibit thc synthesis of \'irtually all cytokincs. Inacti,·atc NF-KB, the
10
corticoste roids transcription factor that induces production ofT 'F-a and other inflammatory agents. 1st-line
therapy for chronic asthma. Vlay cause oral thrush.
11
Muscarinic Tiotro pium, ipratro pium-competitivel) bloc!. muscarinic receptors, preventing
12
a ntago nists bronchoconstriction. Also used for COPD. Tiotropium is long acting.
13
Antile ukotrienes Monte lukast, zafirlukast -bloc!. leu l-ot ricnc Exposure to antigen
14
recep tors (CysLTl). Especially good for (dust pollen. etc)
15 aspiri n-induced and exercise-induced :~sthma.
• 16 Zileuto n -5-lipoxygcnasc pathway inhibitor. t-0- Avoidance
• 17 Blocks conversion of arachidonic acid to
. 18
leukotrienes. Hepatotoxic.
Anti-lgE monoclo nal Omalizumab-binds mostly unbound serum
Antigen and tgE t-0- Omalizumab
• 19 on mast cells
the ra py lgE and blocks binding to Fc£RI. Used in
• 20
allergic asthma with t lgE b ·cls resistant to
• 21 .._..o... Steroids
• inhaled steroids and long-acting B.-agonists.

a
Lock
s
Suspend
8
End Bl ock
Item: 15 of 37 ~. I • M k <:] t> al ~· ~
QIO: 1335 .l. ar Previous Next lab 'lifllues Notes Calculator

1 • •
FA17 p638.1
2
Obstructive lung Obstruction of air fl ow - air trapping in lungs. Airways close prematurely at high lung volumes
3
diseases - t RV and t FRC, t TLC. PFTs: U FEV 1, l FVC - l FEV 1/FVC ratio (hallmark),
4
V/Q mismatch. C hronic, hypoxic pulmonary vasoconstriction can lead to cor pulmonale. Chronic
5 obstructive pulmonary disease (COPD) includes chron ic bronchitis and emphysema.
6 TYPE PRESENTATION PATHOLOGY OTHER
7 Chronic bronchitis Findings: \\·heezing, crackles, Hypertrophy and hyperplasia Diagnostic criteria: productive
8 ("blue bloater") cyanosis (hypoxemia due of mucus-secreting glands cough for> 3 months in a
9 to shunting), dyspnea, C02 in bronchi - Reid index year for> 2 consecutive years.
retention, zopolyc)themia. (thickness of mucosal gland
10
layer to thickness of wall
11 between epithelium and
12 cartilage) > 50%.
13 Emphysema ("pink Centriacinar-associated with Enlargement of air spaces CXR: t AP diameter, flattened
14 puffer") smoking rJ (£). Frequently in l recoil, t compliance, diaphragm, t lung field
15 upper lobes (smoke rises up). l DLCO from destruction of lucency.
Panacinar-associated with alveolar walls (arrow in m). Barrel-shaped chest 1!].
• 16
a 1-antitrypsin deficiency. t elastase activity - t loss Exhalation through pursed lips
• 17
Frequently in lower lobes. of elastic fibers - t lung to increase airway pressure
• 18 compliance. and prevent airway collapse.
• 19 Asthma Findings: cough, wheezing, Branchial hypcrrcsponsi\·cncss Aspirin-i nduced asthma: COX
0 20 tachypnea, dyspnea, - reversible inhibition - leukotriene
0 21 hypoxemia, l inspiratory/ bronchoconstriction. Smooth overproduction - airway •

6
lock
s
Suspend
0
End Block
Item: 15 of 37 ~. I • M k <:] t> al ~· ~
QIO: 1335 .l. ar Previous Next lab 'lifllues Notes Calculator

1 • •
FA17p234.1
2
Sympathomimetics
3 DRUG ACTION APPLICATIONS
4
Direct sympathomimetics
5
Albuterol, salmeterol ~z> ~~ Albuterol for acute asthma or COPO. Salmeterol
6 for long-term asthma or COPO control.
7
Dobutamine ~~>~'a Heart failure (llF) (inotropic > chronotropic),
8 cardiac stress testing.
9 Dopamine 0 1= 02 > ~ > a Unstable bradycardia, HF, shock; inotropic and
10 chronotropic effects at lower doses due to ~
11 effects; vasoconstriction at high doses due to a
12
effects.

13
Epinephrine ~>a Anaphylaxis, asthma, open-angle glaucoma;
a effects predominate at high doses.
14
Significantly stronger effect at ~-receptor than
15 norepinephrine .
• 16
Fenoldopam 01 Postoperative hypertension, hypertensive crisis.
• 17 Vasodilator (coronary, periphera l, renal, and
• 18 splanch nic). Promotes natriuresis. Can cause
• 19 hypotension and tachycardia .
• 20 Isoproterenol ~I= ~ Electrophysiologic evaluation of
• 21 tachyarrhythmias. Can worsen ischemia .
• •

6
lock
s
Suspend
0
End Block
Item: 16 of 3 7 ~ 1 • M k -<:J 1>- Jil ~· !:';-~
QIO: 1927 ..L ar Pre v ious Next Labfli!llues Notes Calcula t o r

IAA]
& &
1
A 55-year-old man with a 30-pack-year smoking history presents to his physician because of a 3-month
2
history of productive cough . He is diagnosed with ch ron ic obstructive pulmonary disease after x-ray of the
3 chest demonstrates hyperinflated lungs and a f lattened diaphragms. The physician prescribes inhaled
4 steroids, a 13r agonist, and ipratropium bromide.
5
6 Ipratropium bromide will produce bronchodilation through which of the following mechanisms?
7 :

8 A. Blockade of acetylcholine at muscarinic receptors


9 B. Inhibition of phosphodiesterase resulting in increased cAMP levels
10
C. Inhibition of the degranu lation of mast cells
11
D. Inhibition of the synthesis of cytokines
12
13 E. Stimu lation of adenylyl cyclase resu lting in increased cAMP levels
14
15
• 16
• 17
• 18
• 19
• 20
• 21

a
Lock
s
Suspend
8
End Bl ock
Item: 16 of 37 ~. I • M k <:] t> al ~· ~
QIO: 1927 .l. ar Previous Next Lab 'lifllues Notes Calculator

1 •

2 The correct answer is A. 70°/o chose this.


Ipratropium bromide is a muscarinic antagonist used to treat chronic obstructive pu lmonary disease and
3
asthma. It competitively blocks muscarinic receptors, preventing acetylcho line-mediated bronchoconstriction. It
4 is administered directly to the airway and is minima lly absor bed, leading to few adverse events. At high doses,
5 however, atropine-like toxicity may occur.
Ipratropium bromide Chronic obstructive pulmonary disease Muscarinic antagonist Bronchoconstriction Asthma Receptor antagonist
6
7 Muscarinic acetylcholine receptor Respiratory tract Respiratory disease Receptor (biochemistry)

8 B is not correct. 10% chose this.


9 Methylxanthines such as theophylline act by inh ibiting phosphodiesterase, resulting in increased cAMP levels.
Theophylline Xanthine Cyclic adenosine monophosphate Phosphodiesterase
10
11 C is not correct. 6°/o chose this.
12 Cromolyn acts by inh ibiting the degranu lation of mast cel ls.
Cromoglicic acid Degranulation Mast cell
13
14 D is not correct. 4°/o chose this.
15 Corticosteroids such as beclomethasone and predn isone act by inhibiting the synthesis of cytokines.
Prednisone Beclometasone dipropionate Corticosteroid Cytokine
16
• 17 E is not correct. 10% chose this .
• 18 ~-Agonists such as albuterol and sa lmeterol act by stimu lating adenyl cyclase, resu lting in increased cAMP
• 19 levels .
Salmeterol Adenylyl cyclase Cyclic adenosine monophosphate Salbutamol
• 20
• 21

6
lock
s
Suspend
0
End Block
Item: 16 of 37 ~. I • M k <:] t> al ~· ~
QIO: 1927 .l. ar Previous Next Lab 'lifllues Notes Calculator

1
Muscarinic acetylcholine receptor Respiratory tract Respiratory disease Receptor (biochemistry)
2
B is not correct. 10% chose this.
3
Methylxanthines such as theophylline act by inh ibiting phosphodiesterase, resulting in increased cAMP levels.
4
Theophylline Xanthine Cyclic adenosine monophosphate Phosphodiesterase
5
C is not correct. 6°/o chose this.
6
Cromolyn acts by inh ibiting the degranu lation of mast cel ls.
7
Cromoglicic acid Degranulation Mast cell
8
D is not correct. 4°/o chose this.
9
Corticosteroids such as beclomethasone and predn isone act by inhibiting the synthesis of cytokines.
10 Prednisone Beclometasone dipropionate Corticosteroid Cytokine
11
E is not correct. 10% chose this.
12
~-Agonists such as albuterol and sa lmeterol act by stimu lating adenyl cyclase, resu lting in increased cAMP
13
levels.
14 Salmeterol Adenylyl cyclase Cyclic adenosine monophosphate Salbutamol

15
16
Bottom Line:
• 17
I pratropium bromide is a muscarinic antagonist that competitively blocks muscar inic receptors, preventing
• 18
acetylchol ine-mediated bronchoconstriction. It is used to treat ch ron ic obstructive pu lmonary disease and
• 19 asthma.
• 20 Ipratropium bromide Chronic obstructive pulmonary disease Muscarinic antagonist Bronchoconstriction Asthma Muscarinic acetylcholine receptor
Receptor antagonist Receptor (biochemistry)
• 21

6
lock
s
Suspend
0
End Block
Item: 16 of 3 7 ~ 1 • M k -<:J 1>- Jil ~· !:';-~
QIO: 1927 ..L ar Pre v ious Next Labfli!llues Notes Calcula t o r
& &
1

2 FA17 p649.1
3 Asthma drugs Bronchoconstriction is mediated b) (I) inOammatory processes and (2) parasympathetic tone;
4 therapy is directed at these 2 path ways.
5 ~-agonists Albuterol- rclaxes bronchial smooth muscle (~hor t acting f3z-agonist). Used during acute
6 exacerbation.
7 Salmeterol, formotero l- long-acting agents for prophylaxis. ,\d\'erse effects are tremor and
8 arrln thmia.
9 Inhaled Fluticasone, budesonide - inhibit the synthesis of ,·irtuallr all cytokines. lnacti,Cite 'F'-KB, the
corticosteroids transcription factor that induces production of T IF-a and other inflam matory agents. 1st-line
10
therapy for chronic asth ma. \'lay cause oral th rush.
11
Muscarinic Tiotropium, ipratropium-competitively block muscarin ic receptors, pre\'enting
12
antagonists bronchoconstriction. Also used for COPD. T iotropium is long acting.
13
Antileukotrienes Montelukast, zafirlukast- block leukotricnc Exposure to ant1gen
14
receptors (C ysLTl). Especially good for (dust. pollen, etc}
15 aspirin-induced and exercise-induced asthma.
16 Zileuton -5-lipoxygenase pathway inh ibitor. ~ Avoidance
• 17 Blocks conversion of arachidonic acid to
leukotrienes. Hepatotoxic.
. 18
Antigen and lgE~ Omahzumab
• 19 Anti-lgE monoclonal Omalizumab- binds mostly un bound serum on mast cells
therapy lgE and blocks binding to FceRI. Used in
• 20
allergic asthma with t lgE levels resistant to
• 21 ~ Steroids
• inhaled steroids and long-acting B,-agonists.

a
Lock
s
Suspend
8
End Bl ock
Item: 16 of 37 ~. I • M k <:] t> al ~· ~
QIO: 1927 .l. ar Previous Next Lab 'lifllues Notes Calculator

1 • •

2 FA17 p638.1

3 Obstructive lung Obstruction of air fl ow - air trapping in lungs. Airways close prematurely at high lung volumes
4 diseases - t RV and t FRC, t TLC. PFTs: U FEV 1, l FVC - l FEV 1/FVC ratio (hallmark),

5
V/Q mismatch. C hronic, hypoxic pulmonary vasoconstriction can lead to cor pulmonale. Chronic
obstructive pulmonary disease (COPD) includes chron ic bronchitis and emphysema.
6
TYPE PRESENTATION PATHOLOGY OTHER
7
Chronic bronchitis Findings: \\·heezing, crackles, Hypertrophy and hyperplasia Diagnostic criteria: productive
8 ("blue bloater") cyanosis (hypoxemia due of mucus-secreting glands cough for> 3 months in a
9 to shunting), dyspnea, C02 in bronchi - Reid index year for> 2 consecutive years.
10 retention, zopolyc)themia. (thickness of mucosal gland
layer to thickness of wall
11
between epithelium and
12
cartilage) > 50%.
13
Emphysema ("pink Centriacinar-associated with Enlargement of air spaces CXR: t AP diameter, flattened
14 puffer") smoking rJ (£). Frequently in l recoil, t compliance, diaphragm, t lung field
15 upper lobes (smoke rises up). l DLCO from destruction of lucency.
16 Panacinar-associated with alveolar walls (arrow in m). Barrel-shaped chest 1!].
• 17 a 1-antitrypsin deficiency. t elastase activity - t loss Exhalation through pursed lips
Frequently in lower lobes. of elastic fibers - t lung to increase airway pressure
• 18
compliance. and prevent airway collapse.
• 19
Asthma Findings: cough, wheezing, Branchial hypcrrcsponsi\·cncss Aspirin-i nduced asthma: COX
0 20
tachypnea, dyspnea, - reversible inhibition - leukotriene
• 21 hypoxemia, l inspiratory/ bronchoconstriction. Smooth overproduction - airway •

6
lock
s
Suspend
0
End Block
Item: 17 of 3 7 ~ 1 • M k -<:J 1>- Jil ~· !:';-~
QIO: 1744 ..L ar Pre v ious Next Labfli!llues Not es Calcula t o r

IAA]
& &
1
A 40-year-old patient with AIDS presents to the clinic with fatigue and weakness . X-ray of the chest is shown
2
in the image . A methenamine si lver stain of a bronchoalveolar lavage specimen reveals multiple organisms.
3 Treatment w ith t r imethoprim-su lfamethoxazole is contraind icated because the patient has a sulfa al lergy.
4
5
6
7
8
9
10
11
12
13
14
15
16
• 17
• 18
• 19
• 20
• 21
• o o •I • I ,. • I ,- II
' ' . ' . • • I • • I • ,-

a
Lock
s
Suspend
8
End Bl ock
12
Which of the fol lowing agents shou ld be used to treat this patient's infection?
13
:
14
A. Penici llin
15
16 B. Pentamid ine
• 17 C. Ivermectin
• 18
D. Metronidazole
• 19
• 20 E. Protami ne
• 21

a
Lock
s
Suspend
8
End Block
Item: 17 of 37 ~. I • M k <:] t> al ~· ~
QIO: 1744 .l. ar Previous Next Lab 'lifllues Notes Calculator

1 •

2 The correct answer is B. 51°/o chose this.


3
The diffuse hazy "groundg lass" opacities and interstit ial infiltrate seen on this chest x-ray are consistent with
Pneumocystis jirovecii pneumonia, which is confirmed by methenamine silver stain of sample (typical ly
4
bronchoalveolar lavage, but sometimes induced sputum or or biopsy samples) . The fi rst-l ine treatment of th is
5 cond ition is t rimethoprim -sulfamethoxazo le for patients with no contraindications to sulfa drugs. I n the case of a
6 patient with a su lfa allergy, the first-line treatment is pentamidine.
7 Pentamidine Pneumocystis jirovecii Trimethoprim/sulfamethoxazole Bronchoalveolar lavage Sulfonamide (medicine) Sputum Pneumonia Chest radiograph Biopsy

8 Allergy X-ray Pneumocystis pneumonia Hexamethylenetetramine

9
A is not correct. 11% chose this.
10 Penicil lin is used to treat a variety of bacter ial infections but is not effective in the treatment of Pneumocystis
11 jirovecii pneumonia.
12 Penicillin Pneumocystis jirovecii Pneumonia Pneumocystis pneumonia

13
C is not correct. 13% chose this.
14
I vermectin is used to treat onchocerciasis (river blindness) . It is thought to block the release of microfilariae
15 from gravid female wor ms . One dose reduces microfi larial counts by up to 95 % .
16 Ivermectin Onchocerciasis Microfilaria Visual impairment Pregnancy

17 D is not correct. 19% chose this.


• 18 Metronidazo le is used to treat a wide variety of anaerobic bacterial infections. It is also effective against
• 19 parasit ic infections such as Giardia Iamblia, Entamoeba histolytica, Gardnerel/a vagina/is, and Trichomonas
• 20
vagina/is .
Giardia Iambiia Metronidazole Entamoeba histolytica Gardnerella vaginalis Trichomonas vaginalis Giardia Anaerobic organism Trichomonas Parasitism Entamoeba
• 21

6
lock
s
Suspend
0
End Block
Item: 17 of 37 ~. I • M k <:] t> al ~· ~
QIO: 1744 .l. ar Previous Next Lab 'lifllues Notes Calculator

1 •
Penicillin Pneumocystis jirovecii Pneumonia Pneumocystis pneumonia
2
3
C is not correct. 13% chose this.
4
I ver mect in is used to t reat onchocerciasis (river bl indness) . It is thought to block the release of microfila riae
from gravid fema le wor ms . One dose reduces microfi larial count s by up t o 95% .
5 Ivermectin Onchocerciasis Microfilaria Visual impairment Pregnancy
6
D is not correct. 19% chose this.
7
Metronidazole is used to t reat a wide va riety of anaerobic bacterial infections . It is also effective against
8 parasit ic infections such as Giardia Iamblia, Entamoeba histolytica, Gardnerel/a vagina/is, and Trichomonas
9 vagina/is.
Giardia Iambiia Metronidazole Entamoeba histolytica Gardnerella vaginalis Trichomonas vaginalis Giardia Anaerobic organism Trichomonas Parasitism Entamoeba
10
11 E is not correct. 6°/o chose this.
12 Protamine is used t o treat hepa rin overdose. It binds t o hepar in to form a stable complex that has no
13 anticoagu lant activity. Used alone, protamine has anticoagulant properties .
Anticoagulant Protamine Heparin Drug overdose
14
15
16 Bottom Line:
17 A var iet y of drugs can be employed for the t reatment of AIDS-related Pneumocystis jirovecii pneumonia,
• 18 including trimethoprim -sulfamethoxazole (contraindicated secondary to su lfa allergy in this case) and
• 19
pent amidine .
Pentamidine Pneumocystis jirovecii Trimethoprim/sulfamethoxazole Pneumonia Sulfonamide (medicine) Allergy Pneumocystis pneumonia
• 20
• 21

6
lock
s
Suspend
0
End Block
Item: 17 of 3 7 ~ 1 • M k -<:J 1>- Jil ~· !:';-~
QIO: 1744 ..L ar Pre v ious Next Labfli!llues Not es Calcula t o r
& &
1
FA17 p 150.1
2

3 Pneumocystisjirovecii Causes Pneumocystis pneumonia (PC P), a diffuse interstitial pneumonia rJ. Yeast-like
fu ngus (originally classified as protozoan). Inhaled. \lost infections are asymptomatic.
4
Immunosuppression (eg, AI OS) predisposes to disease. Diffuse, bilateral ground-glass opacities on
5
CXRICT ()]. Diagnosed b) lung biopS) or lavage. Disc-shaped yeast seen on methenamine sih er
6 stain of lung tissue ~-
7 Treatment/prophylaxis: T~ I P-Si\ I X, pentamidine, dapsone (prophylaxis onJy), atovaquonc. Start
8 prophylaxis when C D4+ count drops to < 200 cells/mm3 in HIV patients.
9
10
11

12
13
14
15
16
17
. 18
• 19 FA17 p 173.1
• 20 Common diseases of As CD4+ cell count l , risks of reactivation of past infections (eg, T B, I ISV, shi ngles), dissemination
• 21 HIV-positive adults ofbactcriaJ infections and fungal i nfection ~ (cg, coccidioidomycosis), and non-Hodgkin

a
Lock
s
Suspend
8
End Bl ock
Item: 17 of 37 ~. I • M k <:] t> al ~· ~
QIO: 1744 .l. ar Previous Next Lab 'lifllues Notes Calculator

1 • •
FA17 p 173.1
2
3 Common diseases of As CD4+ cell counl !, risks of reactivation of past infections (eg, TB, llSV, sh ingles), dissemination
HIV-positive adults of bacterial infections and fungal infections (eg, coccidioidomycosis), and non-Hodgkin
4
lymphomas t.
5
PATHOGEN PRESENTATION FINDINGS
6
CD4+ cell count < 500/mm3
7
Candida albicans Oral thrush Scrapable white plaque, pseudohyphae on
8
mtcroscopy
9
EBV Oral hairy leukoplakia Unscrapable white plaque on lateral tongue
10
Bartonella henselae Bacillary angiomatosis Biopsy wi th neut rophilic inAammation
11
HHV-8 Kaposi sarcoma Biopsy wi th lymphocytic inflammation
12
HPV Squamous cell carcinoma, commonly of anus
13 (men who have sex with men) or cervix
14 (women)
15 CD4+ cell count < 200/mm3
16 Histoplasma Fever, weight loss, fatigue, cough, dyspnea, Oval yeas t cells within macrophages
17 capsulatum nausea, vomiting, diarrhea
• 18 HIV De mentia
• 19 JC virus {reactivation) Progressive multifocalleukoencephalopathy onenhancing areas of demyel ination on M Rl
• 20 Pneumocystis jirovecii PneumOC)'Stis pneumonia "Ground-glass" opacities on CXR
• 21 CD4+ cell count < 100/mm3 •

6
lock
s
Suspend
0
End Block
Item: 17 of 3 7 ~ 1 • M k -<:J 1>- Jil ~· !:';-~
QIO: 1744 ..L ar Pre v ious Next Labfli!llues Not es Calcula t o r
I • I I
1

2 Aspergillus fumigatus Hemoptysis, pleuritic pain Cavitation or infiltrates on chest imaging


3 Candida alb/cans Esophagitis \Vh itc plaques on endoscopy; yeast and
4
pscudohyphae on biopsy
CMV Retinitis, esophagitis, colitis, pneumonitis, Linear ulcers on endoscopy, cotton-wool spots
5
encephalitis on fundoscopy
6
Biopsy re\'eals cells with intranuclear (owl eye)
7 inclusion bodies
8 Cryptococcus Meningitis Encapsulated yeast on India ink stain or
9 neoformans capsular antigen $
10 Cryptosporidium spp. Chronic, watCf}' diarrhea Acid-fast oocysts in stool
11 EBV B-celll~mphoma (eg, non-Hodgkin lymphoma, Cl S lymphoma-ring enhancing, may be
12 C NS lymphoma) solitary (,·s Toxoplasma)
13 Mycobacterium I onspecific systemic symptoms (fever, night
14
avium- intracellulare, sweats, \\'eight loss) or focaI lymph aden it is
Mycobacterium avlum
15
complex
16
Toxoplasma gondii Brain abscesses lultiple ring-enhancing lesions on IRI
17
. 18
• 19 FA17 p 194.2
Prophylaxis in HIV patients
• 20
CELLCOUNT PROPHYLAXIS INFECTION
• 21
• T \,1P-SI\ I X

a
Lock
s
Suspend
8
End Bl ock
Item: 17 of 3 7 ~ 1 • M k -<:J 1>- Jil ~· !:';-~
QIO: 1744 ..L ar Pre v ious Next Labfli!llues Not es Calcula t o r

1
&
CMV Retinitis, esophagitis, colitis, pneumonitis, Linear ulcers on endoscopy, cotton-wool spots &

2
encephal itis on fundoscopy
Biopsy reveals cells with intranuclear (owl eye)
3
inclusion bodies
4
Cryptococcus Meningitis Encapsulated yeast on India ink stain or
5 neoformans capsular antigen$
6 Cryptosporidium spp. Chronic, watery diarrhea Acid-fast oocvsts in stool
7 EBV B-cell lymphoma (eg, non-Hodgkin lymphoma, CNS lymphoma-ring enhancing, may be
8 Cr\S lymphoma) solitary (,·s Toxoplasma)
9 Mycobacterium Nonspecific systemic symptoms (fever, night
10 avium- intracellulare, sweats, weight loss) or focal lymphadenitis
11 Mycobacterium avium
complex
12
Toxoplasma gondii Brain abscesses lultiple ring-enhancing lesions on J\ IRl
13
14
15 FA17 p 194.2

16 Prophylaxis in HIV patients


CELL COUNT PROPHYLAXIS INFECTION
17
C04 < 200 cells/mm3 TMP-SMX Pneumocystis pneumonia
. 18
• 19
C04 < 100 cells/mm3 T.\IIP-SMX Pneumocystis pneumonia and toxoplasmosis
• 20 C04 < 50 cells/mm3 Azithromycin or clarithromycin J\lrcob(ICierium avium complex
• 21

a
Lock
s
Suspend
8
End Bl ock
Item: 18 of 3 7 ~ 1 • M k -<:J 1>- Jil ~· !:';-~
QIO: 2216 ..L ar Pre v ious Next Lab fli!ltues Notes Calcula t o r
& &
1
A 71-year-old man comes to the physician complaining of a 4-month history of fatigue, low-grade fevers,
2
night sweats, and cough. He became extremely worried yesterday when he noticed blood in his sputum. On
3 physical examinat ion, he is extremely thin, w ith enlarged, nontender left-sided cervical lymph nodes. His X-
4 ray film of the chest is shown. The physician prescribes medication not only for the patient, but also for those who
may have been in close contact with him.
5
6
7
8
9
10
11
12
13
14
15
16
17
. 18
. 19
• 20
. 21

a
Lock
s
Suspend
8
End Bl ock
11
Which of the following is the most likely adverse effect of the first-line med ication for t he prevention of this
12
disease?
13
:
14
A. A disu lfiram- like reaction with alcohol
15
B. Aplastic anemia
16
17 C. Hepatot oxicity
. 18 D. Inhibition of bone growth in ch ildren
. 19
E. Nephrotoxicity
• 20
. 21

a
Lock
s
Suspend
8
End Block
Item: 18 of 3 7 ~ 1 • M k -<:J 1>- Jil ~· !:';-~
QIO: 2216 ..L ar Pre v ious Next Lab fli!ltues Notes Calcula t o r
& &
1
The correct a nswer is C. 63°/o chose this.
2
The symptoms of fever, fatigue, night sweats, lymphadenopathy, and
3 hemoptysis are consistent with the diagnosis of tuberculosis (TB). The
4 causative agent of TB is Mycobacterium tuberculosis. Primary tuberculosis
5 infections are only rarely symptomatic in patients with normal immune
function due to rapid containment by resident alveolar macrophages and
6
infiltrating monocytes and lymphocytes. Symptomatic primary infection is
7 mostly seen in the elderly, children, and immunocompromised individuals.
8 Primary TB resembles an acute bacterial pneumonia, is typically located in the
lower and middle lobes, and rarely causes cavitation. Secondary TB often
9
localizes to the apex/upper lobes of the lungs, such shown in the X-ray film.
10 The arrow indicates a right apical opacity compatible with TB. A caseating
11 granuloma is formed in which necrotic tissue and bacteria are surrounded by
12 macrophages and giant cells. TB is initially treated with isoniazid, rifampin,
ethambutol, and pyrazinam ide (with streptomycin as a second-line agent
13
shou ld resistance to f irst- line drugs occur); however, isoniazid can be used
14 alone to prevent TB or to treat suspected cases of latent TB. The adverse
15 effects of isoniazid include neu rotoxicity and hepatotoxicity.
Hemoptysis Isoniazid Pyrazinamide Ethambutol Mycobacterium tuberculosis Streptomycin Hepatotoxicity Lymphadenopathy Rifampicin Tuberculosis
16
Immunodeficiency Pneumonia Mycobacterium Granuloma Lymphocyte Necrosis Night sweats Fatigue (medical) Neurotoxicity Immune system Macrophage
17
Monocyte Bacteria X- ray Caseous necrosis Pulmonary alveolus Bacterial pneumonia Infection Lung Fever
18
. 19 A is n ot correct. 9°/o chose this.
• 20 This is an adverse effect of metronidazole, which is not used to treat or prevent tuberculosis. Metronidazole is
. 21 an inhibitor of nucleic acid synthesis in bacterial cel ls. It is the drug of choice for treatment of iatrogenic
• r i ........ .,_:...J:. · - ...J:a:-:1- --•=•=- T• - - - -•-- a... ... ··--..J ...... •---• --...1----..J:•:- --..J L...--•--:-1 ~~--:---:- n.•t...-- _..,,, ___ _

a
Lock
s
Suspend
8
End Bl ock
Item: 18 of 37 ~. I • M k <:] t> al ~· ~
QIO: 2216 .l. ar Previous Next lab 'lifllues Notes Calculator

1 •
A is not correct. 9°/o chose this .
2 This is an adverse effect of metronidazo le, which is not used to treat or prevent tuberculosis. Metronidazole is
3 an inhibitor of nucleic acid synthesis in bacterial cel ls. It is the drug of choice for t reatment of iatrogenic
Clostridium difficile co litis. It can also be used to treat endocarditis and bacteria l vaginosis. Other adverse
4
events associated with metronidazole use include nausea, vomit ing, abdominal pain, and metal lic taste in the
5 mouth. Other medications that can cause a disulfiram-like reaction with alcohol include cefotetan and other
6 cephalospor ins.
Metronidazole Cefotetan Tuberculosis Iatrogenesis Nucleic acid Endocarditis Bacterial vaginosis Clostridium difficile colitis Clostridium difficile (bacteria)
7
Cephalosporin Nausea Abdominal pain Colitis Adverse effect Vomiting Clostridium Alcohol Enzyme inhibitor
8
9 B is not correct. 10% chose this.
10 Aplastic anemia is not common ly associated with isoniazid . Certain classes of drugs are known to potential ly
11
induce ap lastic anemia as a side effect-examples include ch loramphenicol (commonly prescribed to t reat
conjunctivit is), carbamazepine (useful for treatment of neuralgias and seizure disorders), phenytoin (an
12
anticonvulsant), and quinine (antima larial). None of these drugs would be appropriate to treat th is patient,
13 however.
14 Carbamazepine Anticonvulsant Isoniazid Aplastic anemia Phenytoin Chloramphenicol Quinine Conjunctivitis Anemia Malaria Antimalarial medication

15 Epileptic seizure

16 D is not correct. 5°/o chose this.


17 This effect has been associated with tetracyclines, which are not used as first- line treatment or for prevention of
18 tuberculosis. Tetracyclines are bacteriostatic antibiotics. They bind to and inhibit the 305 r ibosomal subunit.
• 19
Tetracycl ines can be used to treat infections with Rickettsiae, Yersinia pestis, Mycoplasma, Chlamydia, the
spirochetes, and even Bacillus anthracis. They are contraindicated in pregnancy because they can cross the
• 20 placenta and cause damage to feta l bones and teeth, an effect that is also true for treatment of young children
• 21 with tetracycl ines .

6
lock
s
Suspend
0
End Block
Item: 18 of 37 ~. I • M k <:] t> al ~· ~
QIO: 2216 .l. ar Previous Next lab 'lifllues Notes Calculator

1 •
This effect has been associated with tetracyclines, which are not used as first- line treatment or for prevention of
2 tuberculosis. Tetracyclines are bacteriostatic antibiotics. They bind to and inhibit the 305 ribosomal subunit.
3
Tetracycl ines can be used to treat infections with Rickettsiae, Yersinia pestis, Mycoplasma, Chlamydia, the
spirochetes, and even Bacillus anthracis. They are contraindicated in pregnancy because they can cross the
4
placenta and cause damage to feta l bones and teeth, an effect that is also true for treatment of young children
5 with tetracyclines.
Bacillus anthracis Yersinia pestis Bacteriostatic agent Placenta Tuberculosis Tetracycline antibiotics Rickettsia Mycoplasma Chlamydia infection Chlamydia (genus)
6
7 305 Antibiotics Spirochaete Bacillus Ribosome Pregnancy

8 E is not correct. 13% chose this.


9 Nephrotoxicity, ototoxicity, and teratogenic effects are associated with aminog lycosides, which are often used to
10 treat gram-negative infections, not to prevent or treat tuberculosisB. Aminog lycosides are bactericidal inhibitors
of protein synthesis and are particularly useful in treatment of infections with Pseudomonas, Enterobacter, and
11
Acinetobacter species. I t is also worth mentioning that many other drugs can have nephrotoxicity as a side
12 effect, such as amphotericin (anti-funga l agent), cisplatin (chemotherapeutic), and contrast dyes.
13 Cisplatin Nephrotoxicity Ototoxicity Gram-negative bacteria Teratology Aminoglycoside Bactericide Protein Chemotherapy Pseudomonas Enterobacter

14 Radiocontrast agent Acinetobacter Amphotericin B Adverse effect Protein synthesis Protein biosynthesis

15
16
Bottom Line:
17
Fever, fatigue, night sweats, and hemoptysis with consol idation in the apica l lobes of the lung on X-ray fi lm are
18
consistent with reactivated TB. This can be prevented in many cases with isoniazid, whose toxicities include
• 19 hepatotoxicity and neurotoxicity .
• 20 Hemoptysis Isoniazid Hepatotoxicity Night sweats Fatigue (medical) Neurotoxicity lung X-ray Fever

• 21

6
lock
s
Suspend
0
End Block
Item: 18 of 3 7 ~ 1 • Ma r k -<:J I> ~ £!1}>'
• !!":-~
QIO: 2216 ..L Pre v ious Next Lab lues Notes Calcula t o r
& &
1

2 FA17 p 136.1

3 Primary and secondary tuberculosis


4 PPD (±) if current infection or past exposme.
5 PPD 8 if no infection and in sarcoidosis or
6 Ill infection (especially'' ith lo" CD-++ cell
count).
7
lnterferon-y release assay (ICR ) has fewer fal se
8 Primary tuberculosis posili,·es from BCC "accinalion.
9 >90X <10% Caseating granulomas rJ with central necrosis
10 Heabng by fibr®s

ProgresSIVe primary tuberculoSIS (upper left) and Langhans giant cells (arrow)
Calotiutoo (AIDS. matnutntJOn)
11 (tubercwn @l
I are characteristic of 2° tuberculosis.
l
12
13
14
Fibrocaseous
ca111tary le!j()n
1Re.KIIVation
2' tuberculosis
'
.., ,
l
Bacteremia
~~~Av· ._ _______ _j
Progress.ve
lung disease

15
(usually upper
lobes)
..
_. Miliary ~Meninges

16 •':, ~losis
~ Vertebrae
17 1
Loulized dtilructive disease Lymph nodes
' { ...--- (~disease)
18
Ca111ty
••••
• 19 caseation Lungs _ _.,.
t:O '-
caseaoon
• 20 Liver ~:.~·
Adrenal
• 21 gland

a
Lock
s
Suspend
8
End Bl ock
Item: 18 of 3 7 ~ 1 • M k -<:J 1>- Jil ~· !:';-~
QIO: 2216 ..L ar Pre v ious Next Lab fli!ltues Notes Calcula t o r
& &
1
FA17 p 193.1
2
Isoniazid
3
MECHANISM ~ synthesis of mycolic acids. Bacterial catalase-
4 peroxidase (encoded by KatG) needed to
5 com·ert INH to active metabolite.
6 CliNICAL USE lycobacterium tuberculosis. The only agent Different I~ H half-liYes in fast vs slow
7 used as solo prophylaxis against TB. Also used aceh-lators.
8 as monotherapy for latent TB.
9 ADVERSE EffECTS Hepatotoxicity, P-450 inhibition, drug-induced 1'\'11 Injures ~eurons and l lepatocytes.

10
SLE, anion gap metabolic acidosis,' itamin
8 6 deficiency (peripheral neuropathy,
11
sideroblastic anemia). dminister with
12 pyridoxine (8 6).
13 MECHANISM OF RESISTANCE Mutations leading to underexpression of KatC.
14
15 FA17 p 192.1
16 Anti mycobacterial drugs
17 BACTERIUM PROPHYLAXIS TREATMENT

18 M tuberculosis Isoniazid Rifampin, Isoniazid, P yrazinamide,


• 19 Ethambutol (RIPE for treatment)
• 20 M avium- intracellulare Azithromycin, rifabutin ~ l oredrug resistant than .\4 tuberculosis.
zithromycin or clarithromycin +ethambutol.
• 21
• r~n ~rlrl rir!lhutin nr ,..inrnRn,.!l,..in

a
Lock
s
Suspend
8
End Bl ock
Item: 18 of 37 ~ .I • M k <:] t> al ~· ~
QIO: 2216 .l. ar Previous Next lab 'lifllues Notes Calculator

1 FA17 p 192.1
2 Anti mycobacterial drugs
3 BACTERIUM PROPHYLAXIS TREATMENT
4 M tuberculosis Isoniazid Rifampin, Isoniazid, Pyrazinamide,
5 Ethambutol (RIPE for treatment)
6 M avium- intracellulare Azithromycin, rifabutin More drug resistant than M tuberculosis.
7
Azithromycin or clarithromycin +ethambutol.
Can add rifabutin or ciproAoxacin.
8
M leprae 'lA Long-term treatment with dapsone and rifampin
9
for tuberculoid form. Add clofazimine for
10 lepromatous form.
11
MYCOBACTERIAL CELL Plasma
12 Cell wall membrane Interior of cell

13
14
mRNA mRNASYNTHESIS
15 DNA-dependent
MYCOLIC ACID N oly ease
16 SYNTHESIS

l
17
Rifabutin
RNA
18 polymerase Rifampin
• 19
ARABINOGALACTAN
• 20 SYNTHESIS INTRACELLULAR
• 21 (arabinosyl transferase) (unclear mechamsml

6
lock
s
Suspend
0
End Block
Item: 19 of 3 7 ~ 1 • M k -<:J 1>- Jil ~· !:';-~
QIO: 44 70 ..L ar Pre v ious Next Lab fli!ltues Not es Calcula t o r

2
A 45-year-old woman presents w ith a complaint of difficulty sleeping two to three times per week because of IAA]
mi ld wheezing. She is currently taking albuterol by metered-dose inhaler. Other than the wheezing, she
3 reports no worsening of her breath ing condition. She is placed on a medication that stabilizes the type of
4 cells pictured in the image.
5
6
7 •

8
9

10
'
11
• •
12 •
13
14
15
16
17
18
• 19
Wh ich medication was she prescribed?
• 20
:
• 21
• A. Cromolvn sodium
a
Lock
s
Suspend
8
End Bl ock
12
13 Which medication was she prescribed?
14
:
15 A. Cromolyn sodium
16
B. l pratropium brom ide
17
18 C. Montelukast
• 19 D. Salmeterol
• 20
E. Theophyll ine
• 21

a
Lock
s
Suspend
8
End Block
Item: 19 of 3 7 ~ 1 • M k -<:J 1>- Jil ~· !:';-~
QIO: 44 70 ..L ar Prev ious Next Lab fli!ltues Not es Calculat o r

& &
1
2 The correct answer is A. 57°/o chose this.
3 The cel ls w ith many granules, indicated by red circles in the image, are mast
4
cells. Cromolyn works at the surface of mast cel ls to inh ibit their
degranulation. This, in turn, prevents the release of histamine and slow-
5 reacting substance of anaphylaxis, mediators of type I allergic reactions.
6 Cromolyn may also reduce the release of inflammatory leukotrienes. The
7 patient has mild asthma, and cromolyn sodium can be used as an adjuvant
treatment alongside of her albuterol inhaler.
8
Anaphyla. is Cromoglicic acid Histamine Asthma Slow-reacting substance of anaphylaxis Leukotriene Mast cell
9
Degranulation Salbutamol Inhaler Allergy Sodium
10
11
12 8 is not correct. 9°/o chose this.
13 Antimuscarinics (ie, ip ratrop ium bromide) cause vagolytic action by competitive inhibition of muscarinic
receptors on airway smooth muscle (M3 type), leading to bronchodilation. They have no effect on mast cel l
14
activity.
15 Mast cell lpratropium bromide Bronchodilator Smooth muscle tissue Muscarinic acetylcholine receptor Respiratory tract Muscarinic antagonist vagus nerve Muscle

16 Receptor (biochemistry)

17
C is not correct. 20°/o chose this.
18
Leukotriene antagonists (eg, montelukast) act as competitive antagonists of leukotriene (CysLTl) receptors.
19 Montelukast Leukotriene Cysteinyl leukotriene receptor 1 Receptor (biochemistry)

• 20
D is not correct. 7°/o chose this.
. 21
• B.,-Adrenoceotor aaonists (eo. albuterol and sa lmeterol) stimulate airwav B.,-adrenoceotors. Th is causes
a
Lock
s
Suspend
8
End Block
Item: 19 of 37 ~. I • M k <:] t> al ~· ~
QIO: 4470 .l. ar Previous Next lab 'lifllues Notes Calculator

1 •
- - - . ..
Ant imuscarinics ( ie, ipratropium bromide) cause vagolytic action by competit ive inhibit ion of musca rinic
2 recept ors on airway smooth muscle (M3 type), lead ing to bronchodi lation . They have no effect on mast cell
3 activity .
Mast cell Ipratropium bromide Bronchodilator Smooth muscle tissue Muscarinic acetylcholine receptor Respiratory tract Muscarinic antagonist Vagus nerve Muscle
4
5 Receptor (biochemistry)

6 C is not correct. 20% chose this.


7 Leukotriene antagon ists (eg, mont elukast) act as competit ive antagon ists of leukotriene (CysLTl ) receptors.
8 Montelukast leukotriene Cysteinyl leukotriene receptor 1 Receptor (biochemistry)

9 D is not correct. 7°/o chose this.


10 ~rAdrenoceptor agonists (eg, albut erol and salmeterol) stimulat e airway ~radrenoceptors . This causes
11 relaxation of bronch ial smooth muscle.
Salmeterol Smooth muscle tissue Salbutamol Bronchiole Muscle Bronchus Agonist
12
13 E is not correct. 7°/o chose this.
14 Methylxanthines (eg, t heophyl line) cause relaxation of bronch ial smooth muscle by phosphodiesterase
inhibit ion.
15
Theophylline Xanthine Smooth muscle tissue Phosphodiesterase Bronchiole Muscle Bronchus
16
17
18 Bottom Line:
19 Cromolyn sod ium inhibit s degranu lation of t he surfaces of mast cel ls, thus prevent ing release of histamine.
Cromoglicic acid Histamine Degranulation Mast cell Sodium
0 20
0 21

6
lock
s
Suspend
0
End Block
Item: 19 of 3 7 ~ 1 • M k -<:J 1>- Jil ~· !:';-~
QIO: 44 70 ..L ar Pre v ious Next Lab fli!ltues Not es Calcula t o r
& &
1
FA17 p649.1
2
Asthma drugs Bronchoconstriclion is mediated b) (I) inAammatory processes and (2) parasympathetic tone;
3
therapy is directed at these 2 pathways.
4
~-agonists Albuterol- rclaxes bronchial smoot h muscle (short acting 13z-agonist). Used during acute
5 exacerbation.
6
Salmeterol, formoterol - long-acting agents for prophylaxis. Ad,·erse effects are tremor and
7 arrln thmia.
8 Inhaled Fluticasone, budesonide -inhibit the synthesis of ,·irtuallr all cytokines. lnacti,·ate NF'-KB, the
9 corticosteroids transcription factor that induces production ofT lfi'-a. and other inAammatory agents. 1st-line
10 therapy for chronic asthma . .\ lay cause oral thrush.
11 Muscarinic Tiotropium, ipratropium-compet itively block muscarinic receptors, preventing
12 antagonists bronchoconstriction. i\lso used for COPD. Tiotropium is long acting.
13 Antileukotrienes Montelukast, zafirlukast- block lcukotrienc Exposure to an~gen
14
receptors (CysLTl). Especially good for (dust pollen, etc}
aspirin-induced and exercise-induced asthma.
15
Zileuton -5-lipoxygenase pathway inh ibitor. ~ Avoidance
16 Blocks conversion of arachidonic acid to
17 leukotrienes. Hepatotoxic.
Antigen and lgE ~ Omahzumab
18 Anti-lgE monoclonal Omalizumab- binds mostly unbound serum on mast cells
19 therapy lgE and blocks binding to FceRI. Used in
• 20 allergic asthma with t lgE levels resistant to ~ Steroids
inhaled steroids and long-acting 13z-agonists.
• 21

a
Lock
s
Suspend
8
End Bl ock
ATP
2
Bronchodilation - 13-agonists
3 Early response: late response
bronchoconstriction mflammat10n
4 - -cAMP
5 Bronchial tone
6
Bronchkll
7 Symptoms
hyperreactJv1ty
ACh ~ - Adenosine
8
9 Muscarinic ~ ~ Theophylline
antagonists
10 Bronchoconstriction
11
12 FA17 p 387.5
13
Mast cell Mediates allergic reaction in local tissues. invok ed in Lypc I hypersensitivity reactions.
14 Mast cells contain basophilic granules · and Cromolyn sodium prevents mast cell
r.J
15 originate from the same precursor as basoph iIs degranulation (used for asthma prophylaxis).
16 but are not the same cell type. Can bind the
17
F'c portion of IgE to membrane. lgE cross-
links upon antigen binding - degranulation
18
- release of histamine, heparin, tryptase, and
19 eosinophil chemotactic factors .
• 20
. 21

a
Lock
s
Suspend
8
End Block
Item: 20 of 3 7 ~ 1 • M k -<:J 1>- Jil ~· !:';-~
QIO: 1734 ..L ar Pre v ious Next Labfli!llues Not es Calcula t o r
& &
1
A 57-year-old man with a recent history of untreated HI V-infection presents to his physician with headache,
2
nausea and vomiting, and a change in menta l status. No nuchal rigid ity is noted. Lumbar puncture reveals a
3 high opening pressure. A preparat ion of his bronchoa lveolar lavage fluid with I ndia ink stain is shown in the
4 image. Appropriate intravenous treatment is begun. A few hours later, the patient experiences intense chi lls and
spikes a fever. Laboratory values reveal hypocalcemia, hypokalemia, hypomagnesemia, and hyperglycemia.
5
6
7
8
9
10
11

12
13
14
15
16
17
Image cour t esy of CDC/Or. Lean or Haley
18
19
Wh ich of the fol lowing drugs is responsible for this patient's new-onset fever and ch ills?
• 20
:
• 21
• A. Amohotericin B
a
Lock
s
Suspend
8
End Bl ock
5
6
7
8
9
10
I mage courtesy of CDC/Or. Leanor Haley
11
12
Which of the following drugs is responsible for th is patient's new-onset fever and ch ills?
13
14 :
A. Amphotericin B
15
16 B. Caspofung in
17 C. Fluconazo le
18
D. Foscarnet
19
• 20 E. Zidovud ine
. 21

a
Lock
s
Suspend
8
End Block
Item: 20 of 37 ~. I • M k <:] t> al ~· ~
QIO: 1734 .l. ar Previous Next Lab 'lifllues Notes Calculator

1 •
The correct answer is A. 73°/o chose this.
2
The image shown in the vignette above demonstrates an encapsu lated yeast that stains with India ink, wh ich is
3
a pathognomon ic description of Cryptococcus neoformans, a yeast found in pigeon droppings. Infection occurs
4 when patients inhale fungus particles, which can lead to cryptococca l pneumonia. It typically affects HIV-
5 infected patients, and is an AIDS-defining disease. Initial treatment of C. neoformans is induction with
6
intravenous liposomal amphotericin B and flucytosine, followed by consol idation therapy with fluconazole once
the patient's condition is stable . The most commonly associated adverse effects of amphotericin toxicity are
7 nausea, vomiting, infusion -related fever and chil ls, nephrotoxicity (seria lly mon itor BUN and creatinine), and
8 electrolyte disturbances (particu larly hypokalemia, hypomagnesemia) lead ing to arrhythmias .
Cryptococcus neoformans Amphotericin B Hypokalemia Hypomagnesemia Fluconazole Flucytosine Nephrotoxicity Creatinine Pneumonia Electrolyte
9
10 Intravenous therapy Pathognomonic India ink Nausea Yeast Blood urea nitrogen liposome Vomiting India Fever Cardiac arrhythmia Fungus Cryptococcus

11 Infection

12 B is not correct. 6°/o chose this.


13 Caspofungin, an antifungal med ication used to treat aspergillosis infection, inh ibits synthesis of the ~(1,3)-D­
14 glucan component of the fungal cell wal l. Common adverse effects include flushing and gastrointestinal upset.
Caspofungin Antifungal Aspergillosis Cell wall Fungicide Fungus Gastrointestinal tract Pharmaceutical drug Infection
15
16 C is not correct. 13% chose this.
17 Fluconazole is an antifungal that inhibits ergosterol synthesis . It is used to treat systemic mycoses but is less
effective than amphotericin B and is an adjunct therapy in acute cases. I n patients with AIDS, the initial
18
amphotericin and flucytosine course shou ld be followed by maintenance fl uconazole therapy daily for life.
19 Adverse effects include nausea, vomiti ng, headache, fever, and hepatotoxicity.
20 Amphotericin B Ergosterol Fluconazole Flucytosine Hepatotoxicity Mycosis Antifungal Fungicide HIV/AIDS Nausea Headache Vomiting Adjuvant therapy Fever

• 21
• D is not correct. 5°/o chose this .
6
lock
s
Suspend
0
End Block
Item: 20 of 37 ~. I • M k <:] t> al ~· ~
QIO: 1734 .l. ar Previous Next lab 'lifllues Notes Calculator

1 •
D is not correct. 5°/o chose this.
2
Foscarnet is a pyrophosphate ana log that inhibits vi ral DNA/RNA polymerase and HIV reverse t ransciptase . It
3 may cause similar adverse effects to amphotericin B such as fever, electrolyte abnormal it ies (eg, hypokalemia
4 and hypomagnesemia ) that may lead to seizures, and major nephrotoxicity ( ensu re adequate hydration).
5 Foscarnet is used to treat cytomegalovirus retinitis in AIDs patient and acyclovi r -resistant mucocutaneous HSV
infection in immunocompromised patients. It is not indicated for treatment of Cryptococcus neoformans.
6 Cryptococcus neoformans Amphotericin B Hypokalemia Hypomagnesemia Cytomegalovirus Foscarnet Nephrotoxicity Cytomegalovirus retinitis Immunodeficiency
7
Electrolyte Herpes simplex HIV Herpes simplex virus Epileptic seizure Fever HIV/AIDS Pyrophosphate Retinitis Virus
8
E is not correct. 3°/o chose this.
9
Zidovud ine is an antiretroviral drug used to prevent and t reat HIV/ AIDs . It is a drug of the nucleoside ana log
10
reverse t ranscriptase inh ibitor (NRTI ) class and typica lly used in combination with at least two other
11 antiretroviral agents. Although it is likely that this patient was also started on appropriate HIV antiretroviral
12 therapy, his acute-onset symptoms of fever, chills, and electrolyte disturbances ( particu larly hypoka lemia and
hypomagnesemia) are more likely due to amphothericin B, an antifungal therapy used to treat cryptoccous
13
pneumon ia. Important adverse effects of zidovudine include pancytopen ia, lactic acidosis, and severe
14 hepatotoxicity.
15 Zidovudine Management of HIV/AIDS Hypokalemia Pancytopenia Hepatotoxicity Reverse-transcriptase inhibitor Reverse transcriptase lactic acidosis Pneumonia

16 HIV Antifungal Electrolyte Nucleoside analogue Nucleoside Fever Retrovirus

17
18
Bottom Line:
19
Amphotericin toxicity can cause f ever and chills, nephrotoxicity, and arrhythmias due to distu r bances in
20
potassium and magnesium levels .
• 21 Nephrotoxicity Toxicity Cardiac arrhythmia Magnesium Potassium Fever Chills Amphotericin B

6
lock
s
Suspend
0
End Block
Item: 20 of 3 7 ~ 1 • M k -<:J 1>- Jil ~· !:';-~
QIO: 1734 ..L ar Pre v ious Next Labfli!ltues Not es Calcula t o r
& &
1
FA17 p 194.4
2
3 Antifungal therapy
4 FUNGAL CELL
lANOSTEROL SYNTHESIS CEU WALl SYNTHESIS
5
6 Terbtnafine Echinocandins
Anidulafungin
Squalene Caspofungin
7
X :alene epoxidase Mkafungin
8
Squalene epoxide
9
10
11
• I
Lanosterol Polyenes
• Amphotericin 8
~a-demethylase

, ,. .
12 Ergosterol Nystatin
Azoles
Clotnmazole
13
14
Fluconazole
ltraconazole
' ~,1

Ketoconazole
15 Miconazole
Voriconazole I Flu cytosine
16
17
18 FA17 p 149.1
19 Opportunistic fungal infections
20 Candidaalbicans alba = \\hite. Dimorphic; forms pseudohyphae and budding yeasts at 20°C fJ, germ tubes at
. 21 37°C rn.

a
Lock
s
Suspend
8
End Bl ock
Item: 20 of 3 7 ~ 1 • M k -<:J 1>- Jil ~· !:';-~
QIO: 1734 ..L ar Pre v ious Next Labfli!llues Not es Calcula t o r
& &
1
FA17 p 149.1
2 Opportunistic fungal infections
3 Candido olbicons alba = \\hite. Dimorphic; forms pseudohyphae and budding yeasts at zooc fJ, germ tubes at
4 37°C rn.
5 Systemic or superficial fungal infection. Causes oral and esophageal thrush in
6
immunocompromised (neonates, steroids, diabetes, AI OS), ' uJvo,·aginitis (diabetes, use of
antibiotics), diaper rash, endocarditis (IV drug users), disseminated candidiasis (especially in
7
neutropenic patients), chronic mucocutaneous candidiasis.
8 Treatment: oral Auconazole/topical uole for vagin<J I; nyst<Jtin, Auconazole, or caspofu ngin for oral/
9 esophageal; Auconazole, caspofungin, or amphotericin B for systemic.
10 Aspergillus Septate hyphae that branch at 45° Acute Angle • . Produces conid ia in radiating chains at end of
11 fumigotus conidiophore D .
12 Causes invasive aspergillosis in immunocompromised, patients with chronic granulomatous disease.
Can cause aspergillomas in pre-existing lung cavities, especially afterTB infection .
13
Some species of Aspergillus produce AOatoxins (<~ ssoci a ted with hepatocellular carcinoma).
14
15
Alle rgic bronchopulmonary aspe rgillosis (ABPA): hypersensitivity response associated with
asthma and cystic fibrosis; may cause bronchiectasis and eosinophi lia.
16
Cryptococcus 5-IO !Jm with narrow budding. lleavily encapsulated yeast. l ol dimorphic.
17
neoformons Found in soil, pigeon droppings. Acquired through inhalation with hematogenous dissemination
18 to meninges. C ulture on Sabouraud aga r. ll ighl ighted with India ink (clear halo ) and
19 mucicarmine (red inner capsule [!)). Latex aggluti nation test detects polysaccharide capsular
20 antigen and is more specifi e.
. 21 Causes cryptococcosis, cryptococcal meningitis, crr ptococcal encephalitis ("soap bubble" lesions
• • 1 • '\ •J • • I

Lock
a s
Suspend
8
End Bl ock
Item: 20 of 3 7 ~ 1 • M k -<:J 1>- Jil ~· !:';-~
QIO: 1734 ..L ar Pre v ious Next Labfli!llues Not es Calcula t o r
& &
1 Cryptococcus 5-IO !Jm with narrow budding. llcavily encapsulated yeast. ot dimorphic.
2 neoformans round in soil, pigeon droppings. cquired through inhalation with hematogenous dissemination
3 to meninges. Culture on Sabouraud agar. l lighlighted with India ink (clear halo ) and
mucicarmine (reel inner capsule ['!)). Latex agglutination lest detects polysaccharide capsular
4
antigen and is more specific.
5
Causes cry-ptococcosis, cryptococcal meningitis, crrptococcal encephalitis (~soap bubble" lesions
6 in brain), primarily in immunocompromised.
7 Treatment: amphotericin B + Auc) tosine follo"ed b) Auconazole for cryptococcal meningitis.
8 Mucor and Rhizopus Irregular, broad, nonseptate h) phae branching at wide angles CJ.
9 spp. ~ lucormycosis. Causes disease mostly in ketoacidotic diabetic and/or neutropenic patients (cg,

10 leukemia). Fungi proliferate in blood ves el walls, penetrate cribriform plate, and enter brain.
Rhinocerebral, frontal lobe abscess; cavernous sinus thrombosis. Headache, facial pain, black
11
necrotic eschar on f;1ce; may have cranialncr\ e imoh-cmenl.
12 Treatment: surgical debridement, amphotericin B.
13
14
15
16
17
18
19
20
• 21

a
Lock
s
Suspend
8
End Bl ock
Item: 21 of 3 7 ~ 1 • M k -<:J 1>- Jil ~· !:';-~
QIO: 1705 ..L ar Pre v ious Next Labfli!llues Not es Calcula t o r

IAA]
& &
1
A 60-year-old woman with no past medical history comes to the physician for a cough productive of bloody
2
sputum and a 3-day history of chest pain, fever, and malaise . She had an appendectomy 7 years ago. She
3 lives at home by herself and denies any recent hospitalizations.
4
5 Which of the following is the best first- line treatment for this patient?
6 :
7 A. Ciprofloxacin
8
B. Gentamicin
9
C. Levofloxacin
10
11 D. Piperacillin-tazobactam
12 E. Trimethoprim-su lfamethoxazole
13
14
15
16
17
18
19
20
. 21

a
Lock
s
Suspend
8
End Bl ock
Item: 21 of 37 ~. I • M k <:] t> al ~· ~
QIO: 1705 .l. ar Previous Next lab 'lifllues Notes Calculator

1 •
The correct answer is C. 33°/o chose this.
2 Th is is likely to be pneumococca l pneumonia, which general ly man ifests with a productive cough that is
3 classica lly bloody or rust-co lored. Nonspecific symptoms include fever, chi lls, headache, ma laise, and pleurit ic
4 chest pa in. Fi rst-line treatment for a commun ity-acqu ired pneumon ia, especia lly for patients with significant co-
morbidit ies or for those requ iring inpatient treatment, is either a macro lide, such as azithromycin, plus a th ird -
5
generation cephalosporin or a respiratory fl uoroqu inolone ( levofl oxacin or moxifl oxacin). A macrolide alone is
6 insufficient because Streptococcus pneumoniae is increasing ly resistant to macrol ides.
7 Quinolone Cephalosporin Macrolide Azithromycin Streptococcus pneumoniae levofloxacin Moxifloxacin Community-acquired pneumonia Pneumonia Pleurisy

8 Streptococcus Cough Malaise Headache Chest pain Chills Pneumococcal pneumonia Fever

9 A is not correct. 17% chose this.


10 Ciprofloxacin is a fluoroquino lone that is used to t reat gram-negative rods, most common ly in urinary t ract
11 infections. Un like other f luoroquinolones such as moxifl oxacin and levofloxacin, it has poor potency against
Streptococcus pneumoniae and is genera lly not used to treat pneumon ia.
12
Quinolone Streptococcus pneumoniae levofloxacin Moxifloxacin Ciprofloxacin Gram-negative bacteria Pneumonia Streptococcus Urinary system
13
Urinary tract infection
14
15 B is not correct. 16% chose this.
16
Gentam icin is an aminog lycoside used to treat gram -negative microbial infections, such as Klebsiella . It has
notable side effects, such as ototoxicity and nephrotoxicity.
17 Aminoglycoside Gentamicin Gram-negative bacteria Ototoxicity Nephrotoxicity Klebsiella Adverse drug reaction Microorganism
18
D is not co rrect. 19% chose this.
19
Piperacil lin -tazobactam is used to treat Pseudomonas infections, resistant Staphylococcus aureus infections, and
20 many gram- negative infections. It wou ld not be a first- line therapy for a community- acquired pneumonia in an
21 otherwise healthy patient.

6
lock
s
Suspend
0
End Block
Item: 21 of 37 ~. I • M k <:] t> al ~· ~
QIO: 1705 .l. ar Previous Next lab 'lifllues Notes Calculator

1 •
A is not correct. 17% chose this.
2 Ciprofloxacin is a fluoroquinolone that is used to treat gram-negative rods, most commonly in urinary tract
3 infections . Unl ike ot her fluoroquinolones such as moxifl oxacin and levofloxacin, it has poor potency against
4 Streptococcus pneumoniae and is genera lly not used to treat pneumon ia.
Quinolone Streptococcus pneumoniae levofloxacin Moxifloxacin Ciprofloxacin Gram-negative bacteria Pneumonia Streptococcus Urinary system
5
Urinary tract infection
6
7 B is not correct. 16% chose this.
8 Gent am icin is an aminog lycoside used to t reat gram -negative microbia l infections, such as Klebsiella . It has
notable side effects, such as ototoxicity and nephrotoxicit y .
9
Aminoglycoside Gentamicin Gram-negative bacteria Ototoxicity Nephrotoxicity Klebsiella Adverse drug reaction Microorganism
10
11
D is not correct. 19% c hose this.
Piperacil lin -tazobactam is used t o treat Pseudomonas infections, resist ant Staphylococcus aureus infections, and
12
many gram- negative infections. It wou ld not be a first -line t herapy for a communit y- acqui red pneumonia in an
13 otherwise healthy patient .
14 Piperacillin/tazobactam Gram-negative bacteria Staphylococcus aureus Community-acquired pneumonia Pneumonia Pseudomonas Staphylococcus

15 E is not correct. 15% c hose this.


16 Trimethoprim-su lfamethoxazole (TMP-SMX) is typical ly used in t he t reatment of urina ry t ract infections due to
17 Escherichia coli, Klebsiella, and Enterobacter species, in acute ot itis med ia, in acute exacerbat ions of chronic
18
bronchitis, in traveler's dia r rhea (enterotoxigen ic E. coli), and in Shigella flexneri or Shigella sonnei enterit is.
TMP- SMX is used in treatment and prophylaxis for Pneumocytstis pneumonia .
19 Shigella flexneri Traveler' s diarrhea Escherichia coli Otitis media Trimethoprim/sulfamethoxazole Shigella sonnei Shigella Pneumonia Diarrhea Enteritis
20 Enterobacter Klebsiella Bronchitis Chronic obstructive pulmonary disease Urinary tract infection Urinary system Preventive healthcare
21

6
lock Suspend
s 0
End Block
Item: 21 of 37 ~. I • M k <:] t> al ~· ~
QIO: 1705 .l. ar Previous Next lab 'lifllues Notes Calculator

1
- .. - .. -- . -. - --
notabl e side effects, such as ototoxicity and nephrotoxicit y .
--.. -- ~ --- - - - . . -
2 Aminoglycoside Gentamicin Gram-negative bacteria Ototoxicity Nephrotoxicity Klebsiella Adverse drug reaction Microorganism

3
D is not correct. 19% chose this.
4
Piperacil lin -tazobactam is used t o treat Pseudomonas infections, resist ant Staphylococcus aureus infections, and
5 many gram- negative infections. It wou ld not be a first-l ine t herapy for a communit y- acqui red pneumonia in an
6 otherwise healthy patient .
Piperacillin/tazobactam Gram-negative bacteria Staphylococcus aureus Community-acquired pneumonia Pneumonia Pseudomonas Staphylococcus
7

8 E is not correct. 15% chose this.


9 Trimethoprim-su lfamethoxazole (TMP-SMX) is typical ly used in t he t reatment of urina ry t ract infections due to
10
Escherichia coli, Klebsiella, and Enterobacter species, in acute ot itis med ia, in acute exacer bat ions of chronic
bronchitis, in t raveler's dia r rhea (enterotoxigen ic E. coli ), and in Shigella flexneri or Shigella sonnei enterit is.
11 TMP- SMX is used in t reatment and prophylaxis for Pneumocytstis pneumonia .
12 Shigella flexneri Traveler' s diarrhea Escherichia coli Otitis media Trimethoprim/sulfamethoxazole Shigella sonnei Shigella Pneumonia Diarrhea Enteritis

13 Enterobacter Klebsiella Bronchitis Chronic obstructive pulmonary disease Urinary tract infection Urinary system Preventive healthcare

14
15
Bottom Line:
16
Streptococcus pneumoniae, a gram -posit ive diplococcus sensit ive t o optochin, is the most common cause of
17
bacteria l pneumonia in adult s. First- line treatment for pneumon ia in an otherwise healthy pat ient is a
18 respirat ory fluoroqu inolone or a macrolide plus a thi rd-generation cephalosporin (t ypically azith romycin and
19 ceftriaxone) .
Quinolone Cephalosporin Macrolide Azithromycin Streptococcus pneumoniae Ceftriaxone Bacterial pneumonia Gram-positive bacteria Pneumonia Optochin
20
Streptococcus Diplococcus
21

6
lock Suspend
s 0
End Block
Item: 21 of 3 7 ~ 1 • M k -<:J 1>- Jil ~· !:';-~
QIO: 1705 ..L ar Pre v ious Next Labfli!llues Not es Calcula t o r
& &
1

2 FA17 p 184.1

3 Penicillinase-sensitive Amoxicillin, ampicillin; aminopcnicillins.


4 penicillins
5 MECHAN ISM Same as penicillin. Wider spectrum; \ \1 inoPenicillins are Al\ IPed-up penicillin.
6 penicillinase sensitive. Also combine" ith AmO ,icillin has greater Oral bioa,·ailability
clavulanic acid to protect against destruction than ampicillin.
7
br ~-lactamase.
8
CLINICALUSE Extended-spectrum pcnicillin-11 in/luen::ae, Co' crage: ampicillin/amoxicillin III IELPSS
9
H pylori, E coli, Listeria monocrtogenes, kill enterococci.
10
Proteus mirabilis, Salmonella, Shigella,
11 enterococci.
12 ADVERSE EFFECTS Hypersensitivity reactions; rash;
13 pseudomembranous col it is.
14 MECHANISMOF RESISTANCE Penicillinase in bacteria (a type of ~-lactama sc)
15 clem·cs ~-lactam ring.
16
17 FA17 p 132.2

18 Streptococcus Gram<±'>, lancet-shaped diplococci Pneumococcus is associated with "rusty"


19 pneumoniae Encapsulated. lgA protease. Optochin sputum, sepsis in patients with sickle cell
20 sensiti\'e. \tfost common cause of: disease, and asplenic patients.
r.J " • .Meningitis 'o virulence without capsule.
21 ,
• • {)fitic m,o..rli..-. f ir'l l'h;lrl r.o•"\\

a
Lock
s
Suspend
8
End Bl ock
Item: 21 of 3 7 ~ 1 • M k -<:J 1>- Jil ~· !:';-~
QIO: 1705 ..L ar Pre v ious Next Labfli!llues Not es Calcula t o r
& &
1
FA17 p 132.2
2
Streptococcus Gram $, lancet-shaped diplococci · . Pneumococcus is associated with "rusty"
3
pneumoniae Encapsulated. lgA protease. Optochin sputum, sepsis in patients with sickle cell
4
sensitive. yfost common cause of: disease, and asplenic patients.
5 fJ " Meningitis 'o \'irulence without capsule.
,
6 • Otitis media (in children)
7 "' Bacterial pneumonia
8
• Sinusitis
9
10 •
11

12 FA17 p 189.2

13
Macrolides Azithromycin, clarithromycin, erythromycin.
14 MECHANISM Inhibit protein synthesis by blocking translocation ("m acro~li dc~"); bind to the 23S rRNA of the
50S ribosomal subunit. Bacteriostatic.
15
16
CLINICAL USE Atypical pneumonias (Mycoplasma, Chlamydia, Legione/la), STJs (Chlamydia), gn1m (±)cocci
(streptococcal infections in patients allergic to penicillin), and B pertussis.
17
18
ADVERSE EFFECTS 1\ IACRO: Gastrointestinal 1\lotilit}' issues, Arrhythmia caused by prolonged QT interval, acute
Cholestatic hepatitis, Rash, eOsinophilia. Increases serum concentration of theophylline, oral
19
anticoagulants. Clarithromycin and er) thromycin inhibit cytochrome P-450.
20
MECHANISM DF RESISTANCE 1\lethylation of 23S rRNA-binding site pre, cnls binding of drug.
21

a
Lock
s
Suspend
8
End Bl ock
Item: 22 of 37 ~. I • M k <:] t> al ~· ~
QIO: 1732 .l. ar Previous Next Lab 'lifllues Notes Calculator


17
A 50-yea r-old man is treated for septic shock with multiple antibiotics and vasopressors. Seven days later ~~AI
18 the patient is alert and oriented, and his tissue perfusion has returned to normal. However, he describes a
19 ter rible sensation of " ring ing in my ea rs."
20
21 Wh ich of the fo llowing accurately describes the therapeutic agent that is most like ly responsible for th is patient's
new complaint?
• 22
• 23 :

• 24
• 25 A. Blocks translocation by binding to the 50S ribosoma l subunit
• 26
B. D-Aia-D-Aia structu ral ana logs which inhibit the formation of peptidog lycan cross-l inks
• 27
C. Efficacy increases when it is administered in conjunction with cilastatin
• 28
• 29 D. Requires oxygen for uptake into the host cel l and is therefore ineffective aga inst anaerobes
o30 E. Results in a disu lfiram-l ike reaction when taken with alcohol
• 31
• 32
• 33
o34
o35
• 36
• 37 ~

6
lock
s
Suspend
0
End Block
Item: 22 of 37 ~. I • M k <:] t> al ~· ~
QIO: 1732 .l. ar Previous Next Lab 'lifllues Notes Calculator


17 The correct answer is D. 42°/o chose this.
18 Gentamicin is an aminoglycoside antibiotic used in the t reatment of gram-negative rod infections.
19 Aminog lycosides are often combined with ~- l actam antibiotics because they act synergistica lly. Aminoglycosides
requ ire ATP-dependent oxygen transport for both cytosol ic and then r ibosomal uptake and thus are ineffective
20
against anaerobes. Aminoglycosides can cause nephrotoxicity and nono ligu r ic acute tubula r necrosis. Also,
21 aminoglycosides can cause ototoxicity, which can manifest as either vestibu lar or cochlear damage. Vestibula r
22 toxicity may resu lt in vertigo, nausea, vomit ing, or ataxia, whereas coch lear damage causes t innitus or hearing
• 23 loss .
Aminoglycoside Gentamicin Tinnitus Antibiotics Nephrotoxicity Gram-negative bacteria Vertigo Ototoxicity Ataxia Acute tubular necrosis Nausea Vomiting
• 24
Hearing loss Synergy Necrosis Vestibular system Oxygen Toxicity Anaerobic organism Cytosol
• 25
• 26 A is not correct. 21% chose this .
• 27 Eryth romycin is a macro lide antibiotic used to treat atypica l pneumonias and upper respiratory infections caused
by Legionella, Chlamydia, Mycoplasma, and Neisseria species. Erythromycin inh ibitis protein synthesis by
• 28
binding to the 235 rRNA of the 50S ribosomal subun it. Erythromycin toxicity causes gastrointestinal discomfort,
• 29 acute cho lestatic hepatitis, eosinophil ia, and skin rash .
o30 Erythromycin Macrolide Antibiotics legionella Chlamydia infection Mycoplasma Protein biosynthesis Protein sos Chlamydia (genus) Hepatitis Cholestasis

•31 Upper respiratory tract infection Rash Ribosomal RNA Protein synthesis Gastrointestinal tract 235 ribosomal RNA

• 32 B is not correct. 17% chose this.


• 33 ~-Lactam antibiotics, such as penicil lin, inh ibit the for mation of peptidoglycan cross-links in the bacterial cel l
o34 wall. An imbalance between ce ll wa ll production and deg radation develops, causing the cel l to rapidly die.
o35
Adverse effects of penicil lin include diarrhea, hypersensitivity, nausea, rash, urticaria, and fungal infection .
Vancomycin is an antibiotic that has a simila r mechanism to pen ici llin; however, it wor ks by bind ing D-Aia-D-Aia
• 36 and consequently prevents peptidog lycan formation . Vancomycin may cause ototoxicity; however, this is a rare
• 37 ~
side effect, especial ly when monotherapy is used . ~

6
lock
s
Suspend
0
End Block
Item: 22 of 37 ~. I • M k <:] t> al ~· ~
QIO: 1732 .l. ar Previous Next Lab 'lifllues Notes Calculator


17 B is not correct. 17% chose this.
18 ~-Lactam antibiotics, such as penicil lin, inh ibit the for mation of peptidoglycan cross-links in the bacterial cel l
19 wall. An imbalance between ce ll wa ll production and deg radation develops, causing the cel l to rapidly die.
Adverse effects of penicil lin include diarrhea, hypersensitivity, nausea, rash, urticaria, and fungal infection .
20
Vancomycin is an antibiotic that has a similar mechanism to pen ici llin; however, it wor ks by bind ing D-Ala-D-Ala
21 and consequently prevents peptidog lycan formation . Vancomycin may cause ototoxicity; however, this is a rare
22 side effect, especial ly when monotherapy is used .
Peptidoglycan Vancomycin Penicillin Urticaria Cell wall Antibiotics Ototoxicity Diarrhea Hypersensitivity Nausea Adverse effect Rash Fungus Side effect
• 23
Combination therapy
• 24
• 25 C is not correct. 11% chose this .
• 26 I mipenem is a ~-lactamase- resistant agent that has a wide spectrum of activity. It is not effective for treating
• 27 infection by methici llin-resistant Staphylococcus aureus, vancomycin-resistant Enterococcus, or some strains of
Pseudomonas. Adverse effects of imipenem include gastrointestina l distress and seizures. I t is administered with
• 28
cilastatin to decrease its inactivation by enzymes in the rena l tubules .
• 29 Imipenem Staphylococcus aureus Enterococcus Pseudomonas Cilastatin Staphylococcus Enzyme Kidney Infection Gastrointestinal tract

o30
E is not correct. 9°/o chose this .
•31
Metronidazo le is a nitroimidazole antibiotic that has antibacterial activity against anaerobes and antiprotozoal
• 32 activity . It is a component of the triple therapy used to treat Helicobacter pylori infection and is considered first-
• 33 line t reatment for Clostridium difficile infection and bacteria l vaginosis. Toxicity can cause nausea, diarrhea,
o34
stomatitis, peripheral neuropathy, and disulfiram- like effects when taken in conjunction with alcohol.
Metronidazole Helicobacter pylori Nitroimidazole Antibiotics Peripheral neuropathy Bacterial vaginosis Clostridium difficile colitis Clostridium difficile (bacteria)
o35
Diarrhea Nausea Antiprotozoal Anaerobic organism Clostridium Stomatitis Alcohol
• 36
• 37 ~

6
lock
s
Suspend
0
End Block
Item: 22 of 37 ~. I • M k <:] t> al ~· ~
QIO: 1732 .l. ar Previous Next Lab 'lifllues Notes Calculator

• side effect, especial ly when monotherapy is used .


17
Peptidoglycan Vancomycin Penicillin Urticaria Cell wall Antibiotics Ototoxicity Diarrhea Hypersensitivity Nausea Adverse effect Rash Fungus Side effect
18
Combination therapy
19
20 C is not correct. 11% chose this.
21 Imipenem is a ~-lactamase-resistant agent that has a wide spectrum of activity. It is not effective for treating
infection by methici llin- resistant Staphylococcus aureus, vancomycin-resistant Enterococcus, or some strains of
22
Pseudomonas. Adverse effects of imipenem include gastrointestina l distress and seizures. I t is administered with
• 23 cilastatin to decrease its inactivation by enzymes in the rena l tubules .
• 24 Imipenem Staphylococcus aureus Enterococcus Pseudomonas Cilastatin Staphylococcus Enzyme Kidney Infection Gastrointestinal tract

• 25
E is not correct. 9°/o chose this .
• 26 Metronidazo le is a nitroimidazole antibiotic that has antibacterial activity against anae robes and antiprotozoal
• 27 activity . It is a component of the t riple therapy used to treat Helicobacter pylori infection and is conside red fi rst-
• 28 line t reatment fo r Clostridium difficile infection and bacte ria l vaginosis. Toxicity can cause nausea, diarrhea,
stomatitis, pe ripheral neuropathy, and disulfiram- like effects when taken in conjunction with alcohol.
• 29
Metronidazole Helicobacter pylori Nitroimidazole Antibiotics Peripheral neuropathy Bacterial vaginosis Clostridium difficile colitis Clostridium difficile (bacteria)
o30
Diarrhea Nausea Antiprotozoal Anaerobic organism Clostridium Stomatitis Alcohol
•31
• 32
• 33 Bottom Line:
o34 Aminog lycosides can cause nephrotoxicity and nonoliguric acute tubu lar necrosis; they are also ototoxic and
can cause either vestibu lar or cochlear damage.
o35
Acute tubular necrosis Nephrotoxicity Ototoxicity Aminoglycoside Necrosis
• 36
• 37 ~

6
lock
s
Suspend
0
End Block
Item: 22 of 3 7 ~ 1 • M k -<:J 1>- Jil ~· !:';-~
QIO: 1732 ..L ar Pre v ious Next Labfli!llues Not es Calcula t o r
A A

17
FA17 p 187.2
18
Aminoglycosides G entamicin, ;'\eomycin, \ m ikacin, "\lean" (aminoglycoside) C~ATS ca ~~OT
19
Tobramycin, Streptomrcin. kill anaerobes.
20
MECHANISM Bactericidal; irreversible inhibition of initiation
21
complex through binding of the 30S subunit.
22 Can cause misreading of mR1 . Also block
• 23 translocation. Require 0 2 for uptake; therefore
• 24 ineffecti,·e against anaerobes.
• 25 CliNICAl USE Severe gram 8 rod infections. >nergistic with
• 26 ~-lactam antibiotics.
Neomycin for bowel surgerr.
• 27
• 28
ADVERSE EFFECTS 1'\ephrotoxicity, Neuromuscular blockade,
O totoxicity (especially when used with loop
• 29
diuretics). Teratogen.
o30
MECHANISMOF RESISTANCE Bacterial transferase enzymes inaet ivate the
• 31
drug by acetylatio n, phosphorylation, or
• 32 adenylation .
• 33
• 34 FA17 p 187.1
• 35 Protein synthesis inhibitors Specifically target smaller bacterial ribosome
• 36 (70S, made of 30S and 50S subunits), leaving
linezolid Ribosomal A&P site
• 37 (505) r-·n•
' human ribosome (80S) unaffected.

a
Lock
s
Suspend
8
End Bl ock
Item: 22 of 3 7 ~ 1 • M k -<:J 1>- Jil ~· !:';-~
QIO: 1732 ..L ar Pre v ious Next Labfli!llues Not es Calcula t o r
A A

17
FA17 p 187.1
18
Protein synthesis inhibitors Specifically target smaller bacterial ribosome
19
linezotid Ribosomal A&P site {70S, made of 30S and 50S subunits), leaving
20 (50S) r • .., human ribosome (80S) unaffected.
PA
21
305 inhibitors
22 ~~
; .;....,
..;R•NA
_..;m A = ,\minogl~cosides [bactericidal]
• 23 J~ '1 = Tetracyclines (bacteriostatic]
• 24 Initiator tRNA Initiation
complex
1~ (also
ArmnoglycoSides (30SI
cause mtSreadtng 50S inhibitors
• 25 formation of mRNAI
Tetracycbnes
C = C hloramphenicol, C lindamycin
• 26 PA
(3051 [baclerioslatic]
• 27 1!. = Erythromycin (macrolidcs) [bactcriostaticl
• 28 L = Linezolid l,·ariable]
• 29 "13uy \T 30, CCEL (sell) at 50."
o30
• 31
• 32
• 33
• 34 Translocation
• 35
• 36
• 37 • Macrobdes (eq, efVIhrornytin) (50S)

a
Lock
s
Suspend
8
End Bl ock
Item: 23 of 3 7 ~ 1 • M k -<:J 1>- Jil ~· !:';-~
QIO: 1592 ..L ar Pre v ious Next Lab fli!ltues Notes Calcula t o r

IAA]
A A

17
A 7-year-old boy with a history of asthma presents to his physician because of persistent asthma symptoms
18 despite treatment with f luticasone daily and the use of an albuterol rescue inhaler. His physician decides to
19 add a med ication to the patient's daily asthma treatment regimen that will block the adverse action of
leukotrienes w ithout affecting their production.
20
21
Which of the following medications does this describe?
22
• 23 :

• 24
A. Cortisol
• 25 B. Cromolyn
• 26 C. Montelukast
• 27
D. Theophylline
• 28
• 29
E. Zileuton

o30
• 31
• 32
• 33
• 34
• 35
• 36
• 37 •

a
Lock
s
Suspend
8
End Bl ock
Item: 23 of 37 ~. , . M k <:] t> al ~· ~
QIO: 1592 .l. ar Previous Next lab 'lifllues Notes Calculator

17
The correct answer is C. 72°/o chose this.
18 Asthma is a disease characterized by bronchial hyperreactivity resulting in reversible bronchoconstriction.
19 Bronchoconstriction is mediated by inflammatory processes, as well as parasympathetic tone. Asthma therapy is
20
therefore directed at those two processes. Leukotrienes are signaling molecules that cause bronchoconstr iction .
Montelukast and zi leuton are asthma drugs that target the leukotriene pathway. Montelukast is a leukotr iene
21
receptor inhibitor that does not inh ibit the production of leukotr ienes. I t is a particu larly usefu l drug in the
22 treatment of aspi r in-induced asthma.
Bronchial hyperresponsiveness Zileuton Montelukast leukotriene Aspirin-induced asthma Bronchoconstriction Asthma Parasympathetic nervous system Bronchiole
23
• 24 Bronchus lipid signaling Receptor antagonist

• 25 A is not correct. 3°/o chose this.


• 26 Cortisol inhibits phospholipase A 2, the enzyme that converts phosphol ipid A into arachidon ic acid. Arachidon ic
• 27 acid is a precu rsor to both prostagland ins and leukotrienes, which are med iators of inflammation .
Arachidonic acid Cortisol Phospholipid Enzyme Prostaglandin leukotriene Phospholipase A2 Phospholipase Inflammation
• 28
• 29 B is not correct. 6°/o chose this .
o30 Cromolyn prevents release of mediators from mast cel ls that cause bronchoconstriction in inflammatory states .
• 31
It is effective for asthma prophylaxis but not in the setting of acute asthma attack .
Cromoglicic acid Bronchoconstriction Asthma Mast cell Preventive healthcare
• 32

• 33 D is not correct. 5°/o chose this .


o34
Theophyll ine causes bronchodilation by inhibiting phosphod iesterase, thereby increasing the concentration of
cAMP. cAMP promotes bronchodi lation .
o35 Theophylline Bronchodilator Cyclic adenosine monophosphate Phosphodiesterase
• 36
E is not correct. 14% chose this .
• 37
... -·· o I .. ' I - I • 'I I I I • ' I

6
lock
s
Suspend
0
End Block
Item: 23 of 37 ~. , . M k <:] t> al ~· ~
QIO: 1592 .l. ar Previous Next lab 'lifllues Notes Calculator
I' y y y

17
18 A is not correct. 3°/o chose this.
19
Cortisol inh ibits phospholipase A 2, the enzyme that converts phosphol ipid A into arach idon ic acid . Arach idon ic
acid is a precu rsor to both prost ag land ins and leukotrienes, which are mediators of infl ammation.
20 Arachidonic acid Cortisol Phospholipid Enzyme Prostaglandin leukotriene Phospholipase A2 Phospholipase Inflammation
21
B is not correct. 6°/o chose this.
22
Cromolyn prevent s release of med iators from mast cel ls t hat cause bronchoconstriction in inflammatory states .
23 It is effect ive for asthma prophylaxis but not in the setting of acute asthma attack .
• 24 Cromoglicic acid Bronchoconstriction Asthma Mast cell Preventive healthcare

• 25
D is not correct. 5°/o chose this .
• 26 Theophylline causes bronchodilation by inhibiting phosphodiesterase, t hereby increasing the concent ration of
• 27 cAM P. cAM P promotes bronchodi lation .
Theophylline Bronchodilator Cyclic adenosine monophosphate Phosphodiesterase
• 28
• 29 E is not correct. 14% chose this .
o30 Zileut on impa irs the leukot riene synthesis enzyme 5- lipoxygenase, t hereby blocking the conversion of
• 31 arach idonic acid to leukotr ienes .
Zileuton Arachidonic acid leukotriene Enzyme Arachidonate 5-lipoxygenase
• 32
• 33
o3 4 Bottom Line:
o35 Montelukast is a leukotriene receptor blocker and does not affect the production of leukot rienes .
• 36 Montelukast leukotriene Receptor (biochemistry)

• 37 ~

6
lock
s
Suspend
0
End Block
Item: 23 of 3 7 ~ 1 • M k -<:J 1>- Jil ~· !:';-~
QIO: 1592 ..L ar Pre v ious Next Lab fli!ltues Notes Calcula t o r
A A

17
18 FA17 p649.1

19 Asthma drugs Bronchoconstriction is mediated b) (I) inOammatory processes and (2) parasympathetic tone;
20 therapy is directed at these 2 pathways.
21 ~-agonists Albuterol-rclaxes bronchial smooth muscle (~hort acting f3z-agonist). Used during acute
exacerbation.
22
23
Salmeterol, formotero l- long-acting agents for prophylaxis. ,\d\'erse effects are tremor and
arrln thmia.
• 24
• 25
Inhaled Fluticasone, budesonide -inhibit the synthesis of ,·irtuallr all cytokines. lnacti,Cite 'F'-KB, the
corticosteroids transcription factor that induces production of T IF-a and other inflammatory agents. 1st-line
• 26
therapy for chron ic asthma. \'lay cause oral thrush .
• 27
Muscarinic Tiotropium, ipratropium-compet itively block muscarinic receptors, pre\'enting
• 28 antagonists bronchoconstriction. Also used for COPD. Tiotropium is long acting.
• 29
Antileukotrienes Montelukast, zafirlukast- block leukotricnc Exposure to ant1gen
o30 receptors (CysLTl). Especially good for (dust. pollen, etc}
• 31 aspirin-induced and exercise-induced asthma .
• 32 Zileuton -5-lipoxygenase pathway inh ibitor. ~ Avoidance
Blocks conversion of arachidonic acid to
• 33
leukotrienes. Hepatotoxic.
• 34
Antigen and lgE ~ Omahzumab
Anti-lgE monoclonal Omalizumab-binds mostly unbound serum on mast cells
• 35
therapy lgE and blocks binding to FceRI. Used in
• 36
allergic asthma with t lgE levels resistant to
~ Steroids
• 37 inhaled steroids and long-acting B,-agonists.

a
Lock
s
Suspend
8
End Bl ock
Item: 23 of 37 ~. , . M k <:] t> al ~· ~
QIO: 1592 .l. ar Previous Next lab 'lifllues Notes Calculator

• •
17
18 FA17 p638.1
19 Obstructive lung Obstruction of air fl ow - air trapping in lungs. Airways close prematurely at high lung volumes
20 diseases - t RV and t FRC, t TLC. PFTs: U FEV 1, l FVC - l FEV 1/FVC ratio (hallmark),
21 V/Q mismatch. C hronic, hypoxic pulmonary vasoconstriction can lead to cor pulmonale. Chronic
22 obstructive pulmonary disease (COPD) includes chronic bronchitis and emphysema.
TYPE PRESENTATION PATHOLOGY OTHER
23
Chronic bronchitis Findings: wheezing, crackles, Hypertrophy and hyperplasia Diagnostic criteria: productive
• 24
("blue bloater") cyanosis (hypoxemia due of mucus-secreting glands cough for> 3 months in a
• 25
to shunting), dyspnea, C02 in bronchi - Reid index year for> 2 consecutive years.
• 26 retention, zopolyc)themia. (thickness of mucosal gland
• 27 layer to thickness of wall
• 28 between epithelium and
• 29
cartilage) > 50%.

o30
Emphysema ("pink Centriacinar-associated with Enlargement of air spaces CXR: t AP diameter, flattened
puffer") smoking rJ IE). Frequently in l recoil, t compliance, diaphragm, t lung field
• 31
upper lobes (smoke rises up). l DLCO from destruction of lucency.
• 32 Panacinar-associated with alveolar walls (arrow in ~). Barrel-shaped chest [!].
• 33 a 1-antitrypsin deficiency. t elastase activity - t loss Exhalation through pursed lips
o34 Frequently in lower lobes. of elastic fibers - t lung to increase airway pressure
o35
compliance. and prevent airway collapse.
• 36 Asthma Findings: cough, wheezing, Branchial hypcrrcsponsi,·cncss Aspirin-i nduced asthma: COX
tachypnea, dyspnea, - reversible inhibition - leukotriene
• 37 • ' . . ... ... ·' .. •

6
lock
s
Suspend
0
End Block
Item: 23 of 3 7 ~ 1 • M k -<:J 1>- Jil ~· !:';-~
QIO: 1592 ..L ar Pre v ious Next Lab fli!ltues Notes Calcula t o r
A A

17
FA17 p 455.1
18
Arachidonic acid pathway
19
20 MEMBRANE
21 PHOSPHOLIPIDS Glucocorllcoids
(Corticosteroids)
22
Betamethasone Methylpredntsolone
23 Phospholipase A 2 Cortisone Predr~solone
Dexamethasone Predr~sone
• 24 HydrOCortisone Tnamcinolone
• 25
ENDOPEROXIDE SYNntESIS
• 26
NF-KB I - 0 - I~ (cyclooxygenase)
• 27
• 28
( Z1leuton
'--------<~
)
Arachidonic acid

COX-2
f - e -i COX·2 ONLY
Celecoxib J
COX-1, COX-2

Aspirin (irreversible)
• 29
5-Lipoxygenase ~ - Other NSAIDs(reversible)
Diclofenac Ketorolac
COX-1 ~
o30
Ibuprofen Naproxen
• 31 Indomethacin
• 32
Montelukast
5-HPETE Cyclic endoperoxides

\
• 33 Zafirlukast
I \ ~
I
• 34
Leukotrienes Prostacyclin Prostaglandins Thromboxane

r
~

• 35
LTC 4 LTD4 LTE4 LTB4 PGiz PGE, PGEz PGFza TXAz
• 36
T bronchial tone T neutrophil 1 platelet 1 vascular I utenne T utenne I platelet
• 37 chemotaxis aggregat1on tone tone tone aggregation

a
Lock
s
Suspend
8
End Bl ock
17 Triamcinolone
18
19 ENDOPEROXlDE SYNTHESIS
Arachidonic acid NF-KB -0- IKe (cyclooxygenase)
20
21
(...__zt_te_ut_on_ _ _-<.)
~
I f
COX- 2 Gr---1(
COX-2 ONLY
Celecoxib )
COX-1, COX-2

Aspirin (irreversible)
22
23
UUKOTRIENE
RECEPTOR
5-lipoxygenase ~ Other NSAIDs (reversible)
Oiclotenac Ketorolac
• 24 ANTAGONISTS COX-1 e- - Ibuprofen
Indomethacin
Naproxen

• 25
Montelukast
5-HPETE Cyclic endoperoxides
• 26
• 27
• 28
Zafirlukast

1
I
l eukotrienes
\
Prostacyclin
\
Prostaglandins
~
Thromboxane

LTC4 LTD4 LTE4 LTB4 PGJ2 PGE1 PGE 2 PGFzo T XA2


• 29
T bronchial tone T neutrophil I platelet I vascular T utenne T uterine T platelet
o30 chemotax1s aggregat1on tone tone tone aggregation
• 31 I vascular tone T vascular tone

• 32 ( Epoprostenol ) ( Alprostad1l ) ( Dinoprostone ) ( Carboprost )

• 33
• 34 LTB4 is a neutrophil chemotactic agent. Nculrophils arri,·e "B-f" others.

I
o35 PG I2 inhibits platelet aggregation and promotes Platelet-Gathering Inhibitor.
• 36
vasodilation.

• 37 •

a
Lock
s
Su spend
8
End Bl ock
Item: 24 of 37 ~. I • M k <:] t> al ~· ~
QIO: 2498 .l. ar Previous Next lab 'lifllues Notes Calculator


17 The correct answer is B. 70°/o chose this.
18 This cl inical picture is high ly suggestive of restr ictive lung disease; although the FEV1 : FVC ratio is near norma l
19 levels, both the FEV1 and FVC are markedly reduced . Amioda rone is an antiarrhythmic agent that is known to
20
cause pulmonary fibrosis, a restrictive lung disease. Additional adverse eff ects include interstitial pneumonitis,
photosensitivity, thyroid disorders, and gastrointestinal distu r bances.
21 Antiarrhythmic agent Restrictive lung disease Amiodarone Pulmonary fibrosis Interstitial lung disease Photosensitivity Fibrosis Respiratory disease lung Thyroid
22
A is not correct. 6°/o chose this.
23
Asthma is a cause of chronic obstructive pulmonary disease. Asthma-induced abnor mal ities on pu lmonary
24
function testing would not fit a restrictive pathology profile .
• 25 Asthma Pulmonary function testing Pathology

• 26
C is not correct. 7°/o chose this .
• 27
Diltiazem is an antiarrhythmic that is sometimes used in intravenous form to treat atr ial f ibr il lation. It
• 28 infrequently causes hypotension or bradyarr hythmias but is not known to cause pulmona ry fibrosis .
• 29 Oiltiazem Atrial fibrillation Hypotension Pulmonary fibrosis Bradycardia Intravenous therapy Antiarrhythmic agent Fibrosis Atrium (heart)

o30 D is not correct. 8°/o chose this.


• 31 Sota lol works both by nonselectively antagon izing ~-recepto rs and by pro longing action potentials. It is used to
• 32 treat ventricu lar and supraventr icular arr hythmias in chi ldren and life-threatening ventricu lar ar rhythmias in
adults. Sotalo l can sometimes cause torsades des pointes when taken at higher doses. However, it does not
• 33
cause pulmonary fibrosis.
o34 Sotalol Pulmonary fibrosis Action potential Cardiac arrhythmia

o35
E is not correct. 9°/o chose this .
• 36
Tobacco is a known risk factor for ch ron ic obstructive pu lmonary disease (COPD). COPD presents with a reduced
• 37 ~ FFV.: FVC: r "tio. tvnir:" llv <1~0% . ThP.rP. i~ " l ~o " r:onvP.x inw"rrl ~loninn of thP. f low-volt JmP. r:1JrVP. on nJ Jimon"rv

6
lock
s
Suspend
0
End Block
Item: 24 of 37 ~. I • M k <:] t> al ~· ~
QIO: 2498 .l. ar Previous Next lab 'lifllues Notes Calculator

17
.. ... - -- - -- -
~ . -- - - - -
function testing wou ld not fit a restrictive pathology profile.
- -- - -- - - - -- - .. .
18 Asthma Pulmonary function testing Pathology

19 C is not correct. 7°/o chose this.


20 Dilt iazem is an antiarr hythmic t hat is sometimes used in intravenous fo rm to t reat atr ia l f ibr illation . I t
21 infrequently causes hypotension or bradyarr hythmias but is not known to cause pu lmona ry fibrosis.
Oiltiazem Atrial fibrillation Hypotension Pulmonary fibrosis Bradycardia Intravenous therapy Antiarrhythmic agent Fibrosis Atrium (heart)
22
23 D is not correct. 8°/o chose this.
24 Sota lol wo rks both by nonselectively antagon izi ng ~ - recepto rs and by pro longing action potentials. It is used to
• 25 t reat vent ricu lar and supraventr icu lar arr hythmias in chi ldren and life-threatening vent ricu lar ar rhythmias in
adu lts. Sota lol can somet imes cause torsades des point es when taken at higher doses. Howeve r, it does not
• 26
cause pu lmona ry fibrosis .
• 27 Sotalol Pulmonary fibrosis Action potential Cardiac arrhythmia

• 28
E is not correct. 9°/o chose this .
• 29
Tobacco is a known risk factor for ch ron ic obstructive pu lmona ry disease (COPD) . CO PD present s with a reduced
o30 FEV1 : FVC ratio, t ypica lly <80%. The re is also a convex inwa rd sloping of t he flow-vo lume curve on pu lmonary
• 31 function testing .
Chronic obstructive pulmonary disease Pulmonary function testing Tobacco Respiratory disease Pulmonology
• 32
• 33
o34 Bottom Line:
o35 Pulmonary fibrosis is a key adverse effect of am ioda rone .
• 36 Amiodarone Pulmonary fibrosis Fibrosis Adverse effect

• 37 ~

6
lock
s
Suspend
0
End Block
Item: 24 of 3 7 ~ 1 • M k -<:J 1>- Jil ~· !:';-~
QIO: 2498 ..L ar Pre v ious Next Lab fli!ltues Not es Calcula t o r
A A

17 FA17 p639.1
18 Restrictive lung Restricted lung expansion causes ! lung volumes (! FVC and TLC). PFTs: FEV 1/FVC ratio~ 80%.
19 diseases Patient presents with short, shallow breaths.
20 Types:
21 Poor breathing mechanics (extrapulmonary, peripheral hypo,·entilation, normal A-a gradient):
22 Poor muscular effort-polio, myasthenia gra\·is, Cuillain-Barre s~·ndrome
Poor structural apparatus-scoliosis, morbid obesity
23
Interstitial lung diseases (pulmonar) ! diffusing capaci t~ . t A-a gradient):
24
• Pneumoconioses (eg, coal \\Or~ers' pneumoconiosis, silicosis, asbestosis)
• 25 • Sarcoidosis: bilateral hilar lymphadenopathy, noncaseating granuloma; t ACE and Ca 2+
• 26 Idiopathic pulmonary fibrosis (repeated cycles of lung injury and wound healing with
• 27 t collagen deposition, ''honeycomb" lung appearance and digital clubbing)
• 28
• Granulomatosis with polyangiitis (Wegener)
• Pulmonary Langcrhans cel l histiocytosis (eosinoph ilic granuloma)
• 29
• Hypersensitivity pncumonit is
o30 • Drug toxicity (bleomycin, busulfan, amiodarone, methotrexate)
• 31
Hypersensitivity pneumonitis- mixed type III/ IV hypersensitivity reaction to environmental
• 32
antigen. Causes dyspnea, cough, chest tightness, headache. Often seen in farmers and those
• 33 exposed to birds. Reversible in early stages if st imulus is a,·oided.
• 34
• 35 FA17 p 309.2
• 36 Antiarrhythmics- Amiodarone, Tbutilide, Dofetilide, Sotalol. AIDS.
• 37 potassium channel

a
Lock
s
Suspend
8
End Bl ock
Item: 24 of 3 7 ~ 1 • M k -<:J 1>- Jil ~· !:';-~
QIO: 2498 ..L ar Pre v ious Next Lab fli!ltues Not es Calcula t o r
A A

17
FA17 p 309.2
18
19 Antiarrhythmics- Amiodarone, l butilide, Dofetilide, Sotalol. AIDS.
potassium channel
20
blockers (class Ill)
21
MECHANISM t AP duration, t ERP, t QT interval.
22
CLINICAL USE Atrial fibrillation , atrial Rutter; ventricular
23
tachycardia (amiodarone, sotalol).
24
ADVERSE EFFECTS Sotalol-torsades de pointes, e\cessive ~ Remember to check PFTs, LFTs, and TFTs \\hen
• 25
blockade. using amiodarone.
• 26 lbutilide-torsades de pointes. Amiodarone is lipophilic and has class I, II, Il l,
• 27 Amiodarone-pulmonary fibrosis, and IV effects.
• 28 hepatotoxicity, hypothyroidism/
• 29
hyperthyroidism (amiodarone is 40% iodine by
weight), acts as hapten (corneal deposits, blue/
o30 gray skin deposits resulting in photodermatitis),
• 31 neurologic effects, constipation, cardiovascular
• 32 effects (bradycardia, heart block, H F).
• 33 Class Ill
• 34 OmV
• 35 Markedly prolonged
repolanzabOn IIKJ
• 36
• 37 •

a
Lock
s
Suspend
8
End Bl ock
Item: 24 of 3 7 ~ 1 • M k -<:J 1>- Jil ~· !:';-~
QIO: 2498 ..L ar Pre v ious Next Lab fli!ltues Not es Calcula t o r
A A

17 Class Ill
18
19
20
21
22
23 Cell action potential

24
• 25 FA17 p 637.2

• 26 Flow volume loops Obstructive lung volumes> normal (t TLC, t F'RC, f RV); restricti\'e lung volumes< normal. In
• 27 obstructive, FEV 1 is more dramatically reduced compared with FVC - decreased FEV1/F'VC
ratio. In restrictive, FVC is more reduced or close to same compared with F'E 1 - increased or
• 28
normal F'EV1/FVC ralio .
• 29
OBSTRUCTIVE NORMAL
o30
Loop shifts to the left Loop shifts to the right
• 31 8
88 8
·= • •
•' ''''
• 32 ~
I

4~
·a. I I

• 33 ~4 I
I
'' 4
I
I
I
I
''
• ''
• 34
~ '•
• 35
• 36
d
~
... 8 6 ~
4 2 0 8 6 4 2 0 h-6~ 0
Votume(L)

• 37 • c:4 ~. ''
4 \ I 4 ' .. ·''
a
Lock
s
Suspend
8
End Bl ock
17
18 -8SmV --..J

19 Cell acti(J(l potential

20
FA17 p 637.2
21
22 Flow volume loops O bstructi,·e lung volumes> normal (t T LC, t FRC, t RV); restrictive lung volumes< normal. In
23
obstructive, FE\11 is more dramatically reduced compared with FVC- decreased FEVrfF C
ratio. In restrictive, F C is more reduced or close to same compared with FE\11 - increased or
24
normal FEV 1/FVC ratio.
• 25
OBSTRUCTIVE NORMAl
• 26
loop shifts to the left loop shifts to the right
• 27 8 8
• ••
• 28
••• •• ''
I
I
••
+

• 29
''' •• 4 '''
•• '
o30 ' '• ''
• 31
• 32
0 h 4 0 8
'
6
'
4

• 33 4 4
• 34
• 35 8 8
---TlC__,
• 36
• 37 •

a
Lock
s
Suspend
8
End Block
Item: 25 of 37 ~. , . M k <:] t> al ~· ~
QIO: 3443 .l. ar Previous Next lab 'lifllues Notes Calculator


17
A 48-yea r-old immig rant from India complains of ch ronic cough, fatigue, and weight loss. Fo r the past 2 ~~AI
18 months, she has been producing increasing amounts of non-foul-smelling, g reen-ye llow sputum, and has had
19 occasional hemoptysis. She also casual ly mentions that she often has profuse night sweats. A physical
examination revea ls that she is feb rile at 38. 7°C ( 101. 7°F). The X- ray of t he chest show n in the image
20
demonstrates left-sided pleura l effusions and ipsilatera l hilar en large ment. The physician sends the sputum for
21 microbiologic diagnosis and initiates t reatment. One week late r, the patient deve lops a burn ing sensation in her
22 hands and feet.
23
24
0 25
0
26
0 27
0
28
0 29
o30
0
31
0
32
0
33
0 34
0 35
0
36
0 37 ~

6
lock
s
Suspend
0
End Block
Item: 25 of 3 7 ~ 1 • M k -<:J 1>- Jil ~· !:';-~
QIO: 3443 ..L ar Pre v ious Next Lab fli!ltues Not es Calcula t o r
A
-- . - --- - ------- - - - - - - - - - - - - - - - --- - - - - - - -- ---- . .
A

17 microbiologic diagnosis and initiates t reatment . One week later, the patient develops a burn ing sensation in her
18 hands and feet.
19
20
21
22
23
24
• 25

• 26

• 27
• 28
• 29
o30
• 31
• 32

• 33
• 34 Image courtesy of James Heilman, MD

o35
• 36
Wh ich of the following supplements can be administered to minimize these symptoms?

• 37 :

a
Lock
s
Suspend
8
End Bl ock
17
18
19
20
21
22
23
24
• 25
• 26
I mage courtesy of James Heilman, MD
• 27
• 28 Which of the following supplements can be admin istered to minimize these symptoms?
• 29
:
o30 A. Vitamin A
• 31
B. Vitamin 812
• 32
• 33 C. Vitamin 8 2
• 34 D. Vitamin 8 6
o35
E. Vitamin K
• 36
• 37 •

a
Lock
s
Suspend
8
End Block
Item: 25 of 37 ~. , . M k <:] t> al ~· ~
QIO: 3443 .l. ar Previous Next lab 'lifllues Notes Calculator

• The correct answer is D. 71 °/o chose this.


17
18 The pleura l effusion evident in the x-ray (circle) ind icates pulmonary (Mycobacterium tuberculosis) tubercu losis
infection . The initial treatment for tuberculosis involves the four- drug regimen of isoniazid, r ifampin,
19
pyrazinamide, and ethambutol. Although these drugs have several adverse effects, ison iazid is particula r ly
20 known for causing neurotoxicity by inducing vitam in B6 deficiency. Thus, these symptoms can be minimized if
21 the patient is given pyridoxine, or vitamin B6 .
Pleural effusion Isoniazid Pyrazinamide Ethambutol Mycobacterium tuberculosis Rifampicin Tuberculosis Pyridoxine Pleural cavity Vitamin 812 Mycobacterium
22
Neurotoxicity X-ray 8 vitamins Vitamin Infection
23
24 A is not correct. 5°/o chose this.
25 Vitamin A deficiency resu lts in night bl indness, dry skin, and impai red immune response . It is not caused by a
• 26 tubercu losis drug .
Vitamin A Tuberculosis Nyctalopia Vitamin Vitamin A deficiency Immune system Visual impairment Xeroderma
• 27
• 28 B is not correct. 14% chose this.
• 29 Vitamin B12 deficiency can cause neurologic symptoms, such as optic neu ropathy and paresthesias, but
tubercu losis drugs do not cause a deficiency of v itamin B12.
o30 Tuberculosis Vitamin 812 8 vitamins Paresthesia Peripheral neuropathy Optic neuropathy Vitamin Neurology
• 31
C is not correct. 6 °/o chose t his .
• 32
The drugs used to t reat tuberculosis do not cause a vitamin B2 deficiency. Furthermore, deficiency of vitamin B2
• 33
causes angular stomatitis, cheilosis, and cornea l vascu larization, which are not seen in this patient .
o34 Tuberculosis Vitamin 812 8 vitamins Stomatitis Vitamin Angular cheilitis

o35
E is not correct. 4 °/o chose t his .
• 36
Vitamin K deficiency can resu lt in prolonged prothrombin time and activated partial thromboplastin time
• 37 ~ because v itamin K is necessary for synthesis of clottina factors II, VII, I X, and X and oroteins C and S. However, ~

6
lock
s
Suspend
0
End Block
Item: 25 of 37 ~. , . M k <:] t> al ~· ~
QIO: 3443 .l. ar Previous Next lab 'lifllues Notes Calculator


17 Vitamin A deficiency resu lts in night bl indness, dry skin, and impai red immune response . It is not caused by a
18 tuberculosis drug .
Vitamin A Tuberculosis Nyctalopia Vitamin Vitamin A deficiency Immune system Visual impairment Xeroderma
19
20 B is not correct. 14% chose this.
21 Vitamin B12 deficiency can cause neu rologic symptoms, such as optic neu ropathy and paresthesias, but
22
tuberculosis drugs do not cause a deficiency of v itamin B 12.
Tuberculosis Vitamin 812 8 vitamins Paresthesia Peripheral neuropathy Optic neuropathy Vitamin Neurology
23
24 C is not correct . 6°/o chose this.
25
The drugs used to t reat tube rculosis do not cause a vita min B2 deficiency. Furthe rmore, deficiency of vitamin B2
causes angular sto matitis, cheilosis, and cornea l vascu larization, which are not seen in this patient .
• 26 Tuberculosis Vitamin 812 8 vitamins Stomatitis Vitamin Angular cheilitis
• 27
E is not correct. 4°/o chose this .
• 28
Vitamin K deficiency can resu lt in prolonged proth rombin t ime and activated partial t hro mboplastin time
• 29
because v itamin K is necessa ry for synthesis of clotting factors II, VII, I X, and X and proteins C and S. Howeve r,
o30 deficiency is not caused by ison iazid or any othe r dr ug used to treat tuberculosis.
• 31 Isoniazid Prothrombin time Vitamin K Partial thromboplastin time Coagulation Tuberculosis Thrombin Vitamin Protein c Vitamin K deficiency Protein

• 32
• 33 Bottom Line :
o34
Wh ile treating tube rculosis with isoniazid, rifampin, pyrazinamide, and ethambuto l, vitamin B6 supplementation
o35 should be included to prevent neu rotoxicity.
• 36 Isoniazid Pyrazinamide Ethambutol Rifampicin Tuberculosis Vitamin 812 Neurotoxicity 8 vitamins

• 37 ~

6
lock
s
Suspend
0
End Block
Item: 25 of 3 7 ~ 1 • M k -<:J 1>- Jil ~· !:';-~
QIO: 3443 ..L ar Pre v ious Next Lab fli!ltues Not es Calcula t o r
A A

17
FA17 p 193.1
18
Isoniazid
19
MECHANISM l synthesis of mycolic acids. Bacterial catalase-
20
peroxidase (encoded by KatG) needed to
21 com·ert INH to acti,·e metabolite.
22
CLINICAL USE i\lycobacterium tuberculosis. The only agent Different INH half-liYes in fast vs slow
23 used as solo prophylaxis against TB. Also used acetdators.
24 as monotherapy for latent T B.
25 ADVERSE EFFECTS Hepatotoxicity, P-450 inhibition, drug-induced 1~1 I Injures ~eurons and I lepatocytes.
• 26 SLE, anion gap metabolic acidosis,' itamin
• 27
8 6 deficiency (peripheral neuropathy,
sideroblastic anem ia). dminister with
• 28
pyridoxine (8 6).
• 29
MECHANISM OF RESISTANCE Mutations leading to underexpression of KatC.
o30
• 31
FA17 p 63.4
• 32 Vitamin 8 6 (pyridoxine)
• 33
FUNCTION Converted to pyridoxal phosphate (PLP), a cofactor used in transamination (eg, ALT and AST),
• 34 decarboxylation reactions, glycogen phosphorylase. Synthesis of cystathionine, heme, niacin,
o35 histamine, and neurotransmitters including serotonin, epinephrine, norepinephrine ( 1£ ),
• 36 dopamine, and GABA.
• 37 DEFICIENCY Convulsions, hyperirritability, peripheral neuropathy (deficiency inducible by isoniazid and oral

a
Lock
s
Suspend
8
End Bl ock
Item: 25 of 3 7 ~ 1 • M k -<:J 1>- Jil ~· !:';-~
QIO: 3443 ..L ar Pre v ious Next Lab fli!ltues Not es Calcula t o r
A A

17
FA17 p63.4
18 Vitamin 8 6 (pyridoxine)
19
FUNCTION Converted to pyridoxal phosph<Jte (PLP), a cofactor used in transamination (eg, ALT and AST),
20 decarboxylation reactions, glycogen phosphorylase. Synthesis of cystathionine, heme, niacin,
21 histamine, and neurotransmitters including serotonin, epinephrine, norepinephrine (NE),
22 dopamine, and GABA.
23 DHICIENCY Convulsions, hr perirritability, peripheral neuropathy (deficiency inducible by isoniazid and oral
24 contracepti,·es), sideroblastic anemias due to impaired hemoglobin synthesis and iron excess.
25
• 26 FA17 p 136.1

• 27 Primary and secondary tuberculosis

• 28 _, Mycobacterium
......... PPD Cit) if current infection or past exposure.
• 29 H1larnodes
r ·'·' tuberculosis PPD 8 if no infection and in sarcoidosis or
t I-I IV infection (especially with low CD4+ cell
o30 Ghon
complex count}.
• 31
lnterferon-y release assay (IGRA) has fewer false
• 32 Primary tuberculosis positives from BCG vaccination.
• 33 > 90X <10" Caseating granulomas rJ with central necrosis
• 34 Heal1ng by hbrOSis ProgresSive primary tuberculosis (upper left) and Langhans giant cells (arrow)
arc characteristic of zotuberculosis.
Calc1fication (AIDS. malnutntlon)
I
(tUberCulin @)
l
I
o35
• 36 ReacllYallOrl Progres~~ve
lung d1sease
2' tubetculosis
• 37 Bacteremia

a
Lock
s
Suspend
8
End Bl ock
Item: 25 of 3 7 ~ 1 • Ma r k -<:J I> ~ £!1}>'
• !!":-~
QIO: 3443 ..L Pre v ious Next Lab lues Not es Calcula t o r

_, .. Mycobacterium
A
._..,, PPD (D if current infection or past exposure. A

17
Holarnodes
+ •'•' tuberculosis PPD 8 if no infection and in sarcoidosis or
18

Ghon~
HIV infection (especially with low C D4+ cell
19
compltx Ghon +focus --..,. count).
20 (usually 1111d/ lnterferon-y release assay (lGRA) has fc\\ cr false
lower lobes! Primary tuberculosis
21 positi,·es from BC G ' -accination.
22 >90% < 10% Caseati ng granulom as rJ with central necrosis
Healing by llbfOSIS Progr~ P'ltnaiY tuberclAosls (upper left) and Langhans giant cells (arrow)
23 utoflutoon IAJOS. maklutnllOII)
(tubefcWn @ )
I arc characteristic of 2° tuberculosis.
1
I
24
Rtaci!Vatoon ProgressiVe
25
lungdtsease
2' tu~ulosis
• 26

• 27
flbroc.lseouS
caVItary tes.on
J
• 28
(usually upper
lobes) 7 -Miliary r ~lnges

• 29 •'S, ~losis ~ Vertellrae

o30 !
Localized destructive disease
t .,
J .-' (!'ott disease)

• 31
CaVIty
Caseation
• 32 Caseation
• 33
• 34
"?- Joonts and
o35 > tong bones
Cl
• 36
• 37

a
Lock
s
Suspend
8
End Bl ock
Item: 26 of 3 7 ~ 1 • M k -<:J 1>- Jil ~· !:';-~
QIO: 5123 ..L ar Pre v ious Next Labfli!llues Not es Calcula t o r
A A

17
A 4-year-old boy is brought to the emergency department by his mother, who is concerned about his
18 difficulty breath ing. She said he was playing outside when his father was mowing the lawn. On physical
19 examination, the patient's respiratory rate is 30/ min with occasional coughing spells, but he is stil l playful
20
and interactive. Auscultation of his lungs is significant for the sound heard in the aud io clip . The physician correctly
orders a sing le pharmacologic agent that rapidly improves his symptoms.
21
OPE N MEDIA
22
23
24 What is the molecular target of this agent?
25
:
. 26 A. a 1 -adrenergic receptor
. 27
B. 13 1 -adrenerg ic receptor
. 28
• 29
C. 13r adrenerg ic receptor

o30 D. Adenosine receptor


• 31 E. Leukotriene receptor
• 32
F. Muscarinic receptor
• 33
• 34
o35
. 36
. 37 •

a
Lock
s
Suspend
8
End Bl ock
Item: 26 of 37 ~. I • M k <:] t> al ~· ~
QIO: 5123 .l. ar Previous Next Lab 'lifllues Notes Calculator


17
The correct answer is C. 80°/o chose this.
18 The boy in the vignette is having an asthma exacerbation, and his pulmona ry examination in the aud io cl ip is
19 significant for wheezing. The primary agent used in the management of acute asthma fla re is albutero l, a ~ 2 -
20 adrenergic receptor agon ist. Albuterol reverses the bronchospasm that underl ies asthma exacerbations by
relaxing smooth muscle tone.
21
Bronchospasm Asthma Salbutamol Smooth muscle tissue Agonist Wheeze Muscle Receptor (biochemistry)
22
23
A is not correct. 3°/o chose this.
The a 1 -ad renergic receptor is involved in vascu lar smooth muscle contraction. An a1 -adrenergic receptor agonist
24
such as norepinephrine is useful in the management of hypotension, by increasing total peripheral resistance.
25 Vascular resistance Norepinephrine Hypotension Smooth muscle tissue Agonist Vascular smooth muscle Muscle contraction Blood vessel Muscle

26 Receptor (biochemistry)
• 27
B is not correct. 8°/o chose this .
• 28
The ~rad renerg i c receptor is pr imar ily involved in cardiac contracti lity. A ~radrenerg i c receptor agonist such as
• 29 dobutamine is a positive inotropic agent and is indicated for the short-term treatment of card iac
o30 decompensation that may occur after cardiac su rgery, or in patients with congestive heart failure or acute
• 31 myocard ial infarction .
Myocardial infarction Heart failure Dobutamine Congestive heart failure Inotrope Contractility Cardiac surgery Agonist Decompensation
• 32
• 33 Dis not correct. 1°/o chose this .
o34 Adenosine receptor blockers such as theophylline likely cause bronchodi lation by inhibiting phosphod iesterase.
Due to its numerous adverse effects and narrow therapeutic index, use of th is med ication as first-line therapy
o35
has fal len out of favor .
• 36 Adenosine receptor Therapeutic index Theophylline Bronchodilator Adenosine Phosphodiesterase Receptor (biochemistry) Pharmaceutical drug

• 37 ~ ~ =- --~ ------"" l:'n/_ _ .... _ _ _ ... a...:-

6
lock
s
Suspend
0
End Block
Item: 26 of 37 ~. I • M k <:] t> al ~· ~
QIO: 5123 .l. ar Previous Next Lab 'lifllues Notes Calculator

17
• •
Adenosine receptor blockers such as theophylline likely cause bronchodi lation by inhibiting phosphod iesterase.
18
Due to its numerous adverse effects and narrow therapeutic index, use of th is medication as first-l ine therapy
19 has fal len out of favor.
20 Adenosine receptor Therapeutic index Theophylline Bronchodilator Adenosine Phosphodiesterase Receptor (biochemistry) Pharmaceutical drug

21 E is not correct. 5°/o chose this.


22 Leu kotriene receptor antagonists such as zafir lukast are not fi rst-l ine agents for treating an asthma
23 exacerbation . They can be helpful in the long-ter m management of asthma, especially those that are aspirin-
24
induced.
Zafirlukast Leukotriene Asthma Antileukotriene Receptor (biochemistry) Leukotriene receptor
25
26 F is not correct. 3°/o chose this.
• 27 Muscarinic receptor antagonists such as ipratropium inhibit bronchoconstr iction . Although it is often emp loyed
as single-agent therapy for the t reatment of chron ic obstructive pu lmonary disease, it is not first-l ine for the
• 28 treatment of an asthma exacerbation . There is some evidence suggesting that the use of ipratropium in
• 29 conjunction with albuterol can be helpful in the management of asthma .
Muscarinic acetylcholine receptor Chronic obstructive pulmonary disease Ipratropium bromide Bronchoconstriction Muscarinic antagonist Asthma Salbutamol
o30
• 31 Receptor (biochemistry)

• 32
• 33 Bottom Line:
34
~rAdrenergic
0
receptor agon ists such as albuterol are the f irst- line agents used for treatment of acute asthma
0 35 exacerbations.
• 36 Asthma Salbutamol Receptor (biochemistry) Agonist

• 37 ~

6
lock
s
Suspend
0
End Block
Item: 26 of 3 7 ~ 1 • M k -<:J 1>- Jil ~· !:';-~
QIO: 5123 ..L ar Pre v ious Next Labfli!llues Not es Calcula t o r
A A

17 FA17 p649.1
18 Asthma drugs Bronchoconstriction is mediated b~ (I) inOammalory processes and (2) parasympathetic tone;
19 therapy is directed at these 2 pathways.
20 ~-agonists Albuterol- rclaxes bronchial smooth muscle (short acting J3z-agonist). Used during acute
21 exacerbation.
22 Salmeterol, formoterol - long-acting agents for prophylaxis. dverse effects are tremor and
23 arrll\ thmia.
24 Inhaled Fluticasone, budesonide - inhibitthe synthesis of ,·irhtally all cytokines. Inacti,·ate NF-KB, the
25 corticosteroids transcription factor that induces production ofT ' F-<X and other in8ammatory agents. lsi-line
therapy for chronic asthma. \ lay cause oral thrush.
26
Muscarinic Tiotropium, ipratropium-competilivel) blocl. muscarinic receptors, preventing
• 27
antagonists bronchoconstriction. Also used for COPD. Tiolropium is long acting.
• 28
Antileukotrienes Montelukast, zafirlukast- blocl. leul.otriene Exposure to antigen
• 29
receptors (CysLTI). Especially good for (dust pollen, etc)
o30 aspirin-induced and exercise-induced asthma.
• 31 Zileuton -5-lipoxygenasc pathway inh ibitor. t-0- Avoidance
• 32 Blocks conversion of arachidonic acid to
• 33
leukotrienes. Hepatotoxic.
Anti-lgE monoclonal Omalizumab-binds mostly unbound serum Antigen and tgE t-0- Omatizumab
• 34 on mast cells
therapy lgE and blocks binding to FceRI. Used in
o35
allergic asthma with t lgE levels resislanl lo
• 36 ~ Steroids
inhaled steroids and long-acting J3z-agonists.
• 37

a
Lock
s
Suspend
8
End Bl ock
Item: 26 of 37 ~. I • M k <:] t> al ~· ~
QIO: 5123 .l. ar Previous Next Lab 'lifllues Notes Calculator

• •
17 FA17 p638.1

18 Obstructive lung Obstruction of air fl ow - air trapping in lungs. Airways close prematurely at high lung volumes
19 diseases - t RV and t FRC, t TLC. PFTs: U FEV 1, l FVC - l FEV 1/FVC ratio (hallmark),
20 V/Q mismatch. C hronic, hypoxic pulmonary vasoconstriction can lead to cor pulmonale. Chronic
obstructive pulmonary disease (COPD) includes chronic bronchitis and emphysema.
21
TYPE PRESENTATION PATHOLOGY OTHER
22
Chronic bronchitis Findings: wheezing, crackles, Hypertrophy and hyperplasia Diagnostic criteria: productive
23 ("blue bloater") cyanosis (hypoxemia due of mucus-secreting glands cough for > 3 months in a
24 to shunting), dyspnea, C02 in bronchi - Reid index year for> 2 consecutive years.
25 zo
retention, polyc)themia. (thickness of mucosal gland
26 layer to thickness of wall
between epithelium and
• 27
cartilage) > 50%.
• 28
Emphysema ("pink Centriacinar-associated with Enlargement of air spaces CXR: t AP diameter, flattened
• 29
puffer") smoking rJ IE). Frequently in l recoil, t compliance, diaphragm, t lung field
o30 upper lobes (smoke rises up). l DLCO from dest ruction of lucency.
• 31 Panacinar-associated with alveolar walls (arrow in ~). Barrel-shaped chest [!].
• 32 a 1-antitrypsin deficiency. t elastase activity - t loss Exhalation through pursed lips
• 33
Frequently in lower lo bes. of elastic fibers - t lung to increase airway pressure
compliance. and prevent airway collapse.
0 34
Asthma Findings: cough, wheezing, Branchial hypcrrcsponsi,·cncss Aspi rin-i nduced asthma: COX
0 35
tachypnea, dyspnea, - reversible inhibition - leukotriene
• 36
hypoxemia, l inspiratory/ bronchoconstriction. Smooth overproduction - airway
• 37 • exoi ratorv ratio. nul sus muscle hvoertrooh v and constriction. Associated with •

6
lock
s
Suspend
0
End Block
Item: 27 of 37 ~. I • M k <:] t> al ~· ~
QIO: 1716 .l. ar Previous Next lab 'lifllues Notes Calculator


17
A 3 1-year-old intravenous drug user comes to the physician with a 5-day history of shortness of breath . The ~~AI
18 patient says that he also has had a nonproductive cough in the same time period . X-ray f ilm of the chest
19 revea ls bilateral diffuse powdery interstitia l infiltrates, and laboratory resu lts show a WBC count of
2500/mm 3 . Methenamine silver stain shows the causative organism . The patient is started on an appropr iate
20
t reatment.
21
22
Wh ich of the fo llowing is the mechanism of action of the antibiotic used to treat this patient's infection?
23
:
24
A. Blockade of ergosterol synthesis
25
B. Blockade of the pathway that uti lizes pteridine and PABA in nucleotide formation
26
• 27 C. I nhibition of peptidoglycan synthesis
• 28 D . I nhibition of the 305 r ibosomal subunit
• 29
E. I nhibition of the 505 r ibosomal subunit
o30
• 31
• 32
• 33
o34
o35
• 36
• 37 ~

6
lock
s
Suspend
0
End Block
Item: 27 of 3 7 ~ 1 • M k -<:J 1>- Jil ~· !:';-~
QIO: 1716 ..L ar Pre v ious Next Labfli!llues Notes Calcula t o r
A A

17
The correct a nsw e r is B. 42°/o chose this.
18
In an IV drug user w ith leukopenia, the index of suspicion for AIDS should be
19
high. The patient described has Pneumocystis jirovecii pneumon ia, which is
20 commonly associated with AIDS and immunosuppression. It is diagnosed with
21 methenamine silver stain of lung biopsy tissue. Though radiographic findings
may be inconclusive, there is often a characteristic pattern of powdery
22
interstitial infiltrates (shown in the image). Symptoms typically progress
23 slowly, and careful questioning can reveal long-standing, progressive
24 exertional dyspnea. This is different from the presentation in HIV-
25 seronegative patients, who typically have a more rapid onset of symptoms
when infected. P. jirovecii pneumonia is treated primarily with trimethoprim-
26
sulfamethoxazole, but it can be treated with pentamidine or dapsone. Both
27 sulfamethoxazole and trimethoprim inhibit the folate synthesis pathway. In
. 28 the folate synthesis pathway, pteridine and para-aminobenzoic acid (PABA)
Image copyright © Annals of
are incorporated into folic acid, which is important in the formation of nucleic
• 29 Thoracic Medicine
acids and certain amino acids .
o30 Pentamidine Dapsone Leukopenia Folic acid Dyspnea Pneumocystis jirovecii Trimethoprim/sulfamethoxazole Trimethoprim Immunosuppression HIV/AIDS Biopsy
• 31
Sulfamethoxazole Pneumonia Pneumocystis pneumonia 4-Aminobenzoic acid Amino acid Nucleic acid Drug injection Lung
• 32
A is not correct. 18°/o chose this .
• 33
Many antifungal agents inhibit ergosterol synthesis, includ ing fluconazole and terbinafine. Although
• 34
Pneumocystis jirovecii is a fungus, antifungals that block ergosterol synthesis are not effective in the treatment
o35 of this infection.
. 36 Terbinaflne Ergosterol Fluconazole Fungicide Antifungal Fungus

. 37 • C i~ not r.o rrP.r.t . 110/n r.ho~P. thi~ .

a
Lock
s
Suspend
8
End Bl ock
Item: 27 of 37 ~. I • M k <:] t> al ~· ~
QIO: 1716 .l. ar Previous Next lab 'lifllues Notes Calculator
- -

6
lock
s
Suspend
0
End Block
Item: 27 of 3 7 ~ 1 • M k -<:J 1>- Jil ~· !:';-~
QIO: 1716 ..L ar Pre v ious Next Labfli!llues Notes Calcula t o r
A A

17
FA17 p 150.1
18
19 Pneumocystis jirovecii Causes Pneumocystis pneumonia (PC P), a diffuse interstitial pneumonia rJ. Yeast-like
20
fungus (originally classified as protozoan). Inhaled. \lost infections are asymptomatic.
Immunosuppression (eg, AIDS) predisposes to disease. Diffuse, bilateral ground-glass opacities on
21
CXRICT [l]. Diagnosed b) lung biops} or lavage. Disc-shaped yeast seen on methenamine siher
22 stain of lung tissue
23 Treatment/prophylaxis: TYIP-SI\ IX, pentamidine, dapsone (prophylaxis only), ato,·aquone. Start
24 prophylaxis when CD4+ count drops to < 200 cclls/mm) in lllV patients.
25
26

27
• 28
• 29
o30
• 31
• 32

• 33
• 34
o35 FA17 p 190.1
• 36 Sulfonamides Sulfamethoxazole (SMX), sulfisoxazole, PABA+ Pteridine
• 37 sulfadiazine.

a
Lock
s
Suspend
8
End Bl ock
Item: 27 of 3 7 ~ 1 • M k -<:J 1>- Jil ~· !:';-~
QIO: 1716 ..L ar Pre v ious Next Labfli!llues Notes Calcula t o r
A A

17 FA17 p 190.1

18 Sulfonamides Sulfamethoxazolc (SMX), sulfisoxazole,


19
20 MECHANISM
sulfadiazine.
Inhibit dihydropteroate srnthase, thus inhibiting
Dihyo ov UCJ
n ""
lt-0-
PABA+ Pteridine

Sulfonamides.
dapsone

21 folate s~ nthesis. Bacteriostatic (bactericidal


Dihydropteroic acid
when combined with trimethoprim).
22
23
CLINICAL USE Gram<±>. gram 8 , 'ocardia. Sf\ I ' for simple
UTI.
24
ADVERSE EFFECTS Hypersensiti,itr reactions, hemolysis if G6PD
25
deficient, nephrotoxicitr {tubulointerstitial
26 D1hydrofolic acid
nephritis), photosensiti,•ity, Ste,·ens-Johnson
27 syndrome, kern icterus in infants, displace D ~.o of, ·!1-0-- Trimethoprim.
yreductase pyrimethamine
• 28 other drugs from albumin (eg, warfarin).
• 29 MECHANISM OF RESISTANCE Altered enzyme (bacterial dihydroptcroate Tetrahydrofolic acid
o30 synthase), I uptake, or t PAB synthesis. /!~
Thymidine Methionine
• 31 Purines

• 32 !
DNA. RNA
!
DNA
!
Protein
• 33
• 34
FA17 p 190.3
o35
Trimethoprim
• 36
MECHANISM Inhibits bacterial dilwdrofolate reductase.
• 37 n .. • - -·· -· · - ·· - '

a
Lock
s
Suspend
8
End Bl ock
Item: 27 of 3 7 ~ 1 • M k -<:J 1>- Jil ~· !:';-~
QIO: 1716 ..L ar Pre v ious Next Labfli!llues Notes Calcula t o r

• g ( g, )
17
MECHANISM OF RESISTANCE
0

Altered enzyme (bacterial dih)droptcroatc


l
Tetrahydrofolic acid
18
19
synthase), l uptake, or t PAB synthesis.
/1"
Purines Thymidine Methionine
20
21
+
DNA RNA
1
DNA Protein
22
23
FA17 p 190.3
24 Trimethoprim
25 MECHANISM Inhibits bacterial dihydrofolatc reductase.
26 Bacteriostatic.
27 CliNICAL USE Used in combination with sulfonamidcs
• 28 (trimethoprim-sulfamethoxazole [TM P-
• 29 SMX]), causing sequential block of folate
synthesis. Combination used for UTis,
o30
Shigella, Salmonella, Pneumocystis jirovecii
• 31
pneumonia treatment and prophylaxis,
• 32 toxoplasmosis prophylaxis.
• 33 ADVERSE EFFECTS legaloblastic anemia, leukopenia,
• 34 granulocytopenia. (May alleviate with
o35 supplemental folinic acid). T\1P Treats
• 36
\farrow Poorly.
• 37

a
Lock
s
Suspend
8
End Bl ock
Item: 28 of 3 7 ~ 1 • M k -<:J 1>- Jil ~· !:';-~
QIO: 1753 ..L ar Pre v ious Next Labfli!llues Not es Calcula t o r

IAA]
A A

17
A 75-year-old man comes to the hea lth clinic after noticing a burn ing sensation in his mouth and some
18 wh itish fi lm on his tongue and the inside of his cheeks. His past med ical history is significant for long-
19 standing asthma, for which he was recently prescribed an inhaled steroid following an acute asthma
exacerbation about 4 weeks ago.
20
21
Which of the following is the most appropriate agent to prescribe for this patient?
22
23 :
A. Albendazole
24
25 B. Amphotericin B
26 C. Griseofulvin
27
D. Nystatin
. 28
• 29
E. Terbinafine

o30
• 31
• 32
• 33
• 34
o35
. 36
. 37 •

a
Lock
s
Suspend
8
End Bl ock
Item: 28 of 37 ~. , . M k <:] t> al ~· ~
QIO: 1753 .l. ar Previous Next lab 'lifllues Notes Calculator

17
The correct answer is D. 60°/o chose this.
18
This man is suffering from oral candidiasis, also known as th r ush. Candida albicans is a yeast that is a norma l
19 part of the oral flora . It can become pathogenic in immunocompromised ind ividuals. Th is man has been using
20 inhaled steroids, which puts him at risk for ora l candidiasis. Other associations include infancy, dentu res, oral
21 antibiotic use, corticosteroids, excessive antibacterial mouthwash, dry mouth, trauma to the ora l cavity,
diabetes mellitus, iron deficiency, folate deficiency, and immunodeficiency (e.g., transplant, ma lignancy, HI V
22
infection). Ora l nystatin preparations are the first-l ine treatment for candid iasis and have the least harmfu l side
23 effects . Nystatin functions by binding to ergosterol (the sterol specific to fungal cell membranes), thus forming
24 holes in and disrupting fungal membranes.
Candida albicans Nystatin Ergosterol Oral candidiasis Antibiotics Candidiasis Sterol Diabetes mellitus Corticosteroid Folic acid Folate deficiency Candida (fungus)
25
Mouthwash HIV Immunodeficiency Yeast Dentures Mouth Fungus Pathogen Steroid Cell membrane Malignancy Xerostomia Iron deficiency Cancer
26
27 A is not correct. 11% chose this.
28 Albendazole is an antihelminthic drug used to treat a variety of parasitic infections, such as with Ascaris,
• 29 hookworms, pinworms, and others . It has no role in the t reatment of fungal infections .
Albendazole Anthelmintic Parasitism Hookworm infection Ascaris Pinworm Fungus Mycosis
o30
•31 B is not correct. 16% chose this .
• 32
Amphotericin B also binds to ergosterol. It is generally reserved for systemic fungal infections (with
Cryptococcus, Blastomyces, Coccidioides, Aspergillus, Histoplasma, and Candida species) due to its strength and
• 33
ser ious adverse effects . An oral preparation of amphotericin B is available to treat oropharyngeal candidiasis.
o34 This amphoter icin oral suspension causes fa r fewer side effects than the I V version of the drug. However,
o35 nystatin is still considered the first-line treatment for oral candidiasis due to its well-established safety and
efficacy .
• 36
Nystatin Amphotericin B Ergosterol Oral candidiasis Candidiasis Candida (fungus) Mycosis Cryptococcus Aspergillus Blastomyces dermatitidis Fungus Coccidioides
• 37 ... ... . .
6
lock
s
Suspend
0
End Block
Item: 28 of 37 ~. , . M k <:] t> al ~· ~
QIO: 1753 .l. ar Previous Next lab 'lifllues Notes Calculator


17
A is not correct. 11% chose this.
18
Albendazo le is an antihelminthic drug used to treat a variety of parasitic infections, such as with Ascaris,
19 hookworms, pinworms, and others . It has no role in the t reatment of fungal infections.
20 Albendazole Anthelmintic Parasitism Hookworm infection Ascaris Pinworm Fungus Mycosis

21
B is not correct. 16% chose this.
22 Amphotericin B also binds to ergosterol. It is generally reserved for systemic funga l infections (with
23 Cryptococcus, Blastomyces, Coccidioides, Aspergillus, Histoplasma, and Candida species) due to its strength and
24 ser ious adverse effects . An oral preparation of amphotericin B is availab le to treat oropharyngeal candidiasis.
This amphoter icin oral suspension causes fa r fewer side effects than the I V version of the drug. However,
25
nystatin is sti ll considered the first-line treatment for oral candidiasis due to its well-established safety and
26 efficacy.
27 Nystatin Amphotericin B Ergosterol Oral candidiasis Candidiasis Candida (fungus) Mycosis Cryptococcus Aspergillus Blastomyces dermatitidis Fungus Coccidioides

28 Histoplasma Pharynx

• 29
C is not correct. 8°/o chose this.
o30 Griseofulvin interferes with microtubule function and disrupts mitosis in funga l cel ls. It is an oral preparation
• 31 used to treat topica l fungal infections of the hair and skin (tinea, ringwor m) by organisms in the Trichophyton
• 32 and Microsporum genera .
Griseofulvin Microtubule Dermatophytosis Mitosis Trichophyton Mycosis Fungus Microsporum Genus
• 33
o3 4 E is not correct. 5°/o chose this.
o35 Ter binafine inhibits the fungal enzyme squa lene epoxidase. It is primarily used to treat onychomycosis (toenai l
fungus) .
• 36
Terbinafine Squalene monooxygenase Squalene Enzyme Onychomycosis Nail (anatomy) Fungus
• 37 ~

6
lock
s
Suspend
0
End Block
Item: 28 of 37 ~. , . M k <:] t> al ~· ~
QIO: 1753 .l. ar Previous Next lab 'lifllues Notes Calculator
- . -- -- -- -- - -

17 Amphotericin B also binds to ergosterol. It is generally reserved for systemic funga l infections (with
18 Cryptococcus, Blastomyces, Coccidioides, Aspergillus, Histoplasma, and Candida species) due to its strength and
19 ser ious adverse effects . An oral preparation of amphotericin B is available to treat oropharyngeal candidiasis.
This amphoter icin oral suspension causes fa r fewer side effects than the I V version of the drug. However,
20
nystatin is sti ll considered the first-line treatment for oral candidiasis due to its well-established safety and
21 efficacy.
22 Nystatin Amphotericin B Ergosterol Oral candidiasis Candidiasis Candida (fungus) Mycosis Cryptococcus Aspergillus Blastomyces dermatitidis Fungus Coccidioides

23 Histoplasma Pharynx

24 C is not correct. 8°/o chose this.


25 Griseofulvin interferes with microtubule function and disrupts mitosis in funga l cells. It is an oral preparation
26 used to treat topica l fungal infections of the hair and skin (tinea, ringwor m) by organisms in the Trichophyton
27 and Microsporum genera.
Griseofulvin Microtubule Dermatophytosis Mitosis Trichophyton Mycosis Fungus Microsporum Genus
28
• 29 E is not correct. 5°/o chose this .
o30
Ter binafine inhibits the fungal enzyme squa lene epoxidase. It is primarily used to treat onychomycosis (toenai l
fungus) .
• 31 Terbinafine Squalene monooxygenase Squalene Enzyme Onychomycosis Nail (anatomy) Fungus
• 32
• 33
o34
Bottom Line:
o35 Nystatin is the first-line t reatment for ora l candidiasis (th rush) and has the least harmfu l side effects.
Nystatin Oral candidiasis Candidiasis Thrush (bird)
• 36
• 37 ~

6
lock
s
Suspend
0
End Block
Item: 28 of 3 7 ~ 1 • M k -<:J 1>- Jil ~· !:';-~
QIO: 1753 ..L ar Pre v ious Next Labfli!llues Not es Calcula t o r
A A

17
FA17 p 195.2
18
Nystatin
19
MECHANISM Same as amphotericin B. Topical use only as too toxic for S)Stemic use.
20
CLINICAL USE "Swish and swaJlow" for oral candidiasis (thrush); topical for diaper rash or vaginal candidiasis.
21
22
FA17 p 149.1
23
Opportunistic fungal infections
24
Candida albicans alba = "hite. Dimorphic; forms pseudoh) phae and budding} casts at 20°C [],germ tubes at
25
37°C : .
26
Systemic or superficial fu ngal infection. Causes oral and esophageal thrush in
27 immunocompromiscd (neonates, steroids, diabetes, AIDS), vuh-o,·aginitis (diabetes, usc of
28 antibiotics), diaper rash, endocarditis (IV drug users), disseminated candidiasis (especially in
• 29 neutropen ic patients}, ch ronic mucocutaneous cand idiasis.
o30
Treatment: oral Auconazolc/topical azoic for vaginal; nystatin, AucomlZole, or caspofungin for owl/
esophageal; Auconazole, caspofungin, or amphotericin B for systemic.
• 31
Aspergillus Septate hyphae that branch at 45° Acute Angle (!l. Produces conidia in radiating chains at end of
• 32
fumlgatus con id iophore 11.
• 33
Causes invasive aspergillosis in immunocompromised, patients with chronic granulomatous disease.
• 34 Can cause aspergillomas in pre-existing lung cavities, especially after TB infection .
• 35 Some species of Aspergillus produce Anatoxins (associated'' ith hepatocellular carcinoma).
. 36 Allergic bronchopulmonary aspergillosis ( BPA): h) persensitivity response associated with
. 37 • asthma and cystic fibrosis; may cause bronchiectasis and eosinophilia .

a
Lock
s
Suspend
8
End Bl ock
Item: 28 of 3 7 ~ 1 • M k -<:J 1>- Jil ~· !:';-~
QIO: 1753 ..L ar Pre v ious Next Labfli!llues Not es Calcula t o r
A A

17 Cryptococcus 5-10 1-1111 with narrow budding. llcavily encapsulated yeast. 1 ol dimorphic.
18 neoformans round in soil, pigeon droppings. Acquired through inhalation with hematogenous dissemination
19 to meninges. Culture on Sabouraud agar. l ligh lighted with India ink (clear halo ) and
20
mucicarmine (red inner capsule['!!). Latex agglutination lest detects polysaccharide capsular
antigen and is more specific.
21
Causes cr)-ptococcosis, cryptococcal meningitis, crrptococcal encephalitis ("soap bubble" lesions
22 in brain), primarily in immunocompromiscd.
23 Treatment: amphotericin B + flue) tosine follo"ed b) Auconazole for cryptococcal meningitis.
24 Mucor and Rhizopus Irregular, broad, nonseplate h) phae branching at wide angles CJ.
25 spp. ,\lucormycosis. Causes disease most I) in l..ctoacidotic diabetic and/or neutropenic patients (cg,
26 leukemia). Fungi proliferate in blood vessel walls, penetrate cribriform plate, and enter brain.
Rhinocerebral, frontal lobe abscess; cavernous sinus thrombosis. Headache, facial pain, black
27
necrotic eschar on f;1cc; may have cranial ncr\C imoh-cmenl.
28
Treatment: surgical debridement, amphotericin B.
• 29
o30
• 31
• 32

• 33
• 34
• 35
• 36
• 37

a
Lock
s
Suspend
8
End Bl ock
17
18
19 , ...
20
21 FA17 p 194.4
22 Antifungal therapy
23 FUNGAL CELL
lANOSTEROL SYNTHESIS
24
25 Terbmahne Echinocandins
Anidulafungin
26 ~ Squalene Caspofungin
27 / Squalene epoxidase Micafungin

28 Squalene epoxide

• 29
o30
I
Lanosterol Polyenes
Amphotericin B
• 31 ~-a-demethylase Ergosterol
. . L....-_ Nystatin
Azoles ___J
• 32 Clotrimazole
• 33 Fluconazole
ltraconazole
' ''•""'rot
• 34 Ketoconazole
M•conazole 1•
o35
. 36
Voriconazole [
'-· ---
Flucytosine )

. 37 •

a
~k s n
8
Bock
Item: 29 of 3 7 ~ 1 • M k -<:J 1>- Jil ~· !:';-~
QIO: 3391 ..L ar Pre v ious Next Lab fli!ltues Not es Calcula t o r

IAA]
A A

17
A 24-year-old woman present s t o her physician complaining of episodes of difficulty breat hing. She den ies
18 any chest pain, nausea, vomit ing, headache, or other symptoms. She has a prior history of asthma in
19 childhood.
20
21 Which of the following asthma m edications has a mechanism of action involving direct inhibit ion of an enzyme?
22 :
23 A. Albuterol
24 B. Ipratropium
25
C. Salmeterol
26
D. Zafirlukast
27
28 E. Zileuton
. 29
o30
• 31
• 32
• 33
• 34
o35
. 36
. 37 •

a
Lock
s
Suspend
8
End Bl ock
Item: 29 of 37 ~. I • M k <:] t> al ~· ~
QIO: 3391 .l. ar Previous Next lab 'lifllues Notes Calculator


17
The correct answer is E. 59°/o chose this.
18
Zi leuton is a 5-lipoxogenase-pathway inhibitor that blocks the conversion of arach idonic acid into potent
19
bronchoconstrictors called leukotrienes. By inhibiting leukotriene production, zi leuton decreases the tone of the
20 bronchial smooth muscle, making it an effective therapy in ch ron ic asthma . In rare cases, zileuton can cause
21 adverse events such as dark urine, clay-colored stools, jaund ice, skin br uising, and ting ling, numbness, or
weaken ing of muscles .
22
Zileuton Arachidonic acid leukotriene Jaundice Asthma Smooth muscle tissue Enzyme inhibitor Urine Weakness Paresthesia
23
24
A is not correct. 4°/o chose this.
25
Albuterol, a ~rreceptor agonist, relaxes bronchia l smooth muscle and is first-line therapy for acute asthmatic
exacerbations. Common side effects include bronchospasm, headache, irregu lar heart rate, tremors, and
26 dizziness.
27 Bronchospasm Cardiac arrhythmia Agonist Salbutamol Smooth muscle tissue Heart rate Headache Dizziness Palpitations Bronchus Adverse drug reaction Muscle

28 Side effect

29 B is not correct. 10% chose this.


o30 Ipratropium is a muscarinic-receptor blocker that prevents acetylcholine-mediated positive modu lation of
• 31 bronchial smooth-muscle tone. It has a slower onset of action than albuterol, but is usefu l in situations in wh ich
• 32 albuterol is contraindicated or ineffective. I pratropium can cause hypersensitivity reactions, ur inary retention,
and bronchospasm as side effects .
• 33
Bronchospasm Ipratropium bromide Urinary retention Salbutamol Hypersensitivity Adverse drug reaction Side effect Bronchus Smooth muscle tissue
o34
Contraindication
o35
• 36
C is not correct. 4°/o chose this .
• 37
Salmeterol is a long-acting ~rreceptor agonist that chronical ly relaxes bronchial smooth muscle. It is
~ ... ·--...1 -- 1--- ._._. _ _ _. __
------··--~ ~ ... :- c _ _. --•L.--•=- -··--•- T•- _ ...... _ .... __ _ cc__.. _ :--1 .. ..J- ...... _ _ _ ,... --...1
~,..., , 1-

6
lock
s
Suspend
0
End Block
Item: 29 of 37 ~. I • M k <:] t> al ~· ~
QIO: 3391 .l. ar Previous Next lab 'lifllues Notes Calculator
.::»lUI:: t:::ffl:::(..l

17
18
B is not correct. 10% chose this.
19
I pratropium is a muscarinic-receptor blocker that prevents acetylcholine-mediated positive modu lation of
bronchial smooth -muscle tone. It has a slower onset of action than albuterol, but is usefu l in situations in wh ich
20 albuterol is contraindicated or ineffective. I pratropium can cause hypersensitivity reactions, ur inary retention,
21 and bronchospasm as side effects .
Bronchospasm Ipratropium bromide Urinary retention Salbutamol Hypersensitivity Adverse drug reaction Side effect Bronchus Smooth muscle tissue
22
Contraindication
23
24 C is not correct. 4°/o chose this.
25 Salmeterol is a long-acting ~rreceptor agonist that chronical ly relaxes bronchial smooth muscle. It is
26 consequently used as long-ter m prophylaxis for asthmatic events. Its adverse eff ects include tremor and
ar rhythmia .
27
Salmeterol Smooth muscle tissue Cardiac arrhythmia Tremor Agonist Preventive healthcare Bronchus Muscle
28
D is not correct. 23% chose this.
29
Zafirlukast is a leukotriene-receptor blocker used for maintenance in the setting of chronic asthma . Common
o30
side effects include cough, fever, easy bruising, tingling or numbness, muscle weakness, and mood changes .
• 31 Zafirlukast Asthma Cough Fever Side effect Adverse drug reaction Muscle

• 32
• 33
Bottom Line:
o34
Zi leuton is a 5-lipoxogenase-pathway inhibitor that blocks conversion of arachidonic acid into leukotriene
o35
bronchoconstrictors .
• 36 Zileuton Arachidonic acid leukotriene Enzyme inhibitor
• 37 ~

6
lock
s
Suspend
0
End Block
Item: 29 of 3 7 ~ 1 • M k -<:J 1>- Jil ~· !:';-~
QIO: 3391 ..L ar Pre v ious Next Lab fli!ltues Not es Calcula t o r
A A

17
18 FA17 p649.1

19 Asthma drugs Bronchoconstriction is mediated b~ (I) inOammatory processes and (2) parasympathetic tone;
20 therapy is directed at these 2 pathways.
21 ~-agonists Albuterol-relaxcs bronchial smooth muscle (short acting 13z-agonist). Used during acute
exacerbation.
22
23
Salmete rol, formote rol- long-acting agents for prophrlaxis. Ad,·erse effects are tremor and
arrh' Lhmia.
24
Inhaled Fluticasone, budesonide -inhibit thc synthesis of \'irtually all cytokines. lnacti,·atc 1 F-KB, the
25
corticoste roids transcription factor that induces production ofT lJ".a and other in8ammatory agents. 1st-line
26
therapy for chronic asthma. Vlay cause oral thrush.
27
Muscarinic Tiotropium, ipratropium- competitivel) bloc!. muscarinic receptors, preventing
28 a ntagonists bronchoconstriction. Also used for COPD. Tiotropium is long acting.
29 Antile ukotrienes Montelukast, zafirlukast-bloc!. lcul-otricnc Exposure to antigen
o30 receptors (CysLTl). Especially good for (dust pollen, etc)
• 31 aspirin-induced and exercise-induced asthma .
• 32 Zileuton -5-lipoxygcnasc p:1thway inhibitor. t-0- Avoidance
Blocks conversion of arachidonic acid to
• 33
leukotrienes. Hepatotoxic.
• 34
Anti-lgE monoclonal Omalizumab-binds mostly unbound serum Antigen and tgE t-0- Omalizumab
• 35 on mast cells
the ra py lgE and blocks binding to Fc£RI. Used in
• 36 allergic asthma with t lgE lc"cls resistant to ....,0..... Steroids
• 37 inhaled steroids and long-acting 1},-agonists.

a
Lock
s
Suspend
8
End Bl ock
Item: 29 of 37 ~. I • M k <:] t> al ~· ~
QIO: 3391 .l. ar Previous Next lab 'lifllues Notes Calculator

• •
17
FA17 p638.1
18
Obstructive lung Obstruction of air fl ow - air trapping in lungs. Airways close prematurely at high lung volumes
19
diseases - t RV and t FRC, t TLC. PFTs: U FEV 1, l FVC - l FEV 1/FVC ratio (hallmark),
20
V/Q mismatch. C hronic, hypoxic pulmonary vasoconstriction can lead to cor pulmonale. Chronic
21 obstructive pulmonary disease (COPD) includes chronic bronchitis and emphysema.
22 TYPE PRESENTATION PATHOLOGY OTHER
23 Chronic bronchitis Findings: wheezing, crackles, Hypertrophy and hyperplasia Diagnostic criteria: productive
24 ("blue bloater") cyanosis (hypoxemia due of mucus-secreting glands cough for > 3 months in a
25
to shunting), dyspnea, C02 in bronchi - Reid index year for> 2 consecutive years.
retention, zopolyc)themia. (thickness of mucosal gland
26
layer to thickness of wall
27 between epithelium and
28 cartilage) > 50%.
29 Emphysema ("pink Centriacinar-associated with Enlargement of air spaces CXR: t AP diameter, flattened
o30 puffer") smoking rJ IE). Frequently in l recoil, t compliance, diaphragm, t lung field
• 31 upper lobes (smoke rises up). l DLCO from destruction of lucency.
Panacinar-associated with alveolar walls (arrow in ~). Barrel-shaped chest [!].
• 32
a 1-antitrypsin deficiency. t elastase activity - t loss Exhalation through pursed lips
• 33
Frequently in lower lobes. of elastic fibers - t lung to increase airway pressure
0 34 compliance. and prevent airway collapse.
0 35 Asthma Findings: cough, wheezing, Branchial hypcrrcsponsi,·cncss Aspirin-i nduced asthma: COX
• 36 tachypnea, dyspnea, - reversible inhibition - leukotriene
• 37 •
hypoxemia, l inspiratory/ bronchoconstriction. Smooth overproduction - airway •

6
lock
s
Suspend
0
End Block
Item: 30 of 3 7 ~ 1 • M k -<:J 1>- Jil ~· !:';-~
QIO: 3445 ..L ar Pre v ious Next Lab fli!ltues Not es Calcula t o r
A A

17
A 36-year-old disheveled patient is brought to the emergency department ( ED) after being found
18 unresponsive in his home. His heartbeat at the time of transport is regu lar. He is eventual ly awakened by
19 emergency medical technicians, and at presentation to the ED he reports headache, dizziness, and nausea.
20
He is unable to provide any more details. His sister arrives and reveals that he has seemed a bit confused at
times, and because of his chronic financial difficulties has not been able to replace his poorly functioning heating
21 system.
22
23 What is the antidote to this pathologic process?
24
:
25 A. 100% Hyperbaric oxygen
26
B. Dimercaprol
27
28
C. Fomepizole
29 D. Methylene blue
o30 E. NaHC03
• 31
F. Physostigm ine sal icyl ate
• 32
• 33 G. Thiosu lfate
• 34
• 35
. 36
. 37 •

a
Lock
s
Suspend
8
End Bl ock
Item: 30 of 37 ~. I • M k <:] t> al ~· ~
QIO: 3445 .l. ar Previous Next lab 'lifllues Notes Calculator

17 The correct answer is A. 65°/o c hose this.


18 Th is patient is suffering classic symptoms of car bon monoxide (CO) poisoning . Diagnosis is based pr imar ily on
19 symptoms and history . The history of a poor ly function ing heating system in the winter is strong ly suggestive of
20
CO poison ing from a gas heater. Other common causes of CO poisoning includes smoke from burn ing wood . A
high suspicion of CO po ison ing should urge the physician to measure the patient's carboxyhemog lobin levels.
21 Treat with 100% hyperbar ic oxygen or 100% oxygen with high-flow mask.
22 Carbon monoxide Carboxyhemoglobin Oxygen Gas heater Hyperbaric medicine Carbon

23
B is not correct. 5 °/o chose this .
24 Dimercapro l is an antidote for toxicity caused by arsen ic, go ld, mercury, or lead . Overdose with any of the
25 heavy meta ls can general ly lead to headaches, kidney dysfunction, and abdominal symptoms . Lead poison ing is
26 often seen in people who live in houses built before 1976, or in ch ildren who inadvertently swa llow lead objects.
Symptoms of lead poisoning include abdom inal cramps, aggressive behavior, anem ia, and loss of developmenta l
27
skills in chi ldren. Symptoms of arsen ic and mercury overdose also include abdom inal cramps and muscle
28 cramps, but can additiona lly include hair loss, convu lsions, and a persistent meta llic taste in the mouth.
29 Dimercaprol Arsenic Mercury (element) lead poisoning Anemia Toxicity Antidote Toxic heavy metal Kidney Convulsion Hair loss Muscle Cramp Gold

30 Drug overdose

•31 C is not correct . 4 °/o chose this .


• 32 Fomepizo le is an antidote for toxicity caused by methanol and ethylene glyco l (antifreeze) . Upon init ial
• 33 ingestion, methano l and ethylene glycol can mimic ethanol consumption, but this feeling quickly fades . Over the
o34
course of a few hours after ingestion, nausea, vomiting, and convu lsions begin to occu r, and patients exhibit a
state of stupor that can turn into a coma .
o35
Fomepizole Ethylene glycol Methanol Ethanol Ethylene Antifreeze Antidote Nausea Toxicity Coma Vomiting Dial Convulsion
• 36
D is not correct. 13% c hose this .
• 37 'Y .. '" • I I .' ' ' I ' ,.
' ' .' I I • o o 1 •1
.'.
6
lock
s
Suspend
0
End Block
Item: 30 of 37 ~. I • M k <:] t> al ~· ~
QIO: 3445 .l. ar Previous Next lab 'lifllues Notes Calculator


17 D is not correct. 13% chose this.
18 Methylene blue is an antidote for toxicity caused by methemoglobin. Unlike oxyhemog lobin, methemoglobin
19 cannot bind oxygen. The iron in the heme group is in the ferric state (Fe3 +) instead of the ferrous state (Fe 2+) of
normal hemoglobin. Methemoglobinemia can be induced by certa in drugs including sulfonamide and dapsone
20
antibiotics, local anesthetics, aniline dyes, and metoclopramide.
21 Methemoglobinemia Metoclopramide Dapsone Methemoglobin Methylene blue Aniline Sulfonamide (medicine) Sulfonamide Heme Hemoglobin Antidote Antibiotics

22 Toxicity Anesthetic Ferric local anesthetic Iron Oxygen Ferrous

23
E is not correct. 4°/o chose this.
24
NaHC03 is an antidote for toxicity caused by tricyclic antidepressants . Symptoms of tricyclic antidepressant
25 toxicity man ifest with in two hou rs of ingestion and include card iovascular issues (palpitations, hypotension,
26 chest pa in), central nervous system issues (convulsions, drowsiness, decreased mental status) and peripheral
27
autonomic issues includ ing dry mouth, urinary retention, and blurred vision.
Tricyclic antidepressant Antidepressant Central nervous system Urinary retention Hypotension Palpitations Somnolence Autonomic nervous system Antidote
28
Chest pain Xerostomia Toxicity Blurred vision Nervous system Convulsion Tricyclic Circulatory system
29
30 F is not correct. 3°/o chose this.
• 31 Physostigmine sal icylate is an antidote for toxicity caused by antimuscarinic and anticholinerg ic agents .
Overdose of anticholinergic agents leads to symptoms including flushing, dry skin, mydriasis, fever, tachycardia,
• 32
urinary retention, and myoclonic jerking . A popu lar mnemon ic for the anticholinergic toxicity effects is: mad as
• 33 a hatter, blind as a bat, hot as a ha re, dry as a bone, red as a beet .
Anticholinergic Muscarinic antagonist Mydriasis Physostigmine Urinary retention Tachycardia Antidote Mnemonic Myoclonus Salicylic acid Drug overdose Fever
o34
o35 Toxicity Flushing (physiology)

• 36 G is not correct. 6°/o chose this.


• 37 ~ Thiosu lfate is an antidote for cyanide toxicity. Almost half of oatients with cyanide toxicitY reoort noticino an ~

6
lock
s
Suspend
0
End Block
Item: 30 of 37 ~. I • M k <:] t> al ~· ~
QIO: 3445 .l. ar Previous Next lab 'lifllues Notes Calculator
•• ...: - - " "' •• • .- • - • ..- -. 11':- • - .- ·-. - • 'f:ll- •
17 chest pa in), central nervous system issues (convulsions, drowsiness, decreased mental status) and peripheral
18 autonomic issues includ ing dry mouth, urinary retention, and blurred vision.
Tricyclic antidepressant Antidepressant Central nervous system Urinary retention Hypotension Palpitations Somnolence Autonomic nervous system Antidote
19
Chest pain Xerostomia Toxicity Blurred vision Nervous system Convulsion Tricyclic Circulatory system
20
21 F is not correct. 3°/o chose this.
22 Physostigmine sal icylate is an antidote for toxicity caused by antimuscarinic and anticholinerg ic agents.
23 Overdose of anticholinergic agents leads to symptoms including flushing, dry skin, mydriasis, fever, tachycardia,
urinary retention, and myoclonic jerking . A popu lar mnemon ic for the antichol inergic toxicity effects is: mad as
24
a hatter, blind as a bat, hot as a ha re, dry as a bone, red as a beet.
25 Anticholinergic Muscarinic antagonist Mydriasis Physostigmine Urinary retention Tachycardia Antidote Mnemonic Myoclonus Salicylic acid Drug overdose Fever

26 Toxicity Flushing (physiology)


27
G is not correct. 6°/o chose this.
28
Thiosu lfate is an antidote for cyanide toxicity. Almost half of patients with cyanide toxicity report noticing an
29 odor of bitter almonds . Symptoms of toxicity can include general weakness, progressive hypoxia, neu rologic
30 manifestations, apnea and respiratory arrest, and eye and mucous membrane ir ritation (depend ing on the route
• 31 of exposure to the cyanide) .
Mucous membrane Hypoxia (medical) Thiosulfate Cyanide Antidote Respiratory arrest Neurology Toxicity Cell membrane Almond Hypoxia (environmental)
• 32
• 33
o34 Bottom Line:
o35 Patients with CO poisoning present with headache, dizziness, confusion, and nausea. Heating-system problems
• 36 are strongly suggestive . CO poisoning is t reated with 100% hyperbar ic oxygen or high-flow oxygen .
Nausea Hyperbaric medicine Dizziness Headache Oxygen Diving chamber
• 37 ~

lock
6 s
Suspend
0
End Block
Item: 30 of 3 7 ~ 1 • M k -<:J 1>- Jil ~· !:';-~
QIO: 3445 ..L ar Pre v ious Next Lab fli!ltues Not es Calcula t o r


17
FA17 p239.1
18
Specific toxicity TOXIN TREATMENT
19 treatments Acetaminophen N-acetylcysteine (replenishes glutathione)
20
AChE inhibitors, organophosphates Atropine> pralidoxime
21
Antimuscarinic, anticholinergic agents Physostigmine, control hyperthermia
22
Arsenic Dimercaprol. succi mer
23
Benzodiazepines Flumazenil
24
~-blockers Atropine, glucagon
25
26
Carbon monoxide 100% 0 2, hyperbaric 0 2
27 Copper Penicillamine, trientine (Copper penny)
28 Cyanide Nitrile+ thiosulfate, hydroxocobalamin
29 Digitalis (digoxin) Anti-dig Fab fragmen ts
30 Heparin Protamine sulfate
• 31 Iron Deferoxamine, deferasirox, deferiprone
• 32 Lead ED'l 1\, dimercaprol, succimer, penicil lamine
• 33 l\lcrcury Dimercaprol, succimcr
• 34
Methanol, ethylene glycol (antifreeze) Fomepizole >ethanol, dialysis
• 35
l\ lethemoglobin 1\ lethvlene blue, vitamin C
'
• 36
OpiO ids 'alOxOnc
• 37 C" I • I • \ 1• I

a
Lock
s
Suspend
8
End Bl ock
Item: 30 of 3 7 ~ 1 • M k -<:J 1>- Jil ~· !:';-~
QIO: 3445

17
..L ar Pre v ious
., g
Next
. g
Lab fli!ltues Not es Calcula t o r
g g
Heparin Prolamine sulfate
18
Iron Dcferoxamine, deferasirox, defe riprone
19
20 Lead EOTA, dimercaprol, succimer, penicillamine

21 M ercury Dimercaprol, succimcr

22 [ethanol, ethylene glycol (antifreeze) F'omepizole >ethanol, dialysis


23 \lethemoglobin \ leth} lene blue, ,·itamin C
24 O piO ids ~aiOxO nc

25 Sa Iicylates i'!a HC03 (alkalinize urine), dialysis


26 TC s i'!a i-IC03
27 \:Varfarin ilamin K (delayed effect), fresh frozen plasm<l
28 (immediate)
29
30 FA17 p 632.1
• 31 0 2 content.
Hemoglobin Lead to tissue hypoxia from ! 0 2 saturation and !
• 32 modifications
• 33 Methemoglobin Oxidized form of l-Ib (ferric, Fe3+) that docs J\lcthemoglobinemia can be treated with
• 34 not bind 0 2 as readily, but· has f affinity for methylene blue and vi tamin C .
• 35 cyanide. litrites (eg, from dietary intake or polluted/high
Iron in Hb is normally in a reduced stale altitude water sources) and benzocaine cause
. 36
(ferrous, Fe 2+). poisoning by oxidizing Fe 2+ to Fe1+.
. 37 • \ . ................. __... ..... ,..l ..... J....: .................. : .... ............. . ................... , , •• =··" . . ....... . . . . . :,. 17- 2 + 1...: .. -1. A

a
Lock
s
Suspend
8
End Bl ock
Item: 30 of 3 7 ~ 1 • M k -<:J 1>- Jil ~· !:';-~
QIO: 3445 ..L ar Pre v ious Next Lab fli!ltues Not es Calcula t o r
A A

17
FA17 p632.1
18
19
Hemoglobin
modifications
Lead to tissue lwpoxia
. -
from l O,- saturation and l 0, content.

20
Methemoglobin Oxidized form of Hb (ferric, Fe3+) Ihat does :\lethemoglobinemia can be treated with
21 not bind 0 2 as readily, but has t affinil) for methylene blue and \itamin C.
22 cyanide. 'itrites (eg, from dietary intake or polluted/high
23 Iron in Hb is normally in a reduced state altitude \\ater sources) and benzocaine cause
24
(ferrous, Fe 2+). poisoning by oxidizing Fe2+ to Fe3+.
\ lethemoglobinemia ma} present\\ ith cyanosis Fe2 binds O~ .
+
25
and chocolate-colored blood.
26 Induced methemoglobincmi<t (using nitrites,
27 followed by thiosulfate) may be used to treat
28 cyanide poisoning.
29 Carboxyhemoglobin Form of Hb bound to CO in place of 0 2. 20
Causes l oxygen-binding capacity with left r ~atll~~bt
30
shift in oxygen-hemoglobin dissociation curve. -
~
16 v
/
• 31 E

• 32
l 0 2 un loadi ng in tissues.
CO binds competitively to Hb and with 200x
0
~
d- 12 I
1 /~
§ SOXSOHb
• 33 greater affinity than 0 2. ~
:>:: _. . . r-
g 8
• 34 CO poisoning can present with headaches, v 50% Hb (anem.at
~
/
/
• 35 dizziness, and cherry red skin. May be caused /) /
Cf 4
. 36
. 37 •
by fires, car exhaust, or gas heaters. Treat with
100% 0 2 and hyperbaric 0 2.
0
w:/
- -- ·- -- -- ·--
a
Lock
s
Suspend
8
End Bl ock
Item:31of37 ~. , . M k <:] t> al ~· ~
QIO: 5132 .l. ar Previous Next Lab 'lifllues Notes Calculator


17
A 33-yea r-old cachectic-appearing man presents to the emergency department complaining of shortness of
18 breath . His temperature is 38 .6°C (101.5°F) and respiratory rate is 30/min. X-ray of the chest revea ls
19 bilateral interstitial infi ltrates. When asked about his prior med ical history, he confesses that he has not seen
a doctor in years because he does not have hea lth insurance. Laboratory testing reveals a CD4+ cel l count of
20
110/mm 3 . He is diagnosed with pneumonia and appropr iately treated . On discharge, he is prescribed a med ication
21 as prophylaxis against future episodes of this particular type of pneumon ia.
22
23 What is the mechan ism of action of th is med ication?
24
:
25 A. Formation of free radica ls in the bacterial cell
26
B. I nhibition of dihydropteroate synthetase and dihydrofolate reductase
27
28
C. I nhibition of DNA gyrase and topoisomerase IV
29 D. I nhibition of mycol ic acid synthesis
30 E. I nhibition of protein synthesis by bind ing to 305 ribosomal subun its
· 31
F. I nhibition of viral DNA polymerase
• 32
• 33 G. I nhibition of viral uncoating
o34
o35
• 36
• 37 ~

6
lock
s
Suspend
0
End Block
Item:31of37 ~. , . M k <:] t> al ~· ~
QIO: 5132 .l. ar Previous Next Lab 'lifllues Notes Calculator


17
18 The correct answer is B. 56°/o chose this.
19 This patient, with a CD4+ cell count of 110/mm 3 , is severely immunocompromised. The pneumon ia seen in this
patient is most consistent with Pneumocystis jirovecii infection. P. jirovecii infections are t reated with
20
trimethoprim-sulfamethoxazole (TMP-SMX) . Both of these drugs interfere with folic acid metabolism:
21 trimethoprim inh ibits dihydrofolate reductase, and su lfamethoxazole, a sulfonamide that is an analog of para-
22 aminobenzoic acid (PABA), interferes with dihydropteroate synthetase . TMP-SMX is also prescribed as pr imary
23
and secondary prevention of P. jirovecii in patients with CD4+ cell counts <200/mm 3 . It is also used to prevent
Toxoplasma infection in these patients. Of note, patients with AIDS are also at risk for disseminated
24 Mycobacterium avium comp lex (MAC) infection, which may manifest with simi lar symptoms to those for P.
25 jirovecii infection, includ ing fever, cough, and weight loss. However, diarr hea is a common complaint with MAC
26 infection, and MAC is usual ly seen in patients with a CD4+ ce ll count <50/mm 3 . Azithromycin is commonly
given as a prophylactic agent to prevent MAC.
27
Sulfonamide (medicine) Azithromycin Folic acid Dihydrofolate reductase Pneumocystis jirovecii Trimethoprim/sulfamethoxazole Dihydropteroate synthase
28
Trimethoprim Sulfonamide Immunodeficiency Pneumonia Toxoplasma gondii Diarrhea Sulfamethoxazole HIV/AIDS 4-Aminobenzoic acid Pneumocystis pneumonia
29
Mycobacterium avium-intracellulare infection Metabolism Preventive healthcare Mycobacterium Fever CD4 Weight loss Dihydrofolic acid Cough
30
Mycobacterium avium complex
31
• 32 A is not correct. 4°/o chose this .
• 33
Metronidazo le acts by forming free radicals and damaging the DNA in the bacterial cell. I t is not used as
prophylaxis against Pneumocystis jirovecii infection.
o34 Metronidazole Pneumocystis jirovecii Radical (chemistry) DNA Preventive healthcare Infection
o35
C is not correct. 8°/o chose this .
• 36
The fluoroquinolones, such as ciprofloxacin, act by inhibiting DNA gyrase and topoisomerase IV. They are not
• 37 ~ ••c:<=>rl ;,c: n r nnh\lbvic: ;,n;.i nc:t Pn.:>Jtrnf"lrl!c:ric: iirf"lll.:>rii

6
lock
s
Suspend
0
End Block
Item:31of37 ~. , . M k <:] t> al ~· ~
QIO: 5132 .l. ar Previous Next Lab 'lifllues Notes Calculator


17 C is not correct. 8°/o chose this.
18 The fluoroqu inolones, such as ciprofloxacin, act by inhibit ing DNA gyrase and topo isomerase IV. They are not
19 used as prophylaxis against Pneumocystis jirovecii.
DNA gyrase Ciprofloxacin Pneumocystis jirovecii Quinolone Topoisomerase DNA Preventive healthcare
20
21 Dis not correct. 11% chose this.
22 Isoniazid, an ant itubercu lous agent, act s by inhibit ing mycolic acid synthesis. It is not used as prophylaxis
against Pneumocystis jirovecii.
23 Mycolic acid Isoniazid Pneumocystis jirovecii Pneumocystis pneumonia Preventive healthcare
24
E is not correct. 10% chose this.
25
Protein synthesis inh ibitors at the 305 ribosomal subun it include the tetracycl ines and the am inoglycosides, such
26
as gent am icin. These drugs are not used as prophylaxis against Pneumocystis jirovecii.
27 Gentamicin Pneumocystis jirovecii Protein Aminoglycoside Tetracycline antibiotics 305 Protein biosynthesis Preventive healthcare Protein synthesis inhibitor

28 Ribosome

29
F is not correct. 6°/o chose this.
30
Acyclovir, va lacyclovi r, and ganciclovir are examp les of antivira l agent s t hat act by inhibiting viral DNA
31 po lymerase, thus ha lt ing DNA synthesis. They are not used as prophylaxis aga inst Pneumocystis jirovecii.
• 32 However, ganciclovir may sometimes be used for prophylaxis against cytomegalovir us retinit is in patients wit h
• 33 CD4+ counts <50/ mm3 .
Aciclovir Cytomegalovirus Ganciclovir Cytomegalovirus retinitis Valaciclovir Pneumocystis jirovecii DNA polymerase Antiviral drug DNA Preventive healthcare
o3 4
DNA replication Retinitis CD4
o35
• 36 G is not correct. 5°/o chose this .
• 37 Amantadine, cu r rently used primarily as an anti-pa r kinson ian agent, was once used to t reat influenze by
~ • I • I • . • • I • • I o • • I ,. • , ., A ..,. , • I I I • •

6
lock
s
Suspend
0
End Block
Item:31of37 ~. , . M k <:] t> al ~· ~
QIO: 5132 .l. ar Previous Next Lab 'lifllues Notes Calculator


17 Protein synthesis inh ibitors at the 305 ribosomal subunit include the tetracyclines and the aminog lycosides, such
18 as gentamicin. These drugs are not used as prophylaxis aga inst Pneumocystis jirovecii.
Gentamicin Pneumocystis jirovecii Protein Aminoglycoside Tetracycline antibiotics 305 Protein biosynthesis Preventive healthcare Protein synthesis inhibitor
19
Ribosome
20
21 F is not correct. 6°/o chose this.
22 Acyclovir, valacyclovi r, and ganciclovir are examp les of antiviral agents that act by inhibiting viral DNA
po lymerase, thus halting DNA synthesis. They are not used as prophylaxis against Pneumocystis jirovecii.
23
However, ganciclovir may sometimes be used for prophylaxis against cytomegalovir us retinitis in patients with
24 CD4+ counts <50/mm3 .
25 Aciclovir Cytomegalovirus Ganciclovir Cytomegalovirus retinitis Valaciclovir Pneumocystis jirovecii DNA polymerase Antiviral drug DNA Preventive healthcare

26 DNA replication Retinitis CD4

27 G is not correct. 5°/o chose this.


28 Amantadine, cu r rently used primarily as an anti-pa r kinsonian agent, was once used to treat influenze by
29 inhibiting vi ral uncoating by targeting the M2 protein of influenza A. It is not used as prophylaxis against
30
Pneumocystis jirovecii.
Amantadine Pneumocystis jirovecii M2 proton channel Influenza Protein Preventive healthcare Virus Antiparkinson medication
31
• 32
• 33 Bottom Line:
o34 Pneumonia in an immunocompromised patient is concern ing for Pneumocystis jirovecii pneumonia, wh ich is
o35 treated and prevented with the use of trimethopr im-su lfamethoxazole .
Pneumonia Pneumocystis jirovecii Trimethoprim/sulfamethoxazole Immunodeficiency Pneumocystis pneumonia
• 36
• 37 ~

6
lock
s
Suspend
0
End Block
Item: 31 of 3 7 ~ 1 • M k -<:J 1>- Jil ~· !:';-~
QIO: 5132 ..L ar Pre v ious Next Labfli!llues Notes Calcula t o r
A A

17 FA17p150.1
18
Pneumocystisjirovecii Causes Pneumocystis pneumonia (PCP), a diffuse interstitial pneumonia r.J. Yeast-like
19
fungus (originally classified as protozoan). Inhaled. \lost infections are asymptomatic.
20 Immunosuppression (eg, AIDS) predisposes to disease. Diffuse, bilateral ground-glass opacities on
21 CXR/CT (l]. Diagnosed b) lung biops) or lavage. Disc-shaped yeast seen on methenamine siher
22 stain of lung tissue ~-
Treatment/prophylaxis: T~ I P-S IX, pentamidine, dapsone (prophylaxis only), atovaquonc. Start
23
prophylaxis when CD4+ count drops to < 200 cells/mm3 in HIV patients.
24
25
26

27
28
29
30
31
• 32

• 33
• 34
FA17 p 190.1
o35
• 36
Sulfonamides Sulfamethoxazole (S~ifX), sulfisoxazole, PABA t Pteridine

• 37
sulfadiazine.
Oihyw"~" ""c II-A- Sulfonamides,

a
Lock
s
Suspend
8
End Bl ock
Item: 31 of 3 7 ~ 1 • M k -<:J 1>- Jil ~· !:';-~
QIO: 5132 ..L ar Pre v ious Next Labfli!llues Notes Calcula t o r
A A

17 FA17 p 190.1
18
Sulfonamides Sulfamethoxazole (SMX), sulfi soxazole,
19
20
MECHANISM
sulfadiazine.
Inhibit dihydropteroate synthase, thus inhibiting
Oihyn uv uo
n ""
jt-0-
PABA+ Pteridine

Sulfonamides.
dapsone
21 folate s~ nthesis. Bacteriostatic (bactericidal
Dihydropteroic acid
22 when combined with trimethoprim).
23 CLINICAL USE Gram$. gram 8, 'ocardia. Sr-. IX for simple
24 UTI.
25 ADVERSE EFFECTS Hypersensiti\1ty reactions, hemolysis if G6PD
26 deficient, nephrotoxicity (tubulointerstitial
D•hydrofolic acid
27
28
nephritis), photosensitivity, Ste,·ens-Johnson
syndrome, kern icterus in infants, displace
other drugs from albumin (eg, warfarin).
D ywof,,a •
reductase
jl-0-- Trimethoprim.
pyrimethamine
29
MECHANISMOF RESISTANCE Altered enzyme (bacterial dihyclropteroatc Tetrahydrofolic acid
30
31
synthase), l uptake, or t PAB synthesis. /!""-.
Thymidine Methionine
Purines
• 32
!
DNA. RNA
!
DNA
!
Protein
• 33
• 34
o35 FA17 p 194.2
• 36 Prophylaxis in HIV patients
CELL COUNT PROPHYLAXIS INFECTION
• 37

a
Lock
s
Suspend
8
End Bl ock
Item: 31 of 3 7 ~ 1 • M k -<:J 1>- Jil ~· !:';-~
QIO: 5132 ..L ar Pre v ious Next Labfli!ltues Notes Calcula t o r

17
A
fola te synthesis. Bacteriostatic (bactericida I ~ A

Dihydropteroic acid
18
when combined with trimet hoprim).
19 CliNICAl USE Gram $, gram 8, Nocardia. S I for simple
20
UTI.

21 ADVERSEEFFECTS Hypersensiti,·ity reactions, hemolysis if G6PD


deficient, nephrotoxicity (tubulointerst itial
22 Olhydrofolic add
nephritis), photosensitivity, Ste\'ens-Johnson
23
24
S} ndrome, kernicterus in infants, displace
other dmgs from albumin (eg, warfarin).
D 1.. .... "
rf'duc•a. e
, jl-0---
-
Trimelhoprim.
pyrimethamine

25 MECHANISMOF RESISTANCE Altered enzyme (bacterial dih}dropteroate Tetrahydrololic add


synthase), l uptake, or t PABA synt hcsis.
26

27
/!""
Purines Thymidine Methionine

28 l l
DNA, RNA
l DNA Protein
29
30
31 FA17 p 194.2
Prophylaxis in HIV patients
• 32
CEll COUNT PROPHYLAXIS INFECTION
• 33
C04 < 200 cells/mm3 TMP-SMX Pneumocystis pneumon ia
• 34
C04 < 100 cells/mm3 T.\IIP-SMX Pneumocystis pneumonia and toxoplasmosis
o35
C04 < 50 cells/mm3 Azithromycin or clarithromyein J\lrcob(ICierium avium complex
• 36
• 37

a
Lock
s
Suspend
8
End Bl ock
Item: 32 of 37 ~. I • M k <:] t> al ~· ~
QIO: 2486 .l. ar Previous Next lab 'lifllues Notes Calculator


17
A 59-yea r-old healthcare worker who recent ly returned from a month- long vo lunteer commitment in a sma ll
18 cl inic in central India is found to have a Mantoux test of 12 mm indu ration. Subsequent chest x-ray shows a
19 right-sided perihi lar infiltrate with ipsilateral hi lar adenopathy, but she is asymptomatic. The decision is made
to put her on isoniazid therapy. One week later, she returns to her doctor because she had been fee ling poorly for
20
several days and is exper iencing confusion and slurred speech . The physician discovers that the patient has been
21 taking four times the recommended dose of ison iazid . Her physician leaves the examination room briefly and
22 returns to find the patient having a seizure.
23
24 Following stabilization, administration of which of the fol lowing drugs is the most appropriate next step?
25 :
26 A. Levetiracetam
27 B. Phenobar bita l
28
C. Pyrazinamide
29
30 D. Pyridoxine
31 E. Rifampin
• 32
F. Thiamine
• 33
o34
o35
• 36
• 37 ~

6
lock
s
Suspend
0
End Block
Item: 32 of 37 ~. I • M k <:] t> al ~· ~
QIO: 2486 .l. ar Previous Next lab 'lifllues Notes Calculator

• The correct answer is D. 63°/o chose this .


17
18
I soniazid wor ks by inhibiting mycolic acid synthesis, a key component of mycobacteria l cel l wal ls. It also is a
competitive antagonist of pyridoxa l kinase, an essential component in the biosynthesis of GABA from vitamin B6
19
(pyr idoxine). Reduced levels of GABA, an inhibito ry neu rotransmitter, can promote seizu res. Periphe ral
20 neuropathy and hepatitis can also resu lt. Isoniazid is given monotherapy to prevent active tube rculosis in
21 someone found to have latent tuberculosis, or as a part of a multidrug regimen (often rifampin, isoniazid,
pyrazinamide, and ethambuto l, or RI PE therapy) fo r active tuberculosis. Administration of pyridoxine can both
22
prevent and reverse these effects.
23 Isoniazid Ethambutol Mycolic acid Pyrazinamide Rifampicin Pyridoxine Peripheral neuropathy Gamma-Aminobutyric acid Tuberculosis Neurotransmitter

24 latent tuberculosis Pyridoxal kinase Hepatitis Mycobacterium B vitamins Vitamin 812 Vitamin Biosynthesis Epileptic seizure Combination therapy
25
A is not correct. 6°/o chose this.
26
Although levetiracetam is an antiepileptic, ison izaid toxicity causing seizu res shou ld be t reated with both
27 intravenous (IV) administration of pyridoxine (to overcome functional pyr idoxine deficiency) and a
28 benzodiazepine, wh ich acts to overcome the GABA deficiency. Leveti racetam's mechanism of action is relative ly
29 unexplained, although it is known to bind to the synaptic vesicle protein SV2A and has not been associated with
GABA potentiation. Therefo re, levetiracetam would not be fi rst-line treatment.
30 Benzodiazepine Synaptic vesicle levetiracetam Pyridoxine Gamma-Aminobutyric acid Anticonvulsant SV2A Epileptic seizure Protein Mechanism of action Toxicity
31
Intravenous therapy Vesicle (biology and chemistry) Synapse
32
• 33
B is not correct. 9°/o chose this .
Phenobar bita l, like diazepam, is GABAergic and would be ind icated for active seizu res.
o3 4
Diazepam Phenobarbital Gamma-Aminobutyric acid GABAergic Epileptic seizure
o35
• 36
C is not correct. 6°/o chose this .
Pyrazinamide is an antimycobacte ria l prodrug that is converted to the active compound pyrazinoic acid, which
• 37 ~
c.·--~: --- :- L.--~ -L.--- 1··------- T• ···-· .1..1 --~ :- - . . -··- ~L...- ··=
•--=- o ...1-C:-:---·· . . --· .1 .. :-- c .... __ -- =--- =--:.. 1
6
lock
s
Suspend
0
End Block
Item: 32 of 37 ~. I • M k <:] t> al ~· ~
QIO: 2486 .l. ar Previous Next lab 'lifllues Notes Calculator
- . -- -- . - - -- -

17 Phenobar bita l, like diazepam, is GA8Aergic and wou ld be ind icated for active seizures.
18 Diazepam Phenobarbital Gamma-Aminobutyric acid GABAergic Epileptic seizure

19
C is not correct. 6°/o chose this.
20 Pyrazinamide is an antimycobacteria l prodrug that is converted to the active compound pyrazinoic acid, wh ich
21 functions in host phagolysosomes. It would not improve the vitamin 8 6 deficiency resulting from an isoniazid
22 overdose.
Isoniazid Prodrug Pyrazinoic acid Pyrazinamide Vitamin 812 B vitamins
23
24 E is not correct. 7°/o chose this.
25 Rifampin is an antimycobacterial agent that inhibits DNA-dependent RNA polymerase . It wou ld have no effect on
the vitamin 8 6 deficiency resulting from an isoniazid overdose .
26 Isoniazid Rifampicin Vitamin 812 B vitamins RNA polymerase RNA Vitamin
27
F is not correct. 9°/o chose this.
28
Thiamine (vitamin 8 1) is indicated for treatment of Wern icke-Korsakoff synd rome and beriberi, conditions of
29
thiamine deficiency. It is also often deficient in alcoholics . It wou ld have no effect on t reating isoniazid
30 overdose.
31 Wernicke-Korsakoff syndrome Beriberi Isoniazid Thiamine Thiamine deficiency Alcoholism

32
• 33
Bottom Line:
o34
I soniazid decreases the production of vitamin 8 6 (pyr idoxine), which is often given as a supplement to people
o35 on isoniazid to prevent neurologic complications.
• 36 Isoniazid Pyridoxine Vitamin 812 B vitamins Vitamin Neurology

• 37 ~

6
lock
s
Suspend
0
End Block
Item: 32 of 3 7 ~ 1 • M k -<:J 1>- Jil ~· !:';-~
QIO: 2486 ..L ar Pre v ious Next Lab fli!ltues Notes Calcula t o r
A A

17
FA17 p 193.1
18
Isoniazid
19
MECHANISM l synthesis of mycolic acids. Bacterial catalase-
20
peroxidase (encoded by KatG) needed to
21 com·ert INI-I to acti,·e metabolite.
22 CliNICAL USE i\lycobacterium tuberculosis. The only agent Different I. H half-liYes in fast ,.s slow
23 used as solo prophylaxis against TB. Also used aceh·lators.
24 as monotherapy for latent TB.
25 ADVERSE EFFECTS Hepatotoxicity, P-4)0 inhibition, drug-induced 1~1 I Injures ~eurons and llepatocytes.
26 SLE, anion gap metabolic acidosis,' itamin
B6 deficiency (peripheral neuropathy,
27
sideroblastic anemia). Administer with
28
pyridoxine (8 6).
29
MECHANISM OF RESISTANCE Mutations leading to underexpression of KatC.
30
31
FA17 p 63.4
32 Vitamin 8 6 (pyridoxine)
• 33
FUNCTION Converted to pyridoxal phosphate (PLP), a cofactor used in transamination (eg, ALT and AST),
• 34 decarboxylation reactions, glycogen phosphorylase. Synthesis of cystathionine, heme, niacin,
o35 histamine, and neurotransmitters including serotonin, epinephrine, norepinephrine (NE),
• 36 dopamine, and CABA .
• 37 DEFICIENCY Convulsions, hyperirritability, peripheral neuropathy (deficiency inducible by isoniazid and oral

a
Lock
s
Suspend
8
End Bl ock
Item: 32 of 3 7 ~ 1 • M k -<:J 1>- Jil ~· !:';-~
QIO: 2486 ..L ar Pre v ious Next Lab fli!ltues Notes Calcula t o r
A A

17
18 FA17 p63.4
19 Vitamin 8 6 (pyridoxine)
20 FUNCTION Converted top) ridoxal phosphate (PLP), a cofactor used in transamination (eg, ALT and AST),
21 decarboxylation reactions, glycogen phosphorylase. Synthesis of cystathionine. heme, niacin,
histamine, and neurotransmitters including serotonin, epinephrine, norepinephrine (NE),
22
dopamine, and CABA.
23
DEFICIENCY Convulsions, hrperirritability, peripheral neuropathy (deficiencr inducible by isoniazid and oral
24
contracepti,·es), sideroblastic anemias due to impaired hemoglobin synthesis and iron excess.
25
26
FA17 p 514.1
27 Epilepsy drugs
28 GENERALIZED
29
30 ~
"'
=>
~
v
-
.,.. ~
~ :.~
~~
31
~ ~ e; MECHANISM SlDEEFFECTS NOTES
32 •
Ethosuximide Blocks thalamic T-type C32+ I•:FC IIIJ - Io:thosuximide Sucks to have Silent
• 33 ./ channels causes Fatigue. G I distress, (absence) Seizures
lleadache, Itching (and
• 34
urlicaria). and Stevens-
o35 Johnson syndrome
• 36 Benzodiazepines •• t CABA.\ aclion Sed3tion, tolerance, Also for eclampsia sci1urcs (lsi
(eg, diazepam, dependence, respiratory line is lgSO~)
• 37 lorazeoam. de pre~ion

a
Lock Suspend
s 8
End Bl ock
Item: 32 of 3 7 ~ 1 • M k -<:J 1>- Jil ~· !:';-~
QIO: 2486 ..L ar Pre v ious Next Lab fli!ltues Notes Calcula t o r
A A

17 FA17 p514.1
18 Epilepsy drugs
19 GENERALIZED
~
20 <
....... !::! V'>

21
0
!:
:z:
0
~ ...:z..... =>
.....
.......
...... ..., ;::::
~
< v =>::h
;:::: ,_ ~
:z: V'>
-
<0..
22 ""
~ ,_
0
<
,_
V'> ... MECHANISM SIDE EFFECTS NOTES
23 Ethosuximide • Blocks thalamic T-type Ca~· 1•:1 G IIIJ- F thosuximide Sucks to have Silent
24 ./ channels causes Fatigue. G l distress. (absence) Seizures
lleadache. Itching (and
25 urticaria). and Ievens-
26
Johnson synd rome
Benzodiazepines •• t C .\BA.\ action Sedation, tolerance, Also for eclampsia sci7llrcs (lsi
27 ./ dependence, respiratory line is lg$04)
(eg, diazepam,
28 lorazepam, dcprc~sion
midazolam)
29
Phenobarbital ./ ./ tC BA~ action Sedation, tolerance, lst line in neonates
30 dependence, induction
31 of cytochrome P--150,
cardiorespi ratory depression
32 • u•
Phenytoin, ./ Blocks a+ channels; 1ero- \Jcurologic: nystagmus, di plopia, ataxia, sedation, peri pheral
• 33 fosphenytoin ./ ./ order kinetics neuropathy. Ocrmatologic: hirsutism, Stc1·cns-)ohnson
syndrome, gingival hyperplasia, DRESS syndrome.
• 34
l\ lnsculoskeletal: osteopeni:J, SLE-Iike syndrome. llematologic:
o35 megaloblastic anemia. Reproductive: tera togenesis (fetal
hydantoin S) ndrome). Other: C)tochrome P-4;0 induction
. 36
Carbamazepine ./ Blocks a• channels Diplopia, ata~ia, blood 1st line for trigeminal neuralgia
. 37 • ./ d1 M:r:tiia~ (agranulocvlo~is •

a
Lock Suspend
s 8
End Bl ock
Item: 32 of 3 7 ~ 1 • M k -<:J 1>- Jil ~· !:';-~
QIO: 2486 ..L ar Pre v ious Next Lab fli!ltues Notes Calcula t o r

17
A

Carbamazepine • ./ Blocks Na+ channels Diplopia, atm..ia, blood 1st line for trigeminal neuralgia
A

./ d) scra~ia~ (agranulocytosis,
18 aplastic anemia), liver
19 toxicit). teratogenesis.
induction of C) tochrome
20 P-·-150, SIADII, Ste1 ens-
21 johnson s1 ndrome
22 Valproic acid ./ • ./ I '\a+channel inactil·ation, C l distress. rare but futal Also used for myoclonic seizures.
./ I CA BA concentration hepatotoxicity (measure bipolar disorder, migraine
23 by inhibiting CABA LFTs), pancreatitis, neural prophylaxis
transaminase tube defects, tremor, weight
24
gain, contraindicated in
25 pregnancy
26 Vigabatrin t CABA by irrcversibl) Perm.111cnt visual loss (black
inhibiting C.\B.\ bo' liMning)
27 transaminase
28 Gabapentin ./ Primaril) inhibits high-voltage- Sedation, ata,ia Also used for peripheral
activated Ca 2+channels; neuropathy, postherpetic
29
designed as CABA analog neuralgia
30 Topiramate ./ ./ Blocks a+ channels, I C BA Sedation, mental dulling, Also used for migraine
31 action kidney stones, weight loss, prevention
glaucoma
32
Lamotrigine ./ ./ ./ Blocks voltage-gated a' Stevens-Johnson syndrome
• 33 channels, inhibits the release (must be til ra ted slowly)
• 34
of glutamate
Levetiracetam ./ ./ Unknown; may modulate l•:1t igue, dro11siness .
o35 CABA and glutamate release headache, neuropsrchiatric
. 36 S\ mptoms {eg, personality
changes)
. 37 • "9"'1 - - - L I - -
, • ,....-. n--. 1 • • •1 • •• • I

a
Lock
s
Suspend
8
End Bl ock
Item: 32 of 3 7 ~ 1 • M k -<:J 1>- Jil ~· !:';-~
QIO: 2486 ..L ar Pre v ious Next Lab fli!ltues Notes Calcula t o r
A
induction of C) tochrome A

17
P-450, Slt\011, Ste\ens-
18 Johnson syndrome
19 Valproic acid ./ • ./ I 'a+ channel inactivation, C l distress. rare but fatal Also used for myoclonic seizures,
./ I GA BA concentration hepatotoxicity (measure bipolar disord~r, migraine
20 by inhibiting GABA LFTs), pancreatitis, neural proph}iaxis
21 transaminase tube defects, tremor, weight
gain, contraindicated in
22 pregnancy
23 Vigabatrin I GA BA b} irrc\'crsibl) Perm.menl 'isuallos~ (black
inhibiting CAB.\ bo' "arning)
24 transaminase
25 Gabapentin ./ Primaril} inhi~its high-,oltage- Sedation. ata,ia Also used for peripheral
26 acti\'ated Ca-+ channels; neuropathy, postherpetic
designed as CABA analog neuralgia
27
Topiramate ./ ./ Blocks •a+ channels, I C \B!\ Sedation, mental dulling . Also used for migraine
28 action kidney ~ Iones, weight loss, prevention
gbucoma
29
Lamotrigine ./ ./ ./ Blocks voltage-gated 1 a' Stevens-Johnson syndrome
30 channels, inhibits the release (must be titra ted slowly)
31 of glutamate
Levetiracetam ./ ./ Unknown; may modulate ~;11 igue,
drowsiness,
32
CABA and glutamate release headache, neuropsychiatric
• 33 svmptoms (eg, personality
changes)
• 34
Tiagabine ./ I CABA by inhibiting rcuptake
o35
= 1st line; = lst line for acute; ••• = lst line for prophylaxis.
. 36
. 37 •

a
Lock
s
Suspend
8
End Bl ock
Item: 33 of 37 ~ 1 • M k -<:J 1>- Jil ~· !:';-~
QIO: 3250 ..L ar Pre v ious Next Labfli!llues Notes Calcula t o r

IAA]
A A

17
A 59-year-old woman with history of asthma on med ical treatment returns to her primary care physician w ith
18 a complaint of worsening symptoms despite diligent use of her steroid inhaler. Her physician decides to add a
19 drug that works by dilating her bronchioles for up to 12 hours. Before prescribing this drug, however, the
20
physician expla ins some of the possible adverse effects, including muscle tremors and palpitations.

21
Which of the following medications is the physician adding?
22
23 :
A . Albuterol
24
25 B. Cromolyn
26 C. Ipratropium
27
D. Montelukast
28
29
E. Salmeterol

30
31
32
• 33
• 34
o35
. 36
. 37 •

a
Lock
s
Suspend
8
End Bl ock
Item: 33 of 37 ~. I • M k <:] t> al ~· ~
QIO: 3250 .l. ar Previous Next Lab 'lifllues Notes Calculator


17
The correct answer is E. 56°/o chose this.
18
Salmeterol is a long-acting ~ragonist that can be used for asthma prophylaxis . It activates the ~r
19 adrenoceptor, wh ich ultimately causes smooth muscle relaxation and bronchodilation. Known adverse effects
20 include tremor and arrhythmia. It is important to note that long-acting ~radrenergic agon ists increase the risk
21 of asthma -related death. Salmeterol is contraind icated when used without a concomitant long -term asthma
control med ication (eg, inha led corticosteroid) for the treatment of asthma
22 Corticosteroid Salmeterol Bronchodilator Beta2-adrenergic agonist Asthma Tremor Smooth muscle tissue Cardiac arrhythmia Preventive healthcare Muscle
23
Pharmaceutical drug Contraindication
24
A is not correct. 13% chose this.
25
Albuterol is a short-acting ~-agonist used as a rescue inhaler. It works to relax airway smooth muscle similarly
26
to sa lmeterol. The half-life of albuterol is in the range of 3-6 hou rs. Albuterol is first-line treatment used in
27 symptomatic asthmatics, and any patient on a steroid inhaler can be assumed that this patient is already ta king
28 albuterol.
Salmeterol Half-life Smooth muscle tissue Salbutamol Steroid Inhaler Respiratory tract Muscle
29
30 B is not correct. 6°/o chose this.
31 Cromolyn stabi lizes mast cel ls and prevents the release of inflammatory agents, such as histamine. Although
32
cromolyn prevents acute bronchoconstriction during asthmatic episodes, it is not a potent bronchod ilator.
Furthermore, its side effect profile is relatively favorable, sometimes resulting in mi ld local ir ritation or cough,
33 wh ich diverges from the severe side effects seen in th is presentation .
0 34 Bronchodilator Cromoglicic acid Histamine Bronchoconstriction Mast cell Cough Side effect Asthma Inflammation

0 35
C is not correct. 18% chose this.
0
36 I pratropium is a muscarinic antagonist that competitively blocks muscarinic receptors, preventing
0 37 ~
bronchoconstriction. Adverse effects include couqh, anxiety, dry mouth, and nausea. This druq is more often ~

6
lock
s
Suspend
0
End Block
Item: 33 of 37 ~. I • M k <:] t> al ~· ~
QIO: 3250 .l. ar Previous Next Lab 'lifllues Notes Calculator


~ ~ - .. - ~

17 Albuterol is a short-acting ~-agon i st used as a rescue inha ler. It works to relax ai rway smooth muscle similarly
18 to sa lmeterol. The half-life of albutero l is in the range of 3-6 hou rs. Albuterol is first-line treatment used in
19 symptomatic asthmatics, and any patient on a steroid inhaler can be assumed that this patient is already ta king
albuterol.
20
Salmeterol Half-life Smooth muscle tissue Salbutamol Steroid Inhaler Respiratory tract Muscle
21
B is not correct. 6°/o chose this.
22
Cromolyn stabi lizes mast cel ls and prevents the release of inflammatory agents, such as histamine. Although
23
cromolyn prevents acute bronchoconstriction during asthmatic episodes, it is not a potent bronchod ilator.
24 Furthermore, its side effect profile is relatively favorab le, sometimes resulting in mi ld loca l ir ritation or cough,
25 w hich diverges from the severe side effects seen in th is presentation .
Bronchodilator Cromoglicic acid Histamine Bronchoconstriction Mast cell Cough Side effect Asthma Inflammation
26
27 C is not correct. 18% chose this.
28 I pratropium is a muscarinic antagonist that competitively blocks muscarinic receptors, preventing
29
bronchoconstriction. Adverse effects include cough, anxiety, dry mouth, and nausea. This drug is more often
used to treat exacer bations of chronic obstructive pulmonary disease but can be used to treat asthma as we ll.
30 Chronic obstructive pulmonary disease Muscarinic antagonist Ipratropium bromide Bronchoconstriction Asthma Receptor antagonist
31 Muscarinic acetylcholine receptor Nausea Xerostomia Anxiety Cough Receptor (biochemistry)
32
D is not correct. 7°/o chose this.
33
Montelukast is a leukotriene receptor antagonist that is used as a maintenance medication for the treatment of
34
0

asthma and works by reducing bronchoconstriction and decreasing inflammation. The ma in adverse effects that
0 35 have been associated with montelukast are GI upset and headaches.
0
36 Antileukotriene Montelukast Leukotriene Bronchoconstriction Asthma Receptor antagonist Inflammation Receptor (biochemistry) Pharmaceutical drug

0 37 ~

6
lock
s
Suspend
0
End Block
Item: 33 of 37 ~. I • M k <:] t> al ~· ~
QIO: 3250 .l. ar Previous Next Lab 'lifllues Notes Calculator
• I I - • . • I -

17
Cromolyn stabi lizes mast cel ls and prevents the release of inflammatory agents, such as histamine. Although
18 cromolyn prevents acute bronchoconstriction during asthmatic episodes, it is not a potent bronchod ilator.
19 Furthermore, its side effect profile is relatively favorab le, sometimes resulting in mi ld loca l ir ritation or cough,
20 wh ich diverges from the severe side effects seen in th is presentation .
Bronchodilator Cromoglicic acid Histamine Bronchoconstriction Mast cell Cough Side effect Asthma Inflammation
21
22 C is not correct. 18% chose this.
23 I pratropium is a muscarinic antagonist that competitively blocks muscarinic receptors, preventing
bronchoconstriction. Adverse effects include cough, anxiety, dry mouth, and nausea. This drug is more often
24
used to treat exacer bations of chronic obstructive pulmonary disease but can be used to treat asthma as we ll.
25 Chronic obstructive pulmonary disease Muscarinic antagonist Ipratropium bromide Bronchoconstriction Asthma Receptor antagonist

26 Muscarinic acetylcholine receptor Nausea Xerostomia Anxiety Cough Receptor (biochemistry)

27
D is not correct. 7°/o chose this.
28
Montelukast is a leukotriene receptor antagonist that is used as a maintenance medication for the treatment of
29 asthma and works by reducing bronchoconstriction and decreasing inflammation. The ma in adverse effects that
30 have been associated with montelukast are GI upset and headaches.
Antileukotriene Montelukast Leukotriene Bronchoconstriction Asthma Receptor antagonist Inflammation Receptor (biochemistry) Pharmaceutical drug
31
32
33 Bottom Line:
0 34 Salmeterol is a long-acting ~ - ad renerg i c inhaled drug used to t reat moderate to severe asthma . Adverse
0 35 effects common to this class of drugs include muscle t remors and heart palpitations.
Salmeterol Asthma Palpitations Muscle
0
36
0 37

6
lock
s
Suspend
0
End Block
Item: 33 of 37 ~ 1 • M k -<:J 1>- Jil ~· !:';-~
QIO: 3250 ..L ar Pre v ious Next Labfli!llues Notes Calcula t o r
A A

17
FA17 p649.1
18
19 Asthma drugs Bronchoconstriction is mediated b~ (I) inOammalory processes and (2) parasympathetic tone;
therapy is directed at these 2 pathways.
20
21
~-ag onists Albute rol-relaxes bronchial smooth muscle (short acting ~-agonist). Used during acute
exacerbation.
22
Salmete rol, fo rmote ro l- long-acting agents for prophylaxis. Ad\'erse effects are tremor and
23
arrh' thmia.
24
Inhaled Fluticasone, budesonide - inhibit the synthesis of \'irtually all cytokines. Inacti,·ate NF-KB, the
25
cortico ste roids transcription factor that induces production ofT 'F-a and other inflammatory agents. ht-line
26 therapy for chronic asthma. Vlay cause oral thrush.
27 Muscarinic Tiotro pium, ipratro pium-competitivel) bloc!.. muscarin ic receptors, preventing
28 a ntago nists bronchoconstriction. Also used for COPD. Tiotropium is long acting.
29 Antile ukotrienes Monte lukast, zafirlukast - bloc!.. lcul..ot ricnc Exposure to antigen
30 receptors (CysLTl). Especially good for (dust pollen, etc)

31 aspirin-induced and exercise-induced asthma.


32
Zileuton -5-lipoxygenase pathway inhibitor. t-0- Avoidance
Blocks conversion of arachidonic acid to
33
leukotrienes. Hepatotoxic.
• 34
Anti-lgE mo noclo nal Omalizumab-binds mostly unbound serum
Antigen and lgE t-0- Omalizumab
on mast cells
o35 the rapy lgE and blocks binding to Fc£RI. Used in
• 36 allergic asthma with t lgE b ·cls resistant to
...,.0..... Steroids
• 37 inhaled steroids and long-acting ~-agoni sts .

a
Lock
s
Suspend
8
End Bl ock
Item: 33 of 37 ~. I • M k <:] t> al ~· ~
QIO: 3250 .l. ar Previous Next Lab 'lifllues Notes Calculator

• •
17
18 FA17p234.1
19 Sympathomimetics
20 DRUG ACTION APPLICATIONS

21 Direct sympathomimetics
22 Albuterol, salmeterol ~z> ~~ Albuterol for acute asthma or COPO. Salmeterol
23
for long-term asthma or COPO control.
24 Dobutamine ~~>~'a Heart failure (llF) (inotropic > chronotropic),
cardiac stress testing.
25
26
Dopamine 0 1 = 0 2 >~ > a Unstable bradycardia, HF, shock; inotropic and
chronotropic effects at lower doses due to ~
27
effects; vasoconstriction at high doses due to a
28 effects.
29 Epinephrine ~>a Anaphylaxis, asthma, open-angle glaucoma;
30 a effects predominate at high doses.
31 Significantly stronger effect at ~-receptor than
32 norepinephrine.
33 Fenoldopam 01 Postoperative hypertension, hypertensive crisis.
0 34
Vasodilator (coronary, periphera l, renal, and
splanch nic). Promotes natriuresis. Can cause
0 35
hypotension and tachycardia.
0 36
Isoproterenol ~I= ~ Electrophysiologic evaluation of
0 37 • t-:J t'h \1-:t rrh \l l·hn'l;'l (' r-:~n UJI\r<:'~l"' iC'I"l"''Pn'\i'l •

6
lock
s
Suspend
0
End Block
Ite m: 33 o f 3 7 ~ 1 • M k -<:J 1>- Jil ~· !:';-~
QIO: 325 0

17
..L
. .. . ..
ar
..
Previous Next
.. ..
La b fli!llues Not es Calculator
-
(mydriatic), rhinitis (decongestan t).
18
Indirect sympathomimetics
19
Amphetamine Indirect general agonist, reuptake inhibitor, also 'arcolepsy. obesity, ADHD.
20
releases stored catecholamines
21
Cocaine Indirect general agonist, reuptake inhibitor Causes ,·asoconstriction and local anesthesia.
22 1 e\·er gi,·e ~-blockers if cocaine intoxication is
23 suspected (can lead to unopposed cx1 acti\'alion
24 and extreme hypertension).
25 Ephedrine Indirect general agonist, releases stored Nasal decongestion {pseudoephedrine), urinar}
26 catecholamines incontinence, hrpotension.

27
28 FA17 p 387.5
29 Mast cell Mediates allergic reaction in local tissues. Invok ed in type I hypersensitivity reactions.
30 Mast cells contain basophilic granules · and Cromolyn sodium prevents mast cell
31 originate from the same precursor <IS basophi ls degranulation (used for asthma prophylaxis).
but are not the same cell type. Can hind the
32
Fe portion of IgE to membrane. lgE cross-
33
links upon antigen binding- degranulation
• 34 - release of histamine, heparin, tryptase, and
o35 eosinophil chemotactic factors.
. 36
. 37 •

a
Lock
s
Suspend
8
End Block
Item: 34 of 37 ~. I • M k <:] t> al ~· ~
QIO: 1337 .l. ar Previous Next Lab 'lifllues Notes Calculator


17
A 25-yea r-old Vietnamese man presents with new-onset hemoptysis for the past 12 hours. The patient, who
18 recently immigrated to the Un ited States from Vietnam, has had fever and night sweats on a da ily basis for
19 the past 4 years. He has no other complaints and no past med ica l history. The patient is diagnosed with
active tubercu losis and sent home on an antimycobacteria l reg imen . One month later, the patient returns with new
20
comp laints of joint pain, photosensit ivity, and a facia l rash. Liver function tests are found to be elevated .
21
22
Which of the fo llowing is the mechan ism of action of the dr ug with the adverse effects described above'
23
:
24
A. Disr upts the cell membrane's osmotic properties
25
B. Inhibits arabinosyl transferases
26
27 C. Inhibits DNA-dependent RNA polymerase encoded by the rpo gene
28 D . Inhibits folic acid synthesis
29
E. Inhibits synthesis of myco lic acids
30
31
32
33
0 34
0 35
0 36
0 37 ~

6
lock
s
Suspend
0
End Block
Item: 34 of 37 ~. I • M k <:] t> al ~· ~
QIO: 1337 .l. ar Previous Next Lab 'lifllues Notes Calculator


17
The correct answer is E. 54°/o chose this.
18
Of the antimycobacterial drugs, only ison iazid (I NH) produces the lupus-l ike synd rome described in the vignette.
19 INH decreases synthesis of mycolic acids. Further more, hepatotoxicity is common to many antitubercu losis
20 drugs ( rifampin, pyrazinamide, and INH ) . Peripheral neuropathy is a potential side effect of I NH use. The
21
likelihood of the neu ropathy can be minimized with vitamin B6 supplementation .
Isoniazid Pyrazinamide Hepatotoxicity Rifampicin Peripheral neuropathy Tuberculosis management B vitamins Mycolic acid Vitamin Vitamin 812 Tuberculosis
22
Adverse effect
23
24 A is not correct. 7°/o chose this.
25 Disr uption of the ce ll membrane's osmotic properties describes the mechanism of action of po lymyxins.
Polymyxins are not part of the treatment for TB.
26 Cell membrane Polymyxin Osmosis Biological membrane
27
B is not correct. 13% chose this.
28
Ethambutol, not ison iazid, inh ibits the arabinosyl transferase-med iated synthesis of arabinogalactin for
29
mycobacterial cel l wal ls. The side effects of ethambuto l include dose-dependent visual disturbances, decreased
30 visua l acu ity, red-g reen color blindness, optic neuritis, and retina l damage.
31 Optic neuritis Isoniazid Ethambutol Color blindness Visual acuity Retinal

32 C is not correct. 15% chose this.


33 The mechanism of action here is that of rifampin, another antimycobacteria l drug. Rifampin's side effects profile
34 does include hepatotoxicity (though this is a ra re side effect), but also includes GI effects ( nausea, vomiting,
diarr hea ), hemato log ic side effects (thrombocytopenia, neutropenia, acute hemolytic anemia) and CNS effects
o35
( headache, fever) . Rifampin does not, however, cause lupus- like sy mptoms as described in th is case .
• 36 Neutropenia Hepatotoxicity Rifampicin Thrombocytopenia Anemia Diarrhea Mechanism of action Adverse effect Nausea Hemolytic anemia Headache Vomiting
• 37 ~ C:irl.o .off.ort brlv.orc:.o rln•n r.o;:::artinn riOntr::::al n.orvnliC: c:vc:t.om l-l.omnlvc:ic: l-l.om;:::atnlnnv ~.ov.or

6
lock
s
Suspend
0
End Block
Item: 34 of 37 ~. I • M k <:] t> al ~· ~
QIO: 1337 .l. ar Previous Next lab 'lifllues Notes Calculator


17 visua l acu ity, red-g reen color blindness, optic neuritis, and retina l damage .
Optic neuritis Isoniazid Ethambutol Color blindness Visual acuity Retinal
18
19 C is not correct. 15% chose this.
20 The mechanism of action here is that of rifampin, another antimycobacte ria l drug. Rifampin's side effects profile
21
does include hepatotoxicity (though this is a ra re side effect), but also includes GI effects (nausea, vomiting,
diarr hea), hemato log ic side effects (thrombocytopenia, neutropenia, acute hemolytic anemia) and CNS eff ects
22 (headache, fever) . Rifampin does not, however, cause lupus- like symptoms as described in th is case .
23 Neutropenia Hepatotoxicity Rifampicin Thrombocytopenia Anemia Diarrhea Mechanism of action Adverse effect Nausea Hemolytic anemia Headache Vomiting

24 Side effect Adverse drug reaction Central nervous system Hemolysis Hematology Fever

25
Dis not correct. 11% chose this.
26 I nhibition of fo lic acid synthesis is the mechanism of action of dapsone. Dapsone can cause a range of side
27 effects including tachycard ia, fever, GI issues (nausea, vomiting, pancreatitis, abdomina l pa in) and
28 mononucleosis-l ike symptoms. It can also rarely cause drug-induced lupus-like symptoms; however, it is used
as a combination treatment for leprosy, not tubercu losis.
29
Dapsone Folic acid Pancreatitis Tuberculosis leprosy Tachycardia Nausea Abdominal pain Vomiting Fever Adverse drug reaction
30
31
32 Bottom Line:
33 The fi rst-line antitubercu lous drug regimen includes r ifampin, INH, pyrazinamide, and ethambutol. INH
34
functions by decreasing the synthesis of mycol ic acids; potential adverse effects include a lupus-l ike syndrome,
sensory neu ropathy, and drug-induced hepatitis.
o35 Pyrazinamide Ethambutol Rifampicin Mycolic acid Hepatitis Peripheral neuropathy Isoniazid
• 36
• 37 ~

6
lock
s
Suspend
0
End Block
Item: 34 of 3 7 ~ 1 • M k -<:J 1>- Jil ~· !:';-~
QIO: 1337 ..L ar Pre v ious Next Labfli!llues Not es Calcula t o r
A A

17
FA17 p 193.1
18
Isoniazid
19
20
MECHANISM l synthesis of mycolic acids. Bacterial catalase-
peroxidase (encoded by KatG) needed to
21
com·ert INH to acti,·e metabolite.
22
CliNICAL USE Mycobacterium tuberculosis. The only agent Different l~ H half-liYes in fast vs slow
23 used as solo prophylaxis against TB. Also used acetdators.
24 as monotherapy for latent TB.
25 ADVERSE EFFECTS Hepatotoxicity, P-450 inhibition, drug-induced ' ""'' Injures ~eurons and I Iepatocytes.
26 SLE, anion gap metabolic acidosis,' itamin
27 B6 deficiency (peripheral neuropathy,
sideroblastic anemia). dministcr with
28
pyridoxine (B6).
29
MECHANISM OF RESISTANCE Mutations leading to underexpression of KatC.
30
31
FA17 p 136.2
32
Mycobacteria Mycobacterium tuberculosis (TB, often resistant TB symptoms include fe,·er, night sweats,
33
to multiple drugs). weight loss, cough (nonproductive or
34
M avium- intracellulare (causes disseminated, producti,·e), hemoptysis.
o35 non-TB disease in AIDS; often resistant to Cord factor creates a "serpentine cord"
• 36 multiple drugs). Prophylaxis with azilhrOtn)Cin appearance in virulent J\1 tuberculosis
• 37 when C04+ count< 50 cells/mm 3. strains; acti,·ates macrophages (promoting

a
Lock
s
Suspend
8
End Bl ock
Item: 34 of 37 ~ .I • M k <:] t> al ~· ~
QIO: 1337 .l. ar Previous Next Lab 'lifllues Notes Calculator

17 FA17 p 136.2
18 Mycobacteria Mycobacterium tuberculosis (TB, often resistant TB symptoms include fever, night sweats,
19 to multiple drugs). weight loss, cough (nonproductive or
20 M avium-intracellulare (causes disseminated, productive), hemoptysis.
21 non-TB disease in AIDS; often resistant to Cord factor creates a "serpentine cord"
multiple drugs). Prophylaxis with azithromycin appearance in virulent M tuberculosis
22
when CD4+ count< 50 cells/mm 3. strains; activates macrophages (promoting
23 M scrofulaceum (cervical lymphadenitis in granuloma formation) and induces release of
24 children). T F-a. Sulfatides (surface glycolipids) inhibit
25 M marinwn (hand infection in aquarium phagolysosomal fusion.
26 handlers).
All mycobacteria are acid-fast organisms (pink
27
rods; arrows in rl1).
28
29
FA11 p 192.1
30
Anti mycobacterial drugs
31
BACTERIUM PROPHYLAXIS TREATMENT
32
M tuberculosis Isoniazid Rifampin, Isoniazid, Pyrazinamide,
33 Ethambutol (RIPE for treatment)
34
M avium- intracellulare Azithromycin, rifabutin More dmg resistant than M tuberculosis.
o35 Azithromycin or clarithromycin +ethambutol.
• 36 Can add rifabutin or ciproAoxacin .
• 37 M leprae /A Long-term treatment with dapsone and rifampin

6
lock
s
Suspend
0
End Block
Item: 34 of 37 ~ .I • M k <:] t> al ~· ~
QIO: 1337 .l. ar Previous Next Lab 'lifllues Notes Calculator

17 FA17 p 136.2
18 Mycobacteria Mycobacterium tuberculosis (TB, often resistant TB symptoms include fever, night sweats,
19 to multiple drugs). weight loss, cough (nonproductive or
20 M avium-intracellulare (causes disseminated, productive), hemoptysis.
21 non-TB disease in AIDS; often resistant to Cord factor creates a "serpentine cord"
multiple drugs). Prophylaxis with azithromycin appearance in virulent M tuberculosis
22
when CD4+ count< 50 cells/mm 3. strains; activates macrophages (promoting
23 M scrofulaceum (cervical lymphadenitis in granuloma formation) and induces release of
24 children). T F-a. Sulfatides (surface glycolipids) inhibit
25 M marinwn (hand infection in aquarium phagolysosomal fusion.
26 handlers).
All mycobacteria are acid-fast organisms (pink
27
rods; arrows in rl1).
28
29
FA11 p 192.1
30
Anti mycobacterial drugs
31
BACTERIUM PROPHYLAXIS TREATMENT
32
M tuberculosis Isoniazid Rifampin, Isoniazid, Pyrazinamide,
33 Ethambutol (RIPE for treatment)
34
M avium- intracellulare Azithromycin, rifabutin More dmg resistant than M tuberculosis.
o35 Azithromycin or clarithromycin +ethambutol.
• 36 Can add rifabutin or ciproAoxacin .
• 37 M leprae /A Long-term treatment with dapsone and rifampin

6
lock
s
Suspend
0
End Block
Item: 34 of 3 7 ~ 1 • M k -<:J 1>- Jil ~· !:';-~
QIO: 1337 ..L ar Pre v ious Next Labfli!llues Not es Calcula t o r
A A

17 FA17 p 192.1
18 Anti mycobacterial drugs
19 BACTERIUM PROPHYLAXIS TREATMENT

20 M tuberculosis Isoniazid Rifampin, Isoniazid, Pyrazinamide,


21
Ethambutol (RIPE for treatment)
22 M avium- intracellulare Azithromrcin, rifabutin lore drug resistant than .vi tuberculosis.
Azithromycin or clarithromycin + ethambutol.
23
Can add rifabutin or ciproAoxacin.
24
Mleprae '!A Long-term treatment with dapsone and rifampin
25
for tuberculoid form. dd clofaziminc for
26 lepromatous form.
27
MYCOBACTERIAL CELL Plasma
28 Cell wall membrane Interior of cell
r-------------~ ,------------------------- --.
29
30
mRNA
31
32
33
34
o35
ARABINOGALACTAN
polymerase
RNA
Rifabutin
R1fampin
J
. 36 SYNTHESIS INTRACELLUW
. 37 (irabinosyt transferase) (undear mechanism)

a
Lock
s
Suspend
8
End Bl ock
Item: 35 of 3 7 ~ 1 • M k -<:J 1>- Jil ~· !:';-~
QIO: 1333 ..L ar Pre v ious Next Lab fli!ltues Not es Calcula t o r

IAA]
A A

17
A 25-year-old woman with a history of asthma is brought to the emergency department by emergency
18 medical services (EMS) after ingesting a full bottle of theophylline in a su icide attempt. At presentation, she
19 is having a tonic-clonic seizure, is hypotensive to 80/40 mm Hg, is tachycardic to 160/min, and is tachypneic
20
to 30/min. The EMS personnel report that she has been seizing for at least 15 minutes.

21
What is the mechanism of action of the most appropriate drug to counteract her side effects from theophylline
22
intoxication?
23
:
24
A . Decreasing intracellular cAMP
25
26
B. Increasing intracellular cAMP through 132-adrenergic receptors
27 C. Increasing intracellular cAMP through nonselective adrenergic receptors
28 D. Inhibiting the Na+ - K+ -2CI- co-transporter
29
E. Inhibiting the Na+ -K+ -ATPase pump
30
31
32
33
34
• 35
. 36
. 37 •

a
Lock
s
Suspend
8
End Bl ock
Item: 35 of 37 ~. I • M k <:] t> al ~· ~
QIO: 1333 .l. ar Previous Next lab 'lifllues Notes Calculator


17
18 The correct answer is A. 61 °/o c hose this.
19
The drug theophylline is a phosphodiesterase inhibitor that leads to Bet A R('('t ptor Chnrt
the decreased hydro lysis of cAMP to adenosine monophosphate. An ;\eurot nosDiill<'r Rtsult of Ligaod
R~cept o.r Functioo
20 overdose of theophylli ne will the refore result in an elevated Bindint.
~) NE - Epi Enbance adenylyl -Juxtaglomcnllarc-d ls - > t rcoinrelcas c
21 intracellula r level of cAMP, w hich enhances ~ - ad renergic effects, cyclase. t <:AMP . Hea rt ~
t inotropy, chronotropy, and
dromolropy
22 leading to metabol ic complications such as hypo kalemia, ~' NE « Epi Enbrutc--eadeuylyl •Trac-h(!aJibtonchial muscle, uterine, and
cyclase, f c AMP vascular smooth muscle dilation
hyperg lycem ia, metabo lic acidosis, and arryt hmias. In add it ion, ~r Mucous clearance
23 ·At1-eriole-s iu muscles aud veins~ "
ad renergic- med iated vasod ilation can contr ibute to hypotension in vascub r fl ow to striated muscles via
vasodilation
24 these patients . ~ - B l ockers such as metop rolol may therefore be ·l gut motility & lone
-Urinary bla dder detrusor muscle rela."(ation
25 given to reduce cAMP levels th rough inactivation of adenylate -Skd clal musc-le ' promotes potassium
uptake, f contra.ctility,giycogeuolysis
26
cyclase . A cardioselective ~ - b l ocke r must be used in cases of -Liver -> f glycogenolysis and
gluconeogenesis
asthma to avoid inducing bronchial hyperreactivity. Initia l . fndllces ciliM)' musrli'relaxation for far
27 via:ioo
hypotension treatment wou ld be with rapid infusion of isoton ic • Thick<:ns saliva
p,
28 sa line. If the hypotension does not respond, a- ad renergic agon ists NE •>Ep i Enhance adenylyl -Fat cells- Activale-slipoly:sis
cyci.1Se, t cAMP

29 such as phenylephrine or norepineph rine can be used . ~ ­


Adrenergic antagon ists can also help imp rove hypotension. Treatment of her seizu re wou ld be with
30
benzodiazepines. The table details the functions of the th ree ~ - receptors .
31 Metoprolol Phenylephrine Theophylline Hypokalemia Metabolic acidosis Norepinephrine Hypotension Vasodilation Hyperglycemia Phosphodiesterase inhibitor

32 Asthma Adenylyl cyclase Adenosine Hydrolysis Cyclic adenosine monophosphate Benzodiazepine Phosphodiesterase Adenosine monophosphate Saline (medicine)
33 Drug overdose Epileptic seizure Adrenergic receptor Acidosis Intracellular Metabolism Bronchus Enzyme inhibitor
34
B is not correct. 14% chose this .
35
~ -Agon ists
such as albuterol wou ld potentiate the effects of theophyll ine by activating adenylate cyclase and
• 36 increasing the conversion of ATP to cAMP .
• 37 ~
Theophylline Adenylyl cyclase Salbutamol Adenosine triphosphate Agonist

6
lock
s
Suspend
0
End Block
Item: 35 of 37 ~. I • M k <:] t> al ~· ~
QIO: 1333 .l. ar Previous Next lab 'lifllues Notes Calculator
. . .

17
B is not correct. 14% chose this.
18
~-Agonists such as albuterol wou ld potentiate the effects of theophylline by activating adenylate cyclase and
19
increasing the conversion of ATP to cAMP .
20 Theophylline Adenylyl cyclase Salbutamol Adenosine triphosphate Agonist

21
C is not correct. 15% chose this.
22
Epinephrine would potentiate the effects of theophylline by activating adenylate cyclase and increasing cAMP
23 for mation.
24 Theophylline Epinephrine Adenylyl cyclase Cyclic adenosine monophosphate

25 D is not correct. 4°/o chose this.


26 Fu rosemide is a diuretic un related to theophylline overdose. It works by inhibiting the Na+ -K+-2cl- cotransport
27 system of the thick ascend ing loop of Hen le.
Furosemide Diuretic Theophylline loop of Henle Ascending limb of loop of Henle Drug overdose Co-transport
28
29 E is not correct. 6°/o chose this.
30 Digoxin inhibits the Na+-K+-ATPase pump . It is used to increase myocard ial contractility in patients with
31
congestive heart failure. It is un related to theophylline overdose.
Theophylline Digoxin Heart failure Congestive heart failure Contractility Ejection fraction Drug overdose
32
33
34 Bottom Line:
35 Theophylline ove rdose can lead to a toxic accumu lation of intrace llula r cAMP, and ~-b l ockers can be given as
• 36 an antidote to reduce cAMP levels by reducing the activation of adenylate cyclase .
Theophylline Adenylyl cyclase Cyclic adenosine monophosphate Intracellular Drug overdose Toxicity
• 37 ~

6
lock
s
Suspend
0
End Block
Item: 35 of 3 7 ~ 1 • M k -<:J 1>- Jil ~· !:';-~
QIO: 1333 ..L ar Pre v ious Next Lab fli!ltues Not es Calcula t o r
A A

17
FA17 p649.1
18
Asthma drugs Bronchoconstriction is mediated b~ (I) inOammatory processes and (2) parasympathetic tone;
19
therapy is directed at these 2 pal hways.
20
~-agonists Albuterol-relaxes bronchial smooth muscle (short acting J3z-agonist). Used during acute
21
exacerbation.
22
Salmeterol, formoterol - long-acting agents for prophylaxis. dverse effects are tremor and
23 arrh' thmia.
24
Inhaled Fluticasone, budesonide - inhibitthc synthesis of virtually all cytokines. Inacti,·atc NF-KB, the
25 corticosteroids transcription factor that induces production ofT 'F-<X and other inflammatory agents. 1st-line
26 therapy for chronic asthma. Vlay cause oral thrush.
27 Muscarinic Tiotropium, ipratropium-competitivel) bloc!.. muscarin ic receptors, preventing
28 antagonists bronchoconstriction. Also used for COPD. Tiotropium is long acting.
29 Antileukotrienes Montelukast, zafirlukast- bloc!.. leul..otricnc Exposure to antigen
30 receptors (CysLT1). Especia lly good for (dust pollen, etc)
aspirin-induced and exercise-induced asthma.
31
Zileuton -5-lipoxygcnasc p<llhway inhibitor. t-0- Avoidance
32 Blocks conversion of arachidonic acid to
33 leukotrienes. Hepatotoxic.
34 Anti-lgE monoclonal Omalizumab-binds mostly unbound serum Antigen and lgE t-0- Omatizumab
on mast cells
35 therapy lgE and blocks binding to FctRI. Used in
• 36 allergic asthma with t lgE levels resistant to .....o-- Steroids
• 37
inhaled steroids and long-acting J3z-agonisls.

a
Lock
s
Suspend
8
End Bl ock
Item: 35 of 37 ~ .I • M k <:] t> al ~· ~
QIO: 1333 .l. ar Previous Next lab 'lifllues Notes Calculator

• •
17
FA17p237.1
18
19 p-blockers Acebutolol, atenolol, betaxolol, bisoprolol, carvedilol, esmolol, labetalol, metoprolol, nadolol,
20
nebivolol, pindolol, propranolol, timoloL
APPLICATIOtl ACTIONS tiOTES/EXAMPLES
21
Angina pectoris ! heart rate and contractility, resulting in ! 0 2
22
consumption
23
Myocardial infarction ! mortality
24
Supraventricular ! AV conduction ,·elocity (class II Metoprolol, esmolol
25
tachycardia antiarrhythmic)
26
Hypertension ! cardiac output, l renin secretion (clue to
27
~ 1 -receptor blockade on JGA cells)
28
Heart failure ! mortality (bisoprolol, carvedilol, metoprolol)
29
Glaucoma ! production of aqueous humor Timolol
30
Variceal bleeding ! hepatic venous pressure gradient and portal adolol, propranolol
31
hypertension
32
ADVERSE EFFECTS Erectile dysfunction, cardiovascular adverse Use with caution in cocaine users due to risk
33 effects (bradycardia, AV block, HF), CNS of unopposed ex-adrenergic receptor agonist
34 adverse effects (seizures, sedation, sleep activity
35 alterations), dyslipidemia (metoprolol), and
• 36 asthma/COPD exacerbations
• 37 SELECTIVITY ~ 1 -selective antagonists (~ 1 > ~) - acebutolol Selective antagonists mostly go from A to M (~ 1 •

6
lock Suspend
s 0
End Block
Item: 35 of 3 7 ~ 1 • M k -<:J 1>- Jil ~· !:';-~
QIO: 1333 ..L ar Pre v ious Next Lab fli!ltues Not es Calcula t o r
A A

17 ~ 1 -receptor blockade on JGA cells)


18 Heart failure ! mortality (bisoprolol, carvedilol, metoprolol)
19 Glaucoma ! production of aqueous humor Timolol
20 Variceal bleeding ! hepatic ,·enous pressure gradient and portal 'adolol, propranolol
21 hypertension
22 ADVERSE EFFECTS Erectile dysfunction, cardio,ascular ad' erse Use '' ith caution in cocaine users due to risk
23 effects (bradrcardia, AV block, II F'), C . of unopposed a -adrenergic receptor agonist
24 ad,·erse effects (seizures, sedation, sleep activity
alterations), dyslipidcmia (metoprolol), and
25
asthma/CO PO exacerbations
26
SELECTIVITY ~ 1 -selective antagonists (~ 1
> j3z)- acebutolol Selective antagonists mostly go from A to .:\ I (~ 1
27
(partial agonist), atenolol, betaxolol, bisoprolol, with lst ha If of alphabet)
28 esmolol, metoprolol
29
Nonselecti,·e antagonists (~ 1 =j3z)-nadolol, 1\onselective antagon ists mostly go from 1\ to Z
30 pindolol (partial agonist), propranolol, t imolol (~2 with 2nd half of alphabet)
31 Nonselective a- and ~-a ntagonists-ca rvcdi l ol , onselective a- and ~-antagon i sts have modified
32 labctalol suffixes (instead of "-olol")
33 Nebivolol combi nes ca rdiac-selective
34 ~!"adrenergi c blockade with stimulation of

35 ~3 -receptors (activate nitric oxide synthase in


the ,-asculature and ! SVR)
• 36
• 37

a
Lock
s
Suspend
8
End Bl ock
Item: 36 of 3 7 ~ 1 • M k -<:J 1>- Jil ~· !:';-~
QIO: 170 2 ..L ar Pre v ious Next Labfli!llues Notes Calcula t o r
A A

17
An acid-fast stain of a sputum sample t aken from an asymptomatic 49-yea r- old woman who recently
18 imm ig rated to the United Stat es is shown in t he image.
19
20
21
22
23
24
25
26

27
28
29
30
31 Image courtesy of CDC/Or. George P. Kubica

32
33 Wh ich of the fol lowing shou ld be ad m inistered to people who have come in contact w it h th is patient and who have
a positive tubercu lin test but negat ive findings on X-ray of the chest?
34
35 :

• 36
A. Ethambutol and clarith romycin
• 37 B. Isoniazid and pyridoxine

a
Lock
s
Suspend
8
End Bl ock
Item: 36 of 3 7 ~ 1 • M k -<:J 1>- Jil ~· !:';-~
QIO: 170 2 ..L ar Pre v ious Next Labfli!llues Notes Calcula t o r

17
18
19
20
21
22
23
24
25
26
I mage courtesy of CDC/Or. George P. Kubica
27
28
Which of the fol lowing should be administered to people who have come in contact with this patient and who have
29 a positive tuberculin test but negative findings on X-ray of the chest?
30
:
31 A. Ethambutol and clarith romycin
32
B. I soniazid and pyridoxine
33
C. I soniazid, pyrazinamide, rifampin
34
35 D. Levofloxacin
• 36 E. Rifampin and pyridoxine
• 37

a
Lock
s
Suspend
8
End Bl ock
Item: 36 of 37 ~. I • M k <:] t> al ~· ~
QIO: 1702 .l. ar Previous Next lab 'lifllues Notes Calculator

17
18 The correct answer is B. 59°/o chose this.
19 The image revea ls Mycobacterium tuberculosis infection . A person who comes into contact with this infection
and is subsequently found to have a positive purified protein derivative (PPD) test, a negative chest X-ray, and
20
no signs or symptoms of active tuberculosis is considered to have latent tuberculosis . Although rifampin is
21 considered the best antitubercu lous agent, ison iazid monotherapy is the most common treatment for
22 asymptomatic patients with a positive PPD (latent TB) . A 6-month course of isoniazid prevents active
tubercu losis (TB) in 90% of patients for at least 20 yea rs. Isoniazid blocks mycolic acid cell wa ll synthesis and is
23
bactericida l for rapidly multiplying organ isms . Vitamin 8 6 (pyridoxine) is important for heme synthesis,
24 transamination, and neurotransmitter synthesis. It is given with isoniazid to prevent neurotoxicity and
25 sideroblastic anemia . Isoniazid is the most common cause of functiona l pyridoxine deficiency. Other acceptable
26 therapies by the CDC are listed in the table . Of note, the newer combination of isoniazid, pyr idoxine and
r ifapentine administered weekly for 3 months with di rectly observed therapy is as effective as other regimens
27
and is more likely to be completed than the standa rd regimen of 9 months of isoniazid daily without directly
28 observed therapy.
29 Isoniazid Rifapentine Mycolic acid Mycobacterium tuberculosis Rifampicin Pyridoxine Heme Sideroblastic anemia latent tuberculosis Neurotransmitter

30 Tuberculosis Cell wall Mycobacterium Chest radiograph Anemia DOTS {Directly Observed Treatment, Short-Course) Bactericide Protein Asymptomatic B vitamins

31 Neurotoxicity Mantoux test Centers for Disease Control and Prevention Combination therapy Vitamin Vitamin B12 X-ray Infection

32
latent TB Infection Treatment Options
33
Drug Duration Preferred Population
34 lsoniazkl + 9 months (daily OR twice HIVIAJDS, children 2-11 years old,
Pyridoxine weekly") pregnant women
35
lsoniazkl + 6 months (daily OR twice
36 Pyridoxine weekly")
• 37 ~ lsoniazkl + 3 months (once weekly") >12 years of age without HIVIAIDS.

6
lock
s
Suspend
0
End Block
Item: 36 of 37 ~. I • M k <:] t> al ~· ~
QIO: 1702 .l. ar Previous Next lab 'lifllues Notes Calculator

17
- - - - - -- --- -- --- -- -- · - - - - -
asymptomatic patients with a positive PPD (latent TB) . A 6-month course of isoniazid prevents active
18 tubercu losis (TB) in 90% of patients for at least 20 yea rs. Isoniazid blocks mycolic acid cell wa ll synthesis and is
19
bactericida l for rapidly multiplying organ isms . Vitamin 8 6 (pyridoxine) is important for heme synthesis,
transamination, and neurotransmitter synthesis. I t is given with isoniazid to prevent neurotoxicity and
20 sideroblastic anemia . Isoniazid is the most common cause of functiona l pyridoxine deficiency. Other acceptable
21 therapies by the CDC are listed in the table . Of note, the newer combination of isoniazid, pyr idoxine and
22 r ifapentine administered weekly for 3 months with di rectly observed therapy is as effective as other regimens
and is more likely to be completed than the standa rd regimen of 9 months of isoniazid daily without directly
23
observed therapy.
24 Isoniazid Rifapentine Mycolic acid Mycobacterium tuberculosis Rifampicin Pyridoxine Heme Sideroblastic anemia latent tuberculosis Neurotransmitter

25 Tuberculosis Cell wall Mycobacterium Chest radiograph Anemia DOTS {Directly Observed Treatment, Short-Course) Bactericide Protein Asymptomatic B vitamins

26 Neurotoxicity Mantoux test Centers for Disease Control and Prevention Combination therapy Vitamin Vitamin B12 X-ray Infection

27
latent TB Infection Treatment Options
28
Drug Duration Preferred Population
29
lsoniazkl + 9 months (daily OR twice HIVIAJDS, children 2-11 years old,
30 Pyridoxine weekly") pregnant women

31 lsoniazkl + 6 months (daily OR twice


Pyridoxine weekly")
32
lsoniazkl + 3 months (once weekly") :0:12 years of age without HIVIAIDS,
Pyridoxine + pregnant women, women expecting to
33 Rifapentine become pregnant w~h the treatment
period
34
Rifampin 4 months (daily)
35
"Must use d~rectly observed therapy (Don
36 Adapted from: Treatment Regimens for Latent TB Infection (LTBI). (2016).
Retrieved October 30, 2016, from http:/l www.cdc.gov/tbltopicllreatrnent/ltbi.htm
• 37 ~

6
lock
s
Suspend
0
End Block
Item: 36 of 37 ~. I • M k <:] t> al ~· ~
QIO: 1702 .l. ar Previous Next lab 'lifllues Notes Calculator


17
A is not correct. 3°/o chose this.
18
Ethambutol is an antituberculous drug used as part of the four-drug regimen for pulmonary tuberculosis. It is
19 also sometimes used with clar ithromycin and azithromycin to t reat Mycobacterium avium complex (MAC)
20 infection.
Azithromycin Ethambutol Clarithromycin Tuberculosis Mycobacterium avium complex Mycobacterium avium-intracellulare infection Mycobacterium Infection
21
22 C is not correct. 20% chose this.
23 Triple therapy with isoniazid, pyrazinamide, and rifampin is used to treat tuberculosis but is not ind icated in this
24 situation for prophylaxis in asymptomatic patients .
Isoniazid Pyrazinamide Rifampicin Tuberculosis Asymptomatic Preventive healthcare
25
26 D is not correct. 3°/o chose this.
27 Levofloxacin or other fluoroqu inolones can be used to treat mu ltid r ug-resistant (M DR) tubercu losis-infection
with an organism otherwise resistant to isoniazid and rifampin.
28 Isoniazid levofloxacin Rifampicin Quinolone Multiple drug resistance Organism
29
E is not correct. 15% chose this.
30
Rifampin is the most potent antituberculous agent avai lable. Rifampin blocks DNA-dependent RNA polymerase,
31
preventing RNA synthesis. Although it is a better agent than isoniazid for preventing active tubercu losis
32 infection, it has a significant risk of liver toxicity that outweighs its benefits in patients who have been exposed
33 to tubercu losis but are asymptomatic. Vitamin B6 (pyroxidine) is given with isoniazid to prevent neurotoxicity,
an adverse effect of isoniazid therapy. Like rifampin, the other major adverse effect of ison iazid is
34
hepatotoxicity. Rifampin, however, does not cause periphera l neu ropathy, and thus coadministration of vitamin
35 B6 is unnecessary.
36 Isoniazid Hepatotoxicity Rifampicin Peripheral neuropathy Tuberculosis Vitamin 812 Asymptomatic Neurotoxicity B vitamins liver Transcription (genetics)

• 37 ~
RNA polymerase Vitamin Adverse effect Toxicity RNA Infection

6
lock
s
Suspend
0
End Block
Item: 36 of 37 ~. I • M k <:] t> al ~· ~
QIO: 1702 .l. ar Previous Next lab 'lifllues Notes Calculator

17
p p y y p p
Isoniazid Pyrazinamide Rifampicin Tuberculosis Asymptomatic Preventive healthcare
18
19
D is not correct. 3°/o chose this.
Levofloxacin or other fluoroqu inolones can be used to treat mu lt idr ug-resistant (MDR) tubercu losis-infection
20
with an organism otherwise resistant to isoniazid and rifampin.
21 Isoniazid levofloxacin Rifampicin Quinolone Multiple drug resistance Organism

22
E is not correct. 15% chose this.
23
Rifampin is the most potent antituberculous agent avai lable. Rifampin blocks DNA-dependent RNA polymerase,
24 preventing RNA synthesis. Although it is a better agent than isoniazid for preventing active tuberculosis
25 infection, it has a significant risk of liver toxicity that outweighs its benefits in patients who have been exposed
26 to tuberculosis but are asymptomatic. Vitamin B6 (pyroxidine) is given with isoniazid to prevent neurotoxicity,
an adverse effect of isoniazid therapy. Like rifampin, the other major adverse eff ect of ison iazid is
27
hepatotoxicity. Rifampin, however, does not cause peripheral neu ropathy, and thus coadministration of vitamin
28 B6 is unnecessary.
29 Isoniazid Hepatotoxicity Rifampicin Peripheral neuropathy Tuberculosis Vitamin B12 Asymptomatic Neurotoxicity B vitamins liver Transcription (genetics)

30 RNA polymerase Vitamin Adverse effect Toxicity RNA Infection

31
32
Bottom Line:
33
Current guidel ines recommend that isoniazid be used by itself as antimicrobial prophylaxis against
34 Mycobacterium tuberculosis in those who have verified exposure (PPD positive) but no active symptoms. Its
35 neurotoxicity can be minimized by coadministering pyr idoxine (vitamin B6 ).
Isoniazid Mycobacterium tuberculosis Tuberculosis Pyridoxine Mycobacterium Neurotoxicity Preventive healthcare Antimicrobial
36
• 37 ~

6
lock
s
Suspend
0
End Block
Item: 36 of 3 7 ~ 1 • M k -<:J 1>- Jil ~· !:';-~
QIO: 170 2 ..L ar Pre v ious Next Labfli!llues Notes Calcula t o r
A A

17
FA17 p 193.1
18
Isoniazid
19
MECHANISM l synthesis of mycolic acids. Bacterial catalase-
20
peroxidase (encoded by KatG) needed to
21 com·ert INI-I to acti,·e metabolite.
22 CliNICAL USE 1\lycobacterium tuberculosis. The only agent Different 1:-JH half-liYes in fast vs slow
23 used as solo prophylaxis against TB. Also used acetdators.
24 as monotherapy for latent T B.
25 ADVERSE EFFECTS Hepatotoxicity, P-450 inhibition, drug-induced ' "''' Injures ~eurons and llepatocytes.
26 SLE, anion gap metabolic acidosis,' itamin
B6 deficiency (peripheral neuropathy,
27
sideroblastic anem ia). dminister with
28
pyridoxine (B6).
29
MECHANISM OF RESISTANCE Mutations leading to underexpression of KatC.
30
31
FA17 p 63.4
32 Vitamin 8 6 (pyridoxine)
33 FUNCTION Converted to pyridoxal phosphate (PLP), a cofactor used in transamination (eg, ALT and AST),
34 decarboxylation reactions, glycogen phosphorylase. Synthesis of cystathionine, heme, niacin,
35 histamine, and neurotransmitters including serotonin, epinephrine, norepinephrine ( 1£ ),
36 dopamine, and CABA.
• 37 DEFICIENCY Convulsions, hyperirritability, peripheral neuropathy (deficiency inducible by isoniazid and oral

a
Lock
s
Suspend
8
End Bl ock
Item: 36 of 3 7 ~ 1 • M k -<:J 1>- Jil ~· !:';-~
QIO: 170 2 ..L ar Pre v ious Next Labfli!llues Notes Calcula t o r
A A

17
FA17 p 63.4
18
Vitamin 8 6 (pyridoxine)
19
FUNCTION Converted to pyridoxal phosphate (PLP), a cofactor used in transamination (eg, ALT and AST),
20
decarboxylation reactions, glycogen phosphorylase. Synthesis of cystathionine, heme, niacin,
21
histamine, and neurotransmitters including serotonin, epinephrine, norepinephrine (NE),
22 dopamine, and GABA.
23 DEFICIENCY Convulsions, hr perirritability, peripheral neuropathy (deficiency inducible by isoniazid and oral
24 contracepti,·es), sideroblastic anemias due to impaired hemoglobin synthesis and iron excess.
25
26 FA17 p 192.2

27 Rifamycins Ri fampin, ri fabutin.


28 MECHANISM Inhibit ON -dependent R l polymerase. Rifampin's 4 R'!>:
29 CLINICAL USE J\lycobacterium tuberculosis; delay resistance R1 A polymerase inhibitor
to dapsone when used for leprosy. Used Ramps up microsomal cytochrome P-450
30
for meningococcal prophylaxis and Red/orange body Auids
31 Rapid resistimce if used alone
chemoprophylaxis in contacts of children with
32 1-laemophilus illf/uel!;:ae type B. Rifampin ramps up cytochrome P-450, but
33 rifabutin does not.
ADVERSE EFFECTS Minor hepatotoxicity and drug interactions
34 (t cytochrome P-450); orange body Auids
35 (nonhazardous side effect). Rifabut in favored
36 0\·er rifampin in patients with Ill Y infection
. 37
due to less cytochrome P-450 stimulation.

a
Lock
s
Suspend
8
End Bl ock
Item: 37 of 3 7 ~ 1 • M k -<:J 1>- Jil ~· !:';-~
QIO: 45 79 ..L ar Pre v ious Next Lab fli!ltues Not es Calcula t o r

IAA]
A A

17
A 54-year-old man comes to the emergency room complaining of shortness of breath, wheezing, increased
18 expiratory time, and coughing up clear sputum. He does not have a history of smoking. He is given a
19 medication to treat th is acute episode .
20
21 What is the mechanism of the medication most likely prescribed?
22 :
23 A. 132 -Receptor antagonist
24 B. I nhibition of cytokine synthesis
25
C. Inhibition of mast cell degranulation
26
D. Long-acting 132 -receptor agonist
27
28 E. Short-acting 13rreceptor agonist
29
30
31
32
33
34
35
36
. 37 •

a
Lock
s
Suspend
8
End Bl ock
Item: 37 of 37 ~. I • M k <:] t> al ~· ~
QIO: 4579 .l. ar Previous Next lab 'lifllues Notes Calculator


17 The correct answer is E. 76°/o chose this .
18 The patient is having an acute asthma attack. Treatments include a short-acting ~ -receptor agonist,
methylxanth ines, a muscarinic antagonist, or steroids. The med ication with the quickest onset and relief of
19
symptoms would be a short-acting ~r receptor agonist such as albuterol. Long-acting ~-agon i sts and mast cell
20 mediator release inhibitors such as cromolyn are of no benefit in this situation . Cromolyn is used only for
21 asthma prophylaxis and is not effective du r ing an acute episode.
Mast cell Asthma Cromoglicic acid Receptor antagonist Xanthine Muscarinic antagonist Salbutamol Agonist Muscarinic acetylcholine receptor Steroid
22
Preventive healthcare Pharmaceutical drug
23
24 A is not correct. 8°/o chose this.
25 A ~rreceptor antagonist wou ld be contraind icated in this situation and would worsen the symptoms.
Antagonist Receptor antagonist Contraindication
26
27 B is not correct. 4°/o chose this.
28 Steroids are used in both the acute and chronic settings to treat asthma. Steroids block the production of
29 cytokines by inactivating NF-kB, wh ich is a transcription factor that induces the production of a wide range of
proinflammatory cytokines. However, the onset of action of steroids is slower than that of a short-acting ~r
30
receptor agonist, and they would not be the most important treatment for acute rel ief of this patient's
31 symptoms .
32 Transcription factor Asthma Cytokine Steroid Agonist Transcription (genetics) Anabolic steroid Proinflammatory cytokine

33 C is not correct. 5°/o chose this.


34 Mast cel l med iator release inhibitors such as cromolyn are of no benefit in th is situation . Cromo lyn is used for
35 asthma prophylaxis and is not effective du r ing an acute episode.
Mast cell Cromoglicic acid Asthma Preventive healthcare
36
37 ~
D is not correct. 7°/o chose this.

6
lock
s
Suspend
0
End Block
Item: 37 of 37 ~. I • M k <:] t> al ~· ~
QIO: 4579

17
.'
.l. ar Previous Next

Antagonist Receptor antagonist Contraindication


.. . .
lab 'lifllues Notes Calculator
. .
18
B is not correct. 4°/o chose this.
19
Ste roids are used in both the acute and ch ronic settings to treat asth ma. Steroids block the prod uction of
20
cytokines by inactivating NF-kB, wh ich is a transcription factor that induces the production of a wide range of
21 proinflammato ry cytokines. However, the onset of action of steroids is slower than that of a short-acting ~r
22 receptor agonist, and they wou ld not be the most important treatment for acute rel ief of this patient's
23
symptoms .
Transcription factor Asthma Cytokine Steroid Agonist Transcription (genetics) Anabolic steroid Proinflammatory cytokine
24
25 C is not correct. 5°/o chose this.
26
Mast cel l med iato r release inhibitors such as cromolyn are of no benefit in th is situation . Cromo lyn is used for
asthma prophylaxis and is not effective du r ing an acute episode.
27 Mast cell Cromoglicic acid Asthma Preventive healthcare
28
D is not correct. 7°/o chose this.
29
A long-acting ~r receptor agon ist is of no use in an acute setting and is contraindicated.
30 Contraindication Agonist
31
32
Bottom Line:
33
34 Treatment for acute asthma attack includes a short-acting ~- receptor agon ist, methylxanthines, a musca rinic
antagon ist, or steroids. Albute rol, a short-acting ~rrecepto r agonist, has the quickest onset and rel ief of
35
symptoms .
36 Asthma Muscarinic antagonist Agonist Receptor antagonist Xanthine Steroid Muscarinic acetylcholine receptor Salbutamol Anabolic steroid Antagonist

37

6
lock
s
Suspend
0
End Block
Item: 37 of 3 7 ~ 1 • M k -<:J 1>- Jil ~· !:';-~
QIO: 45 79 ..L ar Pre v ious Next Lab fli!ltues Not es Calcula t o r
A A

17
FA17 p649.1
18
19
Asthma drugs Bronchoconstriction is mediated b) (I) inAammatory processes and (2) parasympathetic tone;
therapy is directed at these 2 pathways.
20
~-agonists Albuterol-relaxes bronchial smoot h muscle (~hort acting f3z-agonist). Used during acute
21
exacerbation.
22
Salmeterol, formoterol -long-acting agents for prophylaxis. Adverse effects are tremor and
23 arrln thmia.
24
Inhaled Fluticasone, budesonide -inhibit the synthesis of ,·irtuallr all cytokines. lnacti,Cite ' F'-KB, the
25 corticosteroids transcription factor that induces production ofT ' F-a and other inAammatory agents. 1st-line
26 therapy for chronic asthma. ~ l ay cause oral thrush.
27 Muscarinic Tiotropium, ipratropium-compet itively block muscarinic receptors, preventing
28 antagonists bronchoconstriction. Also used for COPO. Tiotropium is long acting.
29 Antileukotrienes Montelukast, zafirlukast- block lcukotricnc Exposure to an~gen
30 receptors (CysLTl). Especially good for (dust pollen, etc)

31
aspirin-induced and exercise-induced asthma.
Zileuton -5-lipoxygenase pathway inh ibitor. ~ Avoidance
32
Blocks conversion of arachidonic acid to
33 leukotrienes. Hepatotoxic.
34 Antigen and lgE ~ Omahzumab
Anti-lgE monoclonal Omalizumab-binds mostly unbound serum on mast cells
35 therapy lgE and blocks binding to FceRI. Used in
36 allergic asthma with t lgE levels resistant to ~ Steroids
37 inhaled steroids and long-acting f3z-agonists.

a
Lock
s
Suspend
8
End Bl ock

You might also like